Case Files Cardiology pdf am-medicine com [PDF] | Online Book Share (2024)

431 Pages • 147,587 Words • PDF • 25.8 MB

+ Case+ files+ Cardiology

Uploaded at 2021-07-02 11:41

Report DMCA

PREVIEW PDF

CASE FILES

®

Cardiology Eugene C. Toy, Md Vice Chair of Academic Affairs and Clerkship Director Director of Division of General Obstetrics-Gynecology Department of Obstetrics and Gynecology The Methodist Hospital Houston, Texas Clinical Professor and Clerkship Director Department of Obstetrics and Gynecology University of Texas Medical School at Houston Houston, Texas Associate Clinical Professor Weill Cornell College of Medicine New York, New York

Michael d . Faulx, Md Assistant Professor of Medicine Case Western Reserve University Lerner College of Medicine Cleveland, Ohio Associate Program Director Internal Medicine Residency Cleveland Clinic Cleveland, Ohio Staff Cardiologist, Section of Clinical Cardiology Miller Family Heart and Vascular Institute Cleveland Clinic Cleveland, Ohio

New York Chicago San Francisco Athens London Madrid Mexico City Milan New Delhi Singapore Sydney Toronto

Copyright © 2015 by McGraw-Hill Education. All rights reserved. Except as permitted under the United States Copyright Act of 1976, no part of this publication may be reproduced or distributed in any form or by any means, or stored in a database or retrieval system, without the prior written permission of the publisher, with the exception that the program listings may be entered, stored, and executed in a computer system, but they may not be reproduced for publication. ISBN: 978-0-07-179920-1 MHID: 0-07-179920-6 The material in this eBook also appears in the print version of this title: ISBN: 978-0-07-179919-5, MHID: 0-07-179919-2. eBook conversion by codeMantra Version 1.0 All trademarks are trademarks of their respective owners. Rather than put a trademark symbol after every occurrence of a trademarked name, we use names in an editorial fashion only, and to the bene t of the trademark owner, with no intention of infringement of the trademark. Where such designations appear in this book, they have been printed with initial caps. McGraw-Hill Education eBooks are available at special quantity discounts to use as premiums and sales promotions or for use in corporate training programs. To contact a representative, please visit the Contact Us page at ww.mhprofessional.com. Notice Medicine is an ever-changing science. As new research and clinical experience broaden our knowledge, changes in treatment and drug therapy are required. The authors and the publisher of this work have checked with sources believed to be reliable in their efforts to provide information that is complete and generally in accord with the standard accepted at the time of publication. However, in view of the possibility of human error or changes in medical sciences, neither the editors nor the publisher nor any other party who has been involved in the preparation or publication of this work warrants that the information contained herein is in every respect accurate or complete, and they disclaim all responsibility for any errors or omissions or for the results obtained from use of the information contained in this work. Readers are encouraged to con rm the information contained herein with other sources. For example, and in particular, readers are advised to check the product information sheet included in the package of each drug they plan to administer to be certain that the information contained in this work is accurate and that changes have not been made in the recommended dose or in the contraindications for administration. This recommendation is of particular importance in connection with new or infrequently used drugs. TERMS OF USE This is a copyrighted work and McGraw-Hill Education and its licensors reserve all rights in and to the work. Use of this work is subject to these terms. Except as permitted under the Copyright Act of 1976 and the right to store and retrieve one copy of the work, you may not decompile, disassemble, reverse engineer, reproduce, modify, create derivative works based upon, transmit, distribute, disseminate, sell, publish or sublicense the work or any part of it without McGraw-Hill Education’s prior consent. You may use the work for your own noncommercial and personal use; any other use of the work is strictly prohibited. Your right to use the work may be terminated if you fail to comply with these terms. THE WORK IS PROVIDED “AS IS.” McGRAW-HILL EDUCATION AND ITS LICENSORS MAKE NO GUARANTEES OR WARRANTIES AS TO THE ACCURACY, ADEQUACY OR COMPLETENESS OF OR RESULTS TO BE OBTAINED FROM USING THE WORK, INCLUDING ANY INFORMATION THAT CAN BE ACCESSED THROUGH THE WORK VIA HYPERLINK OR OTHERWISE, AND EXPRESSLY DISCLAIM ANY WARRANTY, EXPRESS OR IMPLIED, INCLUDING BUT NOT LIMITED TO IMPLIED WARRANTIES OF MERCHANTABILITY OR FITNESS FOR A PARTICULAR PURPOSE. McGraw-Hill Education and its licensors do not warrant or guarantee that the functions contained in the work will meet your requirements or that its operation will be uninterrupted or error free. Neither McGraw-Hill Education nor its licensors shall be liable to you or anyone else for any inaccuracy, error or omission, regardless of cause, in the work or for any damages resulting therefrom. McGraw-Hill Education has no responsibility for the content of any information accessed through the work. Under no circ*mstances shall McGraw-Hill Education and/or its licensors be liable for any indirect, incidental, special, punitive, consequential or similar damages that result from the use of or inability to use the work, even if any of them has been advised of the possibility of such damages. This limitation of liability shall apply to any claim or cause whatsoever whether such claim or cause arises in contract, tort or otherwise.

d ed ic at io n

To the amazing members o our Cambodia medical mission team o 2014, who sacrif cially shared their talents and compassion to help hundreds o villagers in the province o Kratie, Kingdom o Cambodia. To my daughter A llison, our team leader, who is already showing wisdom at such a young age and is my hero. To my wi e Terri, who served as team administrator and “team mom”, and is an organizational genius. To our super nurses, Erin, N atalie, and Elizabeth, who went above and beyond each day, tending to the medical and spiritual needs o our patients. To A my, who organized and updated our pharmacy, keeping hundreds o dispenses straight. To Khai and Meredith, our gi ted counselors, who shared the love o God each day; A nd f nally to A rchie and his amily, his leaders, his interpreters, and his church, who blazed the trail, ministered to us, and who continue God’s work in the lovely Kingdom o Cambodia. —ECT To my lovely wi e A shley and my sons, Jackson and Gregory, or making it so easy to f nd balance and happiness in my li e. You are the reason I can’t wait to get home every evening. To the medical students, residents, and ellows at Case Western Reserve University Lerner College o Medicine and Cleveland Clinic, or making it so easy to f nd balance and happiness in my career. You are the reason I can’t wait to get to work every morning. —MDF

This page intentionally left blank

c o n t en t s

Contributors / vii A cknowledgments / xi Introduction / xiii Section I How to Approach the Cardiology Patient ................................................................ 1 P r 1. c r v l rH ry Phy l ex ...................................... 2 P r 2. a ppr h h el r r gr (ec G) ................................................ 22 P r 3. c r v l r Pr v r Pr r ................................................. 40 Section II Clinical Cases .......................................................................................................... 57 t h r yc s r ............................................................................................. 59 c 1–5. c r ry V l rd ...................................................... 59 c 6–10. s r r lH r d ............................................................... 125 c 11–15. Rhy h d r r .......................................................................... 177 c 16–20. P p d r r ............................................................................. 237 c 21–25. c l c pl .............................................................. 287 c 26–30. o h r c r d ................................................................. 335 Section III Listing of Cases ..................................................................................................... 399 L g yc n r ....................................................................................... 401 L g y d r r (a lph l) ....................................................................... 402 Index / 403

This page intentionally left blank

c o n t Ribu t o Rs

Mosi Bennett, Md , Phd Advanced Heart Failure and Transplant Cardiologist Minneapolis Heart Institute at Abbott Northwestern Hospital Minneapolis, Minnesota A cute decompensated heart ailure E mon Cronin, Md , MRCPI Staff Physician, Cardiology and Cardiac Electrophysiology Hartford Hospital Hartford, Connecticut A trial f brillation Sudden cardiac death Mohame Elshazly, Md Fellow, Cardiovascular Medicine Miller Family Heart and Vascular Institute Cleveland Clinic Cleveland, Ohio A cute type A aortic dissection Michael d . Faulx, Md Assistant Professor of Medicine Case Western Reserve University Lerner College of Medicine Cleveland, Ohio Associate Program Director Internal Medicine Residency Cleveland Clinic Cleveland, Ohio Staff Cardiologist, Section of Clinical Cardiology Miller Family Heart and Vascular Institute Cleveland Clinic Cleveland, Ohio A dult congenital heart disease How to approach the cardiology patient Cardiogenic shock A pproach to the patient with chronic dyspnea Cardiac risk assessment prior to noncardiac surgery A am Gol berg, Md Fellow, Cardiac Electrophysiology and Pacing Miller Family Heart and Vascular Institute Cleveland Clinic Cleveland, Ohio Bradycardia AV nodal reentrant tachycardia

vii

viii

c o n t Rib u t o Rs

Justin Gro in, Md Fellow, Cardiovascular Medicine Miller Family Heart and Vascular Institute Cleveland Clinic Cleveland, Ohio Cardiomyopathies Hypertension Serge C. Harb, Md Fellow, Cardiovascular Medicine Miller Family Heart and Vascular Institute Cleveland Clinic Cleveland, Ohio A cute pericarditis N ael Hawwa, Md Fellow, Cardiovascular Medicine Miller Family Heart and Vascular Institute Cleveland Clinic Cleveland, Ohio Pulmonary hypertension Michael Hoosien, Md Fellow, Cardiovascular Medicine Miller Family Heart and Vascular Institute Cleveland Clinic Cleveland, Ohio Chronic stable coronary artery disease Preventive cardiology Michael Johnson, Md Fellow, Cardiovascular Medicine Miller Family Heart and Vascular Institute Cleveland Clinic Cleveland, Ohio In ective endocarditis Jason Lappe, Md Fellow, Cardiovascular Medicine Miller Family Heart and Vascular Institute Cleveland Clinic Cleveland, Ohio Wide complex tachycardia Si harth Mahure, Md Research Fellow NYU Hospital for Joint Diseases Department of Orthopaedic Surgery Shoulder & Elbow Division New York, New York Peripheral arterial disease

c o n t Rib u t o Rs

Christopher May, Md Advanced Cardiovascular Imaging Fellow Miller Family Heart and Vascular Institute Cleveland Clinic Cleveland, Ohio A nterior ST EMI N ST EMI Kenneth A. Mayuga, Md , FACC, FACP Clinical Instructor of Medicine Cleveland Clinic Cleveland, Ohio Associate Staff, Section of Cardiac Electrophysiology and Pacing Miller Family Heart and Vascular Institute Cleveland Clinic Cleveland, Ohio Syncope Shruti Patel, Md Resident, Nassau University Medical Center Department of Internal Medicine East Meadow, New York Hypertrophic cardiomyopathy Liane Porepa, Md , FRCPC, FACC Advanced Heart Failure Cardiologist Director, Heart Failure Program Southlake Regional Health Centre Newmarket, Ontario, Canada A dvanced heart ailure and transplantation Grant Ree , Md Fellow, Cardiovascular Medicine Miller Family Heart and Vascular Institute Cleveland Clinic Cleveland, Ohio Severe aortic stenosis Chest pain, undi erentiated Brett Sperry, Md Fellow, Cardiovascular Medicine Miller Family Heart and Vascular Institute Cleveland Clinic Cleveland, Ohio A cute RV ailure complicating MI Chronic heart ailure

ix

x

c o n t Rib u t o Rs

N ewton Wiggins, Md Chief Fellow, Cardiovascular Medicine Miller Family Heart and Vascular Institute Cleveland Clinic Cleveland, Ohio A cute aortic valve regurgitation Chronic aortic valve regurgitation Allison L. Toy Senior Nursing Student Scott & White Nursing School University of Mary Hardin-Baylor Belton, Texas Primary Manuscript Reviewer

a c kn o w Led Gmen t s

The curriculum that evolved into the ideas for this series was inspired by Philbert Yau and Chuck Rosipal, two talented and forthright students, who have since graduated from medical school. It has been a tremendous joy to work with my excellent coauthor, Dr. Michael Faulx, who exemplifies the qualities of the ideal physician—caring, empathetic, and brilliant educator who can make complex topics understandable; he also has the unique ability to bridge the disciplines of internal medicine and cardiology, no easy feat! Michael D. Faulx would like to acknowledge Dr. Eugene Toy for his vision for and commitment to this wonderful book series. He would also like to acknowledge Catherine Johnson and Cindy Yoo for their helpful editorial suggestions and Anupriya Tyagi for her tireless attention to detail. He lastly wish to acknowledge the Cleveland Clinic Department of Medical Art and Photography, particularly Joe Pangrace, Jeff Loerch, and Ken Celebucki for their outstanding medical illustrations. I am greatly indebted to my editor, Catherine Johnson, whose exuberance, experience, and vision helped to shape this series. I appreciate McGraw-Hill’s belief in the concept of teaching through clinical cases. I am also grateful to Catherine Saggese for her excellent production expertise, and Cindy Yoo for her wonderful editing. I cherish the ever-organized and precise Anupriya Tyagi who has nurtured this book from manuscript to print. It has been a privilege and honor to work with my daughter Allison, a senior nursing student, who was the principal manuscript reviewer. I appreciate Linda Bergstrom for her sage advice and passion. At Methodist, I appreciate Drs. Judy Paukert, Marc Boom, and Alan Kaplan for their support. Without my dear colleagues, Drs. Konrad Harms, Priti Schachel, Gizelle Brooks Carter, and Russell Edwards, this book could not have been written. Most of all, I appreciate my ever-loving wife Terri, and our four wonderful children, Andy and his wife Anna, Michael, Allison, and Christina, for their patience and understanding. Eugene C. Toy and Michael D. Faulx

xi

This page intentionally left blank

in t Ro d u c t io n

Mastering the cognitive knowledge within a field such as cardiology is a formidable task. It is even more difficult to draw on that knowledge, procure and filter through the clinical and laboratory data, develop a differential diagnosis, and, finally, to make a rational treatment plan. To gain these skills, the student learns best at the bedside, guided and instructed by experienced teachers, and inspired toward selfdirected, diligent reading. Clearly, there is no replacement for education at the bedside. Unfortunately, clinical situations rarely encompass the breadth of the specialty. Perhaps the best alternative is a carefully crafted patient case designed to stimulate the clinical approach and the decision-making process. In an attempt to achieve that goal, we have constructed a collection of clinical vignettes to teach diagnostic or therapeutic approaches relevant to cardiology. In this age of technology and high-definition imaging, we would like to reinforce the importance of the history and physical examination. We urge that students diligently read through this area in Section I of the book, and practice their skills. We likewise urge our peer colleagues to spend time demonstrating to students and trainees how to properly perform the physical exam maneuvers. We hope that our cases will stimulate excitement for the clinical care of patients. Most importantly, the explanations for the cases emphasize the mechanisms and underlying principles, rather than merely rote questions and answers. This book is organized for versatility: it allows the student “in a rush” to go quickly through the scenarios and check the corresponding answers, and it allows the student who wants thought-provoking explanations to obtain them. The answers are arranged from simple to complex: the bare answers, an analysis of the case, an approach to the pertinent topic, a comprehension test at the end, clinical pearls for emphasis, and a list of literature sources for further reading. The clinical vignettes are purposely placed in random order to simulate the way that real patients present to the practitioner. A listing of cases is included in Section III to aid the student who desires to test his/her knowledge of a certain area, or to review a topic, including basic definitions. Finally, we intentionally did not use a multiple-choice question format in the case scenarios, because clues (or distractions) are not available in the real world.

xiii

This page intentionally left blank

SECTION I: H O w TO Ap p r O ACH TH E CAr d IO l O g y p ATIENT

SECTION I

How to Approach the Cardiology Patient Part 1

A

oach to the patient (Histo

an ph sica )

Part 2

A

oach to the E ect oca io am (ECg )

Part 3

p ovi e s an p oce u es

1

2

CASE FIl ES: CAr d IO l Og y

Part 1. Cardiovascular History and Physical Examination There are four main components of cardiovascular history and physical examination: A. Taking a cardiovascular history B. Performing the cardiovascular examination C. Interpreting heart sounds D. Evaluating cardiac murmurs Despite the proliferation of medical technology over the past several decades, there remains no single imaging study or laboratory assay more valuable to patient care than a proper history and physical examination (H&P). A thoughtful H&P will provide you with the correct diagnosis for most patients presenting with cardiovascular disease complaints. The act of performing the H&P also affords the caregiver an opportunity to forge a therapeutic relationship with the patient. The attention paid to a frightened patient by a thoughtful practitioner during the H&P, however brief, can have both diagnostic and therapeutic benefits. Finally, in this current era of cost-conscious medical care, there are few tools as cost-effective as a good H&P.

CLINICAL PEARL C

The most im o tant too in the assessment o the atient ith ca iovascu a isease is a e - e o me histo an h sica examination.

A. TAKING A CARDIOVASCULAR HISTORY Prior to entering into a discussion of the cardiovascular history, there are a few general rules of history taking that merit a review. The first is to establish a meaningful rapport with the patient. As the provider, you should be the adaptable member of this relationship as you will need to alter your history-taking approach from one patient to the next to account for differences in individual language comprehension, cultural background, and level of education. The use of medical or technical jargon during the history should be avoided. Similarly, common colloquial medical terms should be carefully scrutinized as they often mean different things to different patients. For example, a patient may tell you that she has had five heart attacks in the past 2 years, but a careful review of her records reveals no evidence of myocardial infarction but rather five emergency department visits for chest pain and severe hypertension in the setting of medication noncompliance. Another important skill in history taking is the ability to adjust one’s interview style to best suit the patient and setting. It is generally advisable to begin the interview with open-ended questions (eg, “What brings you to the emergency department today, Mr. Smith?”) to allow patients to guide you through their histories. However, there are certainly patients who do not provide much open-ended information (eg, “My wife made me come.”), in which case direct initial questions may

SECTION I: H O w TO Ap p r O ACH TH E CAr d IO l O g y p ATIENT

3

Table I-1 • A GENERAL APPROACH TO CARDIAC HISTORY TAKING Symptom Feature

Examples

d esc i tion

Subjective cha acte (sha , u , bu nin , etc)? l ocation ( oca , i use, e t, i ht, etc)?

Associate items

r a iation o ain? Accom an in s m toms?

Seve it

Nume ica sca e ( ain 1–10 o 10 sca e)? w ith activit (CCS o NyHA c ass 1–4)?

Timin Onset d u ation F equenc Tem o

w hen i s m tom sta t? Ho on i s m tom ast? Ho o ten oes s m tom occu ? A e s m toms becomin mo e o ess seve e?

In uences Ti e s r e ieve s

Exe tion, eatin , bo osition, etc? r est, me ications, h sica mani u ation, etc?

Im ai ment

l imitations at o k, ith Ad l s, socia i e, etc?

Note: Each atient s m tom shou be assesse o seve a ke eatu es as esc ibe above. CCS; Cana ian Ca iovascu a Societ . NyHA; Ne yo k Hea t Association.

have a higher yield (eg, “Have you been having any chest pain, Mr. Smith?”) . It is helpful to repeat the history back to the patient to ensure that the patient agrees with the history as you understand it. This approach tends to give patients a greater sense of involvement in the evaluation process and also an opportunity to edit the history before you proceed with the physical examination. Finally, it is important to be consistent and thorough in your approach to each element of the cardiovascular history (Table I-1). Every symptom should be assessed for its subjective description and overall level of severity. The timeframe for each symptom, including time of onset, duration, frequency, and pattern of evolution should be ascertained. Whenever possible, one should quantify symptom severity (eg, “On a scale of 1 to 10 how severe was your chest pain?” or “How many yards can you walk before you develop calf pain?”). It is also important to know whether the symptom has any clear triggers or relieving factors. Understanding how symptoms impact the patient’s quality of life is also instructive (eg, “We have discussed a number of symptoms, Mr. Jones; which one worries you the most?”). You will occasionally find that the presenting complaint is not the patient’s major concern. For example, Mr. Jones may have visited the office today because his wife is worried about his newly swollen ankles, but the most concerning issue to Mr. Smith may be his worsening erectile dysfunction and the tension it is causing in his marriage.

CLINICAL PEARL C

Be consistent in ou a oach to each e ement o the histo an acqui e as much objective in o mation as ossib e.

4

CASE FIl ES: CAr d IO l Og y

Chest Pain Chest pain is the most common complaint presented by patients to cardiologists. The differential diagnosis for chest pain is quite broad and includes both cardiac and noncardiac conditions. A full review of the approach to chest pain is beyond the scope of this introductory chapter, but the topic of chest pain is covered as a case file later in this book. In a patient complaining of chest pain, the principal concern is whether the patient is experiencing angina, or chest pain secondary to myocardial ischemia. Chest pain can be classified as typical for angina, atypical for angina, or noncardiac, on the basis of its description, triggers, and response to intervention (Table I-2). Typical angina pectoris is relatively easy to recognize; unfortunately, many patients with myocardial ischemia lack typical symptoms of angina, especially women and patients with diabetes mellitus. Angina is typically described as diffuse and pressurelike and localizes to the retrosternal area. Dullness and burning are common descriptors of anginal chest pain, and the inability to adequately describe one’s chest pain is also suggestive of angina. The act of balling the fist and holding it against the chest while trying to describe chest pain, called Levine’s sign, is actually fairly specific for angina. Pain radiating from the chest to the neck, jaw, or arms is suggestive of angina. Angina typically lasts between 5 and 30 minutes; chest pain lasting for mere seconds or for hours or days without resolution is practically never angina. Angina is usually triggered by physical activity or emotional distress and resolves with rest, relaxation, or sublingual nitroglycerin. Typical chest pain occurring 20−30 minutes after a meal is also consistent with postprandial angina, although this often is mislabeled as esophageal reflux or dyspepsia. Chest pain provoked or worsened by manual palpation is unlikely to be angina. Angina severity is commonly graded using the Canadian Cardiovascular Society (CCS) classification system (Table I-3). Class 3 or 4 angina is considered to be severe. Recent-onset (within 2 weeks) severe angina is referred to as unstable angina and generally requires hospitalization and immediate medical attention.

Table I-2 • CHARACTERIZATION OF CHEST PAIN AS TYPICAL FOR ANGINA, ATYPICAL FOR ANGINA, OR NONANGINAL ACCORDING TO KEY CLINICAL FEATURES

*

Features

Aggravating Factors

Alleviating Factors

d i use subste na iscom o t d u , essu e ike qua it r a iation to neck o ja 5–30-minute u ation

ph sica exe tion Emotiona ist ess post an ia state*

r est o e axation Nit o ce in

Typical chest ain

A three o the above

Atypical chest ain

An two o the above

Non-anginal chest ain

One or none o the above

Chest ain that consistent be ins a

oximate 30 minutes a te mea s is su

estive o

ost an ia an ina.

SECTION I: H O w TO Ap p r O ACH TH E CAr d IO l O g y p ATIENT

5

Table I-3 • CLASSIFICATION OF ANGINA AND DYSPNEA SEVERITY Class

CCS Angina

NYHA Dyspnea

1

O ina h sica activit oes not cause an ina; an ina occu s on ith st enuous, a i , o o on e h sica activit at o k o u in ec eation

No imitation o h sica activit ; h sica activit oes not cause s nea, ati ue, o a itations

2

S i ht imitation o h sica activit ; an ina occu s ith a kin > 2 b ocks o c imbin > 1 FoS ith usua ace an ci c*mstances or occu s ith ess istance i un e st ess ( a i ace, stee inc ine, a te mea s, emotiona u set, ea mo nin )

S i ht imitation o h sica activit ; com o tab e at est but ext ao ina h sica activit causes s nea, ati ue, o a itations

3

Ma ke imitation o h sica activit ; an ina occu s ith a kin 1–2 b ocks o < 1 FoS at usua ace an ci c*mstances

3A. l imite activit ; com o tab e at est but o ina activit causes s nea, ati ue, o a itations 3B. Si ni icant imitation; com o tab e at est but ess than o ina activit causes s nea, ati ue, o a itations

4

Inabi it to e o m an an ina; an ina ma be

Inabi it to e o m an h sica activit ithout s nea, ati ue, o a itations; s m toms ma occu at est

h sica activit esent at est

ithout

Abbreviations: CCS, Cana ian Ca iovascu a Societ ; NyHA, Ne yo k Hea t Association; FoS, f i ht o stai s.

Angina present for more than 2 weeks that is clearly triggered by predictable physical activity is termed stable angina and is often managed in the outpatient setting. Another common type of chest pain encountered in patients with heart disease is inflammatory chest pain, which can accompany conditions such as pericarditis, myocarditis, or acute pulmonary thromboembolism. Unlike angina, inflammatory chest pain is typically sharp and focal. It is commonly pleuritic, becoming more severe during inspiration or laying supine and improving with expiration or leaning forward. Pleuritic chest pain that can be reproduced by manual palpation of the chest wall is most likely related to costochondritis. Pulmonary infections such as pneumonia can also produce pleuritic chest pain, although there are often other symptoms that support the diagnosis such as fevers or productive cough. Pulmonary thromboembolic disease can also cause pleuritic plain and should be considered early in the differential diagnosis of any patient with pain and risk factors for venous thromboembolic disease. The combination of severe, tearing, or ripping chest and/ or back pain and hypertension raises the concern for an acute aortic syndrome such as aortic dissection. The evaluation and differential diagnosis of the patient presenting with chest pain is separately discussed later in this book.

CLINICAL PEARL C

women an iabetic atients a e mo e ike to ex e ience at ica s m toms u in e iso es o m oca ia ischemia.

6

CASE FIl ES: CAr d IO l Og y

Shortness of Breath Shortness of breath (dyspnea) is another common symptom that can be the presenting complaint for many cardiac diagnoses, including heart failure, valvular heart disease, atrial fibrillation, and even myocardial ischemia. As with angina, we quantify shortness of breath using a four-point scale developed by the New York Heart Association (NYHA; Table I-3). Shortness of breath accompanied by difficulty breathing while supine (orthopnea) or paroxysmal nocturnal dyspnea (PND) is strongly suggestive of increased left atrial pressure, a common feature of heart failure or left-sided valvular heart disease. Associated symptoms such as exercise tolerance and fatigue are also suggestive of shortness of breath due to cardiovascular causes. Heart failure is also frequently accompanied by central volume overload; this may be heralded by symptoms such as leg swelling (edema), weight gain, or tighter-fitting clothes. Excessive somnolence, profound weakness, and a subjective decrease in urine output that accompanies shortness of breath may be features of advanced heart failure with low cardiac output. It should be noted that symptoms attributable to heart failure can be present in the setting of normal left ventricular systolic function. The differential diagnosis of dyspnea is enormous and includes a wide range of cardiac and noncardiac diagnoses. The evaluation and differential diagnosis of the patient presenting with shortness of breath is separately discussed later in this book.

Dizziness and Syncope Loss of consciousness (syncope) and dizziness are also common cardiovascular complaints with a broad differential diagnosis. These complaints are common in patients presenting with rhythm disorders (both tachy- and bradyarrhythmias) and structural heart disease, particularly conditions that limit ventricular outflow such as aortic or mitral valve stenosis, or hypertrophic cardiomyopathy with dynamic obstruction of the left ventricular outflow tract. Syncope that occurs during or just after physical exertion suggests the presence of reduced outflow. Syncope that is accompanied by palpitations suggests the presence of a tachyarrhythmia. Syncope accompanied by lightheadedness and nausea and followed by diaphoresis suggests a neurocardiogenic cause such as vasovagal syncope. Ominous syncope features include abrupt onset without warning, prolonged unconsciousness, and injury as a result of syncope; these features suggest a high-risk cause for syncope such as a malignant arrhythmia.

CLINICAL PEARL C

A e is an im o tant o nostic acto in atients esentin ith s nco e. patients a e 60 ea s o ten have s nco e ue to otentia an e ous ca iac causes.

Adjunctive History A complete cardiac H&P should include a thorough noncardiac review of systems. This is important because cardiovascular diseases can have extracardiac

SECTION I: H O w TO Ap p r O ACH TH E CAr d IO l O g y p ATIENT

7

manifestations and noncardiac illnesses can have cardiovascular implications. For example, erectile dysfunction in an otherwise asymptomatic patient might suggest the presence of occult vascular disease. Alternatively, a history of rash with swollen and painful joints can indicate the presence of rheumatoid arthritis, a diagnosis associated with an increased risk for many cardiovascular problems including coronary artery disease, pericarditis, and pulmonary arterial hypertension. Some noncardiac conditions are highly prevalent among patients with heart disease, particularly obstructive sleep apnea (OSA). Untreated severe OSA can have a major impact on a patient’s general health and quality of life, and for this reason it is important to inquire about the signs and symptoms of OSA during the H&P, including daytime sleepiness, snoring, witnessed apneas, and cognitive impairment. Another important and often overlooked feature of the H&P involves a thorough review of the patient’s medication list for potential drug-drug or drug-food interactions and adverse drug reactions (ADRs). In some cases the patients’ presenting complaints may be related to the adverse effects of their prescribed medications. Although some cardiovascular medications have well-described side effects or interaction potential (Table I-4), one must maintain a high index of suspicion for ADRs related to any agent, particularly when a patient has symptoms that are not readily explainable despite extensive evaluation or bear some temporal relationship to a

Table I-4 • COMMONLY PRESCRIBED CARDIAC MEDICATIONS AND THEIR POTENTIAL ADVERSE EFFECTS Drug Class

Examples

Common Adverse Effects

ACE inhibito s

l isino i , ami i

Non o uctive cou h

A oste one ece to b ocke s

S i ono actone

g necomastia

Anticoa u ants

wa a in, abi at an, iva oxaban, a ixaban

B ee in , co ( a a in)

Anti ate et a ents

As i in, c o i o e , tica e o

B ee in ,

Beta b ocke s

Ca ve i o , meto o o , ateno o , o ano o

Fati ue, e ecti e s unction, sho tness o b eath *

Ca cium channe b ocke s

Am o i ine, i tiazem, ve a ami

l e e ema, consti ation

HMg -CoA e uctase inhibito s

r osuvastatin, ato vastatin, avastatin, simvastatin

Musc e ain an

H

H

Ab omina ain, u us ike s m toms ( a e)

a azine

asu e ,

a azine

Nicotinic aci

Niacin, Nias an

Facia

Nit o

Sub in ua , isoso bi e init ate an mononit ate

Hea ache

ce in

sensation s nea (tica e o )

eakness

ushin

Abbreviations: ACE, an iotensin-conve tin enz me; HMg -CoA, 3-h ox -3-meth - uta -coenz me A. * Sho tness o b eath is mo e ike to occu in atients takin non-β1-se ective a ents (ca ve i o , o ano o ) ho have a histo o eactive ai a isease o b onchos asm.

8

CASE FIl ES: CAr d IO l Og y

recent hospital discharge, office visit, or other encounter where medications may have been introduced or changed. In keeping with this theme, it is also important to inquire about the use of over-the-counter and alternative medical agents.

B. PERFORMING THE CARDIOVASCULAR EXAMINATION The physical examination of the cardiac patient, particularly cardiac auscultation, has intimidated medical students and residents alike since the invention of the stethoscope by Dr. René Laennec in 1816. The process of examination and cardiac auscultation can be challenging, but the experience need not be stressful if one has a solid understanding of cardiovascular physiology, a good stethoscope, a quiet examination area, patience, and a bit of curiosity. Like most things in life, good examination skills develop with practice over time. You should also appreciate the limitations of the physical examination; even the most experienced cardiologist will miss a soft diastolic murmur in a tachycardic, morbidly obese patient on mechanical ventilation in a noisy intensive care unit. Whenever possible, it is useful to compare your examination findings to the findings of dynamic cardiac studies such as echocardiograms and magnetic resonance images (MRIs). Did you miss the murmur of the severe, posteriorly directed mitral regurgitation that was seen by echocardiography? Go back and listen to the patient again, armed with the knowledge of the imaging study. You will probably hear the murmur and be able to recognize it when you admit your next patient with severe mitral regurgitation.

Nonauscultory Exam ination The cardiovascular examination should begin with a general inspection of the patient and the patient’s chest. You should note any obvious abnormalities, as these may provide you with diagnostic clues. Does the patient have any scars indicative of prior cardiac surgery? Is there an implanted cardiac device? Are there any chest wall deformities that might indicate a congenital disorder such as Marfan syndrome (pectus excavatum or carinatum) or Turner syndrome (shield-shaped chest)? Blood pressure measurements should be obtained from both arms and compared; differences in systolic and diastolic blood pressures of greater than 10 or 5 mmHg, respectively, are considered abnormal and may warrant further investigation for disorders such as aortic dissection or subclavian stenosis. If one suspects the diagnosis of aortic coarctation, then bilateral lower extremity blood pressure measurements should also be obtained. Jugular Veins Examination of the right internal jugular vein allows for estimation of a patient’s central volume status and provides clues for the diagnosis of rightsided heart disease and pericardial disease. The right internal jugular vein is best examined with the patient seated at a 45° angle. It is noteworthy that, in patients with marked central volume overload, the top of the venous column of blood (meniscus) may not be visible until the patient is seated upright at 90°. Similarly, patients with low filling pressures may not exhibit a visible meniscus until they are nearly flat (180°). To estimate the right atrial pressure, the examiner measures the distance between the sternal angle (angle of Louis) and the meniscus of the jugular pressure wave

SECTION I: H O w TO Ap p r O ACH TH E CAr d IO l O g y p ATIENT

9

Me nis cus

45° Angle of Louis 5 cm

Right a trium

Figure I-1 Estimation o the ju u a venous

essu e.

(Figure I-1). The jugular venous pulsation may be distinguished from the carotid pulse by applying gentle tension to the overlying skin with your finger; this should obliterate the venous pulse, whereas the arterial pulse should remain visible. The center of the right atrium is approximately 5 cm below the angle of Louis, so the measured distance (in centimeters) between the angle of Louis and the meniscus plus 5 cm approximates right atrial pressure. Normal jugular venous pressure is 8 cm or less. Recall that 1 cm H 2O is equivalent to 0.735 mmHg. Increased jugular venous pressure is indicative of high right atrial pressure, and this finding can suggest several cardiac diagnoses, including left or right ventricular failure, tricuspid valve disease, and pericardial disease. During inspiration the normal right heart will dilate in response to negative intrathoracic pressure and accommodate more venous flow, resulting in a decrease of the jugular venous pulse (JVP). A paradoxical increase in the JVP during inspiration is called Kussmaul’s sign, and this finding is indicative of abnormal right ventricular filling. Kussmaul’s sign can be seen with pericardial disease (constriction and tamponade), restrictive cardiomyopathies, and advanced right ventricular systolic failure. The response of the jugular venous pulse to prolonged abdominal palpation, termed hepatojugular reflux or the abdominojugular test, can also be a useful examination tool. With this maneuver, the JVP is observed during and after at least 10 seconds of sustained firm palpation over the right upper quadrant or midepigastrium. This maneuver will increase venous return to the heart. A normal heart can quickly accommodate the extra preload, and the JVP will increase momentarily before returning to normal. Conversely, a patient with a sustained increase in JVP of >4 cm or an abrupt decrease in JVP of >4 cm on release of abdominal pressure is considered to have an abnormal response. Abnormal abdominojugular testing correlates with increased pulmonary capillary wedge pressure and is typically seen in patients with left ventricular failure.

10

CASE FIl ES: CAr d IO l Og y

A

V

C

Y

X

S1

S2

Figure I-2 The no ma cent a venous ave o m.

The normal jugular vein has two visible waves, the A and V waves (Figure I-2). A third small c wave is measurable invasively but is practically never seen on physical examination. The c wave is essentially a ripple in the jugular waveform caused by the upward motion of the tricuspid valve during early right ventricular systole. The A and V waves are followed by negative pressure deflections, called the x and y descents, respectively. The A wave occurs just before the first heart sound (S1) and is caused by atrial contraction. If the patient has a fourth heart sound, the A wave will occur simultaneously with the S4. The x descent occurs during atrial relaxation after closure of the tricuspid valve. The V wave is the result of right atrial filling during ventricular systole and is normally smaller and broader than the A wave. The V wave is followed by the y descent, which is caused by emptying of the right atrium after opening of the tricuspid valve in early diastole. Abnormalities of the right internal jugular venous waveforms are directly related to right-heart pathology. Consistently large A waves are present in disorders that result in right atrial pressure overload such as tricuspid stenosis or pulmonary hypertension. Intermittently large A waves, called “cannon A waves,” can be seen during arrhythmias when the right atrium contracts while the tricuspid valve is closed. Complete absence of the A wave occurs when there is no atrial contraction such as in atrial fibrillation. Large V waves indicate volume overload of the right atrium during ventricular systole; this most commonly occurs with tricuspid regurgitation but can also be seen in patients with atrial septal defects. The x and y descents are often affected by pericardial diseases. In pericardial tamponade one typically sees a prominent x descent because the contracting right ventricle occupies less space within the fluid-engorged pericardial sac during systole, reducing the pericardial pressure around the adjacent right atrium and allowing it to expand in size. During diastole, pericardial pressure is highest as the ventricles expand and passive flow between the right atrium and ventricle practically stops, producing an absent or significantly blunted y descent. In pericardial constriction, hemodynamic changes occur more as the result of tethering of the cardiac chambers to the noncompliant pericardium. The x descent is steep because the right atrium is pulled toward the contracting right ventricle during systole, expanding its size. The y descent is also prominent because the resting early diastolic pressure in the right ventricle is typically much lower than the resting right atrial pressure, resulting in rapid early diastolic filling. The y descent abruptly ends by middiastole in constrictive pericarditis as the pressure hits a plateau (“square-root sign”) because the thickened pericardium limits full expansion of the right ventricle.

SECTION I: H O w TO Ap p r O ACH TH E CAr d IO l O g y p ATIENT

11

CLINICAL PEARL C

Examination o the inte na ju u a vein usin ob ique i umination b a en i ht can o ten make the ju u a venous u se easie to see.

Arterial Pulse The normal arterial pulse has an initial brisk upstroke followed by a systolic peak that corresponds to early ventricular ejection (Figure I-3). This is followed by a decline in systolic pressure as the elastic aortic walls expand to accommodate the systolic pressure wave. Aortic valve closure produces the incisura, a small wave that appears at the end of the systolic pressure tracing in central aortic tracings but appears later in peripheral arterial tracings as the wave is transmitted down the arterial tree. Following systolic ejection, arterial runoff results in a decrease in atrial pressure during early diastole while pressure recovery due to the elastic recoil of the artery produces a plateau phase in the pressure tracing in late diastole. Arterial pulses should be assessed for their contour, intensity, and timing relative to apical systolic ejection. Pulses with two palpable systolic peaks (bisferiens) can be due to dynamic left ventricular outflow obstruction or the combination of aortic valve stenosis and severe regurgitation; the latter is typically associated with a widened pulse pressure. In contrast, low-volume pulses with palpable peaks in systole and early diastole are common in low-cardiac-output states and are referred to as dicrotic pulses. A weak pulse (parvus) with a delayed peak (tardus) is the hallmark of severe aortic stenosis. A palpable low-frequency vibration over a peripheral artery (thrill) is indicative of peripheral arterial stenosis. Peripheral pulses can also vary with the respiratory cycle. A small (3 cm) o

l e t vent icu a en a ement

l ate a

i use

is ace

l e t vent icu a en a ement

Faint o non a ab e

pe ica ia e usion o const iction; mo bi obesit ; COpd

d namic an b ie

Acute mit a e u itation

d namic an sustaine

l e t vent icu a h e t o h

Faint an sustaine

Seve e ao tic stenosis; e t vent icu a s sto ic

d oub e

H e t o hic ca iom o ath ( ith obst uction

p es sto ic ( a ab e A ave)*

Ao tic stenosis; h e t o hic ca iom o ath ; acute mit a e u itation

s unction

namic l VOT

Abbreviations: COpd , ch onic obst uctive u mona isease; l VOT, e t vent icu a outf o t act. * p es sto ic a ica im u ses a e best a eciate b obse vin o movement o a stethosco e that is i ht a to the chest a u in auscu tation.

ie

SECTION I: H O w TO Ap p r O ACH TH E CAr d IO l O g y p ATIENT

13

CLINICAL PEARL C

Consi e the ia nosis o e ica ia tam ona e o atients ith h otension an tach ca ia ho o not have a a ab e a ica im u se.

Cardiac Auscultation To understand what you are hearing during cardiac auscultation, it is imperative that you understand what the valves and chambers are doing throughout the cardiac cycle and that you can readily identify systole and diastole (Figure I-4). Systole occurs between the first and second heart sounds and generally has a fixed duration. Systole includes an early period of isovolumic contraction of both ventricles

S e miluna r clos e Ce ntra l a rte ria l pre s s ure S e miluna r ope n Ve ntricula r pre s s ure AV ope n AV clos e

S4 La te dia s tole

Atria l pre s s ure

S1

S2 Sys tole

S3 Ea rly dia s tole

Figure I-4 The no ma ca iac c c e. Cent a a te ia , venous, an vent icu a essu es a e sho n in conjunction ith the ECg an hono a hic in in s [AV; at iovent icu a va ves (t icus i an mit a )]. “Semi una ” e e s to the u monic an ao tic va ves. (Reprinted with permission, Cleveland Clinic Center for Medical Art & Photography © 2013. All rights reserved.)

14

CASE FIl ES: CAr d IO l Og y

followed by the opening of the semilunar valves (aortic and pulmonic valves) and ventricular ejection. The atria are filling during systole. Diastole occurs between the second and first heart sounds, and its duration varies with heart rate; at normal heart rates [under 100 beats per minute ( 100 bpm.

Heart Rate The heart rate on the ECG can be calculated by noting the time interval between two R adjacent waves (provided that the rhythm is regular). Recall that if the paper speed is standard at 25 mm/s, then the time interval of one large square is 200 ms and the time interval of one small box is 40 ms (Figure I-8). The heart rate can then be determined by the following equation: HR = [1 beat / R-R interval (seconds)] × (60 sec / min). If there are three large boxes and two small boxes between two consecutive R waves, then the R-R interval is 680 ms or 0.68 second. The heart rate would then be 1/0.68 × 60/1 = 60/0.68 = 88 bpm. A common “quick and dirty” method for heart rate estimation is to simply count the number of large boxes between consecutive R waves. At standard paper speed the heart rate for R-R intervals of 1, 2, 3, 4, and 5 large boxes is 300, 150, 100, 75, and 60 bpm, respectively. If the rhythm is

28

CASE FIl ES: CAr d IO l Og y

irregular or extremely slow, one can refer to the vertical hash marks situated at the bottom of most ECG tracings. These marks are spaced 3 seconds apart, so counting the number of complexes within a 6-second span and multiplying by 10 can give you an estimate of the heart rate. If the hash marks are not there, you can draw your own; 15 large boxes = 3 seconds.

Regularity Each ECG should be assessed for rhythm regularity, and calipers are a useful tool for this purpose. One should assess the regularity of the atrial waveforms (P waves) and ventricular waveforms (QRS complexes) using the rhythm strips at the bottom of the ECG tracing. Are the P-P and R-R intervals regular? Are these intervals the same? With normal conduction the P-P and R-R intervals should match, but in disorders such as atrioventricular (AV) block or ventricular tachycardia, the intervals will differ. An atrial rate that is regular and greater than the ventricular rate is referred to as complete heart block. In complete heart block the atrial impulses are not activating the ventricles, forcing the QRS complexes to originate from a lower “secondary” pacemaker such as the AVN-His bundle junction. A ventricular rate that is regular and greater than the atrial rate is referred to as A-V dissociation. A-V dissociation is a hallmark finding in ventricular tachycardia where the ventricular complexes originate from an abnormal area of the ventricle that depolarizes independently.

QRS Axis The QRS axis is a vector that represents the net direction of ventricular depolarization. Deviation of the QRS axis occurs in a number of cardiovascular disorders, and detection of QRS axis deviation is useful for generating a differential diagnosis. Axis deviation is also a criterion for some ECG diagnoses such as anterior and posterior fascicular blocks. The limb leads form an axial array in the frontal plane that serves as the template for QRS axis derivation. Inspection of the limb leads will allow the reader to determine the QRS axis. In reality the QRS vector is three-dimensional (3D), but the bedside measurement of a 3D QRS vector using ECG data, although possible via a process called spatial vectorcardiography, is technically cumbersome and impractical in real-world cardiology practice. The normal QRS vector lies between - 30° and 90°in the frontal plane (Figure I-12). Deviation of the axis from - 30° to - 90° is termed left-axis deviation, and deviation of the axis between 90° and 180° is termed right-axis deviation. Left- or right-axis deviation can occur as the result of structural heart disease such as left or right ventricular hypertrophy or dilatation. Extreme axis deviation (180° to - 90°) is typically seen with conduction from the ventricles such as during ventricular tachycardia. There are numerous techniques available for determining the QRS axis. One commonly used method is to examine the orientation of the QRS complexes in leads I and aVF (see Figure I-12). A positive QRS deflection in lead I tells you that the QRS vector lies between 90° and - 90°. A positive QRS deflection in lead aVF tells you that the QRS vector lies between 0° and 180°. Thus, if you have an

SECTION I: H O w TO Ap p r O ACH TH E CAr d IO l O g y p ATIENT

29

upright QRS deflection in I and aVF, then the QRS vector must lie in the shared “right lower quadrant” between 0° and 90°, and therefore the axis must be normal. If you have a positive QRS in lead I but a negative QRS in lead aVF, the QRS axis might be normal (0° to - 30°) or deviated to the left (- 30° to - 90°). In this case one should look at lead II. A positive deflection in lead II tells you that the QRS must lie between - 30°and 150°. Thus, if you have an upright QRS in leads I and II, then the QRS axis must project to the shared area between 0° and - 30°. If the QRS is negative in II, then left axis deviation is present.

Intervals Assessment of the PR and QTc intervals should be performed on each ECG tracing using calipers. The best lead to use for this assessment is whichever lead gives you the best landmarks for caliper measurement. The PR interval can be shortened (< 140 ms) in conditions such as ventricular preexcitation or lengthened (> 200 ms) by AVN disease. The QTc can be lengthened by myocardial ischemia, hypokalemia, congenital ion channel disorders, and numerous medications (Table I-8). QTc prolongation is dangerous because it is associated with a form of ventricular tachycardia called torsades de pointes (TdP, commonly referred to simply as torsade). When the QTc is prolonged, there is a longer relative refractory period during which a premature beat may fall and cause early depolarization of myocardium that is not yet ready to be depolarized. These early “afterdepolarizations” (EADs) can reach a threshold potential and cause TdP (Figure I-13). Torsades de pointes has a characteristic undulating appearance from which its name (translated from French as “twisting of the points”) is derived.

Segm ents Part of the routine assessment of the ECG should include assessment of the segments between the individual waveforms on the ECG (Figure I-14). The segment between the T wave and the P wave of the next cardiac cycle is called the T-P segment. The T-P segment is an electrically silent period and serves as the baseline reference point for comparison with the other baseline segments for deviation. The PR segment is included within the PR interval. During the PR segment the depolarization wavefront is moving through the atrioventricular node, His bundle, and bundle branches. These structures have very little mass, and so their depolarization does not produce a deflection within the PR segment. Additionally the atria have already depolarized and have not yet repolarized (the wave of repolarization is normally buried within the QRS). For these reasons the normal PR segment is isoelectric. However, the PR segment can be elevated or depressed in the setting of atrial ischemia or injury, respectively. Atrial ischemia or infarction can sometimes occur in the setting of myocardial infarction. Pericarditis is also associated with PR segment depression. The ST segment is the focus of extreme attention in patients presenting with chest pain or who are suspected of having an acute coronary syndrome. During the normal ST segment the ventricles have already depolarized but have not yet started to repolarize, and the atria have repolarized and are waiting to be depolarized again. Thus, like the PR segment, the normal ST segment is isoelectric. However, the ST segment can be elevated or depressed in the setting of ventricular injury or ischemia,

30

CASE FIl ES: CAr d IO l Og y

Table I-8 • PARTIAL LIST OF QT-PROLONGING DRUGS Drug Family

Examples

Antia h thmics

Amio a one d o eti i e Ibuti i e p ocainami e Quini ine Sota o

Antibiotics

Ci o oxacin C a ith om cin E th om cin g ati oxacin It aconazo e Ketoconazo e l evo oxacin Moxi oxacin

Anti e essants

Amit i t ine d esi amine d oxe in F uoxetine Imi amine Se t a ine Ven a axine

Antihistamines

Astemizo e Te ena ine

Anti s chotics

d o ei o Ha o e i o Quetia ine Thio i azine Zi asi one

Misce aneous

A senic Cisa i e d i oxin d o aset on Metha one r ano azine Sumat i tan Te ena ine Zo mit i tan

respectively. ST segment deviation will be discussed more completely in the next section.

Waves and Com plexes Routine inspection of the ECG should also include assessment of each waveform for any diagnostic abnormalities. The normal P wave should be monophasic in every lead except for V1, and the P wave should be upright in the inferior limb leads (II, III, and aVF).

SECTION I: H O w TO Ap p r O ACH TH E CAr d IO l O g y p ATIENT

I

a VR

V1

V4

II

a VL

V2

V5

III

a VF

V3

V6

31

V1

II V5 “R on T” phe nome non

Long QT inte rva l

Figure I-13 To sa es e ointes. Note the onset o o mo hic vent icu a tach ca ia o o in the a ea ance o a ematu e vent icu a com ex u in the e ative e acto e io (“r on T”). St ikin o on ation o the QT inte va is esent.

The normal QRS complex can vary in appearance from lead to lead. A small Q wave, which represents the normal left-to-right depolarization of the interventricular septum, is not seen in every lead and is most prominent in the lateral leads (I, aVL, V5, and V6). The R and S waves represent depolarization of the left and right ventricles together, but since the left ventricle is so massive, the appearance of the entire QRS complex is dominated by the left ventricle in a normal heart. The relative size and orientation of the R and S components of the QRS complex R

T P

P ST

PR Q

S Figure 1-14 The ECg se ments e ine .

TP

32

CASE FIl ES: CAr d IO l Og y

De po larizatio n – – – –– – – – –

+ + + + + +

Endoca rdium

Re po larizatio n – – – –– – – – –

+ + + + + +

+

+

+

+

+

+ + + + + +

– – – –– – – –

+

+

+ Epica rdium

– – – –– – – –

Endoca rdium

+ – – – –– – – –

+

+ +

+ + + + + +

+ + + + + +

Le a d II

Le a d II

Epica rdium

Figure I-15 Ca toon i ust ation o e o a ization an e o a ization at the ce u a eve . The ositive o e o ea II a a s “sees” a ositive ext ace u a otentia as the e ica ia a e is the ast to e o a ize an the i st to e o a ize. This o uces u i ht Qr S com exes an T aves in ea II.

vary depending on which lead is being assessed. In general, as one progresses from right to left across the precordial leads, the R waves grow taller and the S waves grow shorter. Abnormalities in the amplitude and width of the QRS complex may indicate pathology such as ventricular hypertrophy or myocardial injury. The QRS complex is discussed in greater detail in the next section. The normal T wave is monophasic and oriented in the same direction as the dominant vector of the QRS complex. This concordance between the QRS and the T wave occurs because the depolarization and repolarization waves of the left ventricle move in opposite directions (Figure I-15). The depolarization wave moves from the endocardium to the epicardium, converting the negative intracellular resting potential of the myocytes to the positive state along the way. Recall that ECG leads record extracellular potentials; the extracellular potential “seen” by a positive ECG electrode from an approaching depolarization wavefront is positive until the very end of depolarization, and so the QRS is recorded as an upright deflection. During repolarization the intracellular potential of the myocytes returns to the negative state with a positive extracellular charge. Since this process begins with the epicardial myocytes, the extracellular potential detected by the positive surface ECG pole is positive from the very beginning of repolarization and the recorded T wave appears upright as well.

C. THE ECG IN CARDIOVASCULAR DISEASE The following section represents a cursory review of the ECG abnormalities associated with cardiovascular disease states; a complete review of this subject is beyond the scope of a single chapter. As previously mentioned proficient ECG interpretation

SECTION I: H O w TO Ap p r O ACH TH E CAr d IO l O g y p ATIENT

33

comes from practice and correlation of ECG findings with clinical and cardiovascular imaging findings. Remember that a prior ECG tracing for comparison can be extremely helpful and should be sought whenever possible.

Ischemic Heart Disease The management of ST segment elevation myocardial infarction (STEMI) by definition depends on the patient’s presenting ECG. In a patient presenting with chest discomfort, regional ST segment elevation is usually caused by acute thrombotic occlusion of an epicardial coronary artery and is an indication for emergency coronary revascularization, either catheter-based or with fibrinolytic therapy. The location of the ST segment elevation often indicates the culprit epicardial vessel (Table I-9). One exception to this rule is myocardial infarction occurring in the setting of circumflex artery occlusion. The circumflex usually supplies the posterolateral aspect of the left ventricle, and the leads on a standard 12-lead ECG do not represent this area well. The diagnosis of a circumflex artery occlusion requires a high index of suspicion and can be aided by the use of unconventional ECG leads across the left scapular region (leads V7−V9). The ST segment elevation associated with myocardial infarction is typically horizontal or upwardly concave (“tombstone”); downwardly concave ST elevation can occur in the setting of myocardial infarction, but this is less specific (Figure I-16). Diffuse ST segment elevation that does not correspond to coronary anatomy may be due to diffuse processes such as acute pericarditis or myopericarditis. Diffuse precordial ST segment elevation within 80 ms of the J point in young subjects with no symptoms may represent normal variant early repolarization. In the clinical management of acute coronary syndromes, the discovery of a new left bundle branch block pattern in the setting of convincing chest pain and/or elevated cardiac biomarkers is treated as an anterior STEMI equivalent. ST segment depression can be a manifestation of subendocardial ischemia. Unlike ST segment elevation, ST segment depression is less specific for ischemia and the location of the ST depression does not correlate well with the location of the culprit coronary vessel. Horizontal and downsloping ST segment depression

Table I-9 • CORONARY ANATOMY AND ST ELEVATION ON THE SURFACE ECG Coronary Vessel

Representative ECG leads

l e t main (l M)

aVr an V1

l e t ante io

V1−V4 (se ta e o ato s) V1−V6 (enti e l Ad ) I an aVl ( ia ona b anches)

escen in (l Ad )

Ci cum ex (CX)

r i ht co ona

a te

I an aVl ( oxima / hi h ate a b anches) V5−V6 (obtuse ma ina b anches) “r eve se” V1−V2 ( oste o ate a b anches) (r CA)

II, III, an aVF ( oste io escen in a te ) “r eve se” V1−V2 ( oste o ate a vent icu a b anch) “r eve se” V4 ( i ht vent icu a ma ina b anches)

I

a VR

V1

V4

II

a VL

V2

V5

III

a VF

V3

V6

V1

II

V5 Figure I-16 Acute in e io a inju . Note the ho izonta ST se ment e evation in ea s II, III, an aVF (above b ack ines) ith eci oca ST e ession in ea s I an aVl (be o a k b ack ines ).

SECTION I: H O w TO Ap p r O ACH TH E CAr d IO l O g y p ATIENT

35

is more consistent with ischemia than upsloping ST segment depression. The differential diagnosis of ST segment depression is long and includes left ventricular hypertrophy and digitalis effect. T wave abnormalities can also occur in the setting of myocardial ischemia or infarction. T wave changes associated with myocardial ischemia are typically symmetric and often occur in the territory of the culprit coronary vessel. Deep, symmetric T wave inversions across the anterior precordial leads in a patient with chest pain are highly suggestive of proximal left anterior descending artery disease; this association is commonly referred to as Wellens’ syndrome, after the physician who first described it (Figure I-17). As in ST segment depression, the causes of T wave abnormalities are numerous and asymmetric T wave abnormalities in otherwise asymptomatic patients are a common and nonspecific finding on many ECGs. Prior myocardial infarction can be demonstrated by the presence of a pathologic Q wave on the ECG. Small Q waves are not uncommon in some ECG leads (particularly lateral leads) and represent normal depolarization of the interventricular septum. Any Q wave in leads V1−V3 should be considered abnormal. Q waves that are >30 ms in duration and >1 mm (0.1 mV) in depth should be considered abnormal and likely the consequence of an old myocardial infarction.

Cham ber Enlargem ent The relative size of the cardiac chambers can be ascertained from the surface ECG, and these findings provide valuable clues for making both cardiac and noncardiac diagnoses. As mentioned previously, the voltage amplitude of any waveform or complex on a surface ECG is proportional to mass. As such, enlargement or hypertrophy of the atria and ventricles can be detected on the basis of the voltage, duration, and appearance of P waves and QRS complexes, respectively. Atrial enlargement can cause changes in the P wave. In most leads the P wave appears as a monophasic wave, but the exception to this rule is lead V1, which lies to the right of the upper sternum and is the lead nearest to the atria. In this lead the P wave is usually biphasic with the proximal half representing (roughly) right atrial depolarization and the terminal half representing left atrial depolarization. The first half of the P wave is positive in V1 because this lead is located directly above the right atrium and the wavefront (a 3D entity) moves toward it. The terminal portion of the P wave in V1 represents left atrial depolarization and produces a negative deflection. Since normal atria are similar in size, these deflections are practically mirror images of each other, but in the presence of right or left atrial enlargement one may see a disproportionally positive or negative P wave in V1, respectively (Table I-10). The inferior leads (particularly lead II) are also useful for assessment of the atria. Left atrial enlargement tends to prolong the duration of the P wave in lead II, and there is often a notched appearance to the P wave that resembles the letter M, called p mitrale, for the historic association between left atrial enlargement and severe mitral stenosis. Right atrial enlargement tends to affect the amplitude of the P wave in lead II; tall P waves with a sharp or peaked appearance are often called p pulmonale for the historic association between right atrial enlargement and severe pulmonic stenosis or pulmonary hypertension.

Te s t Type : ECG COMP WOINTER

I

a VR

V1

V4

II

a VL

V2

V5

III

a VF

V3

V6

V1

II

V5 Figure I-17 we ens’ s n ome. Note the ee s mmet ic T ave inve sions ence o oxima e t ante io escen in co ona a te isease.

esent om V1 to V6. In the settin o chest ain, this in in st on

su

ests the

es-

SECTION I: H O w TO Ap p r O ACH TH E CAr d IO l O g y p ATIENT

37

Table I-10 • ECG CRITERIA FOR ATRIAL ENLARGEMENT Condition

Lead V1

Lead II

No ma

Bi hasic positive e ection < 1.5 mm (0.15 mV) Ne ative e ection < 1.0 mm (0.10 mV)

Mono hasic an u i ht d u ation 80−110 ms Am itu e < 2.5 mm (0.25 mV)

l e t at ia en a ement

Bi hasic Ne ative (te mina ) e ection > 1.0 mm (0.10 mV) ee ith u ation > 40 ms

Mono hasic an u i ht Notche p ave ith u ation > 120 ms

r i ht at ia en a ement

Bi hasic positive ( oxima ) e ection > 1.5 mm (0.15 mV) ta

Mono hasic an u i ht pointe p ave ith am itu e > 2.5 mm (0.25 mV)

Left ventricular hypertrophy (LVH) causes a general increase in QRS voltage amplitude. There are several established criteria for the diagnosis of LVH. Of these, the Cornell criteria are considered the most sensitive and specific. With the Cornell criteria, one simply adds the height of the R wave in lead aVL to the depth of the S wave in lead V3; summed voltage > 28 mm for men and > 20 mm for women are indicative of LVH. Right ventricular hypertrophy is recognizable when there is a dominant R wave in the early precordial leads (R/S ratio in V1 > 1, R wave height in V1 > 7 mm, R in V1 + S in V5 or V6 > 10.5 mm) and rightward QRS axis deviation ( ≥100°).

Conduction Disease Abnormal electrical conduction can occur at any level from the sinoatrial node to the bundle branches and Purkinje fibers. Disorders affecting the sinus node (sick sinus syndrome, sinus node arrest) may be difficult to distinguish from disorders affecting atrial activation by the sinus node (sinoatrial exit block) as both can result in intermittent or complete failure to generate atrial waveforms or propagate sinus impulses to the ventricles. Disorders of the atrioventricular node (AVN) are not uncommon and are characterized by delayed or ineffective transmission of atrial impulses to the ventricles. First-degree AV block is characterized by fixed prolongation of the PR interval (>200 ms) and is a common finding with a long list of causes, including increased vagal tone and the effect of innumerable medications. There are two types of seconddegree AV block. Mobitz type 1 AV block (often called Wenckebach AV block) is characterized by gradually lengthening of the PR interval until a point is reached when an atrial impulse arrives while the AVN is still refractory, resulting in failure to produce a QRS (Figure I-18a). This pause allows the AVN to repolarize and the cycle repeats itself. Wenckebach AV block is usually caused by dynamic inhibition of the AV node and is often the result of transient insults such as ischemia, vagal inhibition and drug effects. In Mobitz type 2 AV block the AV node can only handle a set number of consecutive atrial impulses before it fails to produce a QRS (Figure I-18b). The PR interval does not vary, but the ratio of conducted beats to blocked beats tends to follow an integral pattern such as 3:1 or 4:1. Mobitz type 2 AV block is more commonly the result of fixed injury to the AV node due to infarction,

38

CASE FIl ES: CAr d IO l Og y

A

B

II C

II D

P wave s, a tria l ra te 75 bpm R wave s, ve ntricula r ra te 60 bpm

Figure I-18 Exam es o at iovent icu a (AV) b ock: (a) secon - e ee AV b ock, Mobitz t e 1 (wenckebach)–note the a ua o on ation o the pr inte va io to oss o AV con uction; (b) secon - e ee AV b ock, Mobitz t e 2–note the ixe pr inte va ith oss o con uction eve thi beat (3:1 AV b ock); (c) secon - e ee AV b ock ith 2:1 con uction–this cou be Mobitz t e 1 o 2 (see text o a itiona etai s); ( ) thi - e ee AV b ock–this atient is in sinus h thm ith com ete AV b ock an an un e in junctiona esca e h thm.

trauma, or age-related degeneration, and it carries a greater risk for progression to complete AV block than Wenckebach AV block. Occasionally patients will present with second-degree AV block with 2:1 conduction (Figure I-18c). The mechanism for this may be consistent with Mobitz type 1 or type 2 second-degree AV block; there are clinical and ECG features that can help you differentiate between the two (Table I-11).

Table I-11 SECOND-DEGREE AV BLOCK WITH 2:1 AV CONDUCTION * Favors Mobitz Type 1 If …

Favors Mobitz Type 2 If …

Na o / unchan e

wi e

r es onse to maneuve s that inc ease hea t ate o e uce va a tone (e , at o ine)

B ock im oves

B ock o sens

r es onse to maneuve s that ec ease hea t ate o inc ease va a tone (e , Va sa va)

B ock o sens

B ock im oves

In the settin o an acute m oca ia in a ction

In e io ischemia o inju

Ante io ischemia o inju

Feature Qr S i th (com a e to no ma o to base ine)

*

evious

These acto s avo Mobitz t e 1 ve sus Mobitz t e 2 as the cause.

SECTION I: H O w TO Ap p r O ACH TH E CAr d IO l O g y p ATIENT

39

The distinction has some clinical relevance as Mobitz type 2 AV block is associated with a higher risk for progression to complete AV block than Mobitz type 1. Thirddegree AV block is characterized by complete electrical disconnection between the atria and ventricles. Also called complete heart block, this abnormality is characterized by the presence of an atrial rhythm with a regular P-P interval and fixed rate that is unrelated to the regular R-R interval and (usually) slower rate produced by a secondary “escape” rhythm, typically arising from the AVN-His junction. Bundle branch blocks result from injury to the right and left bundle branches. This produces a wider, abnormal-appearing QRS complex as more of the ventricle is activated via cell-to-cell conduction (Figure I-19). In right bundle branch block the QRS complex begins normally as the usually left-to-right activation of the interventricular septum and left ventricle is unaffected. The latter portion of the QRS is wide and abnormal as the right ventricle slowly depolarizes, producing a characteristic second R wave or R′ in lead V1. With left bundle branch block the QRS is abnormal from the start as the interventricular septum actives from right to left via the intact right bundle. The entire left ventricle then depolarizes from the right, resulting in an extremely wide QRS with a characteristic slurred appearance.

Arrhythm ias Tachyarrhythmias and bradyarrhythmias are discussed in greater depth later in this book, so we will only review some general features here. One important question to ask during the interpretation of every ECG is whether the rhythm is in fact sinus. Sinus rhythm is defined by the presence of P waves with an axis between 0° and 70° (essentially upright in the inferior leads) that, provided the AV node is working properly, results in corresponding QRS complexes. If these criteria are not met, then the next question to ask is whether the rhythm is originating from above or below the AV node.

Right bundle bra nch block

A

Le ft bundle bra nch block

V1

V4

V1

V4

V2

V5

V2

V5

V3

V6

V3

V6

B

Figure I-19 Bun e b anch b ocks: (a) i ht bun e b anch b ock is cha acte ize b Qr S u ation ≥120 ms, sr ′ o Sr ′ atte n in V1 an / o V2, Qr S- eak r u ation > 50 ms in V1 an V2 an i e, s u e S aves in I, V5, an V6; (b) e t bun e b anch b ock is cha acte ize b Qr S u ation ≥ 120 ms (c ose to 160 ms), S o QS in V1, Qr S- eak r ave u ation >50 ms an b oa , mono hasic r aves in I, V5, an V6.

40

CASE FIl ES: CAr d IO l Og y

Supraventricular rhythms tend to produce narrow QRS complexes or QRS complexes that are identical to the patient’s native QRS during sinus rhythm. If the rhythm appears to be supraventricular, then the presence of atrial waveforms will help distinguish automatic atrial arrhythmias (atrial tachycardia, atrial flutter) from reentrant supraventricular rhythms that produce no obvious antegrade atrial waves (AV nodal reentry or AV reentry). An irregular supraventricular rhythm with no obvious atrial waveforms is almost certain to be atrial fibrillation. Wide complex tachycardias are often due to ventricular tachycardia, especially in adults with structural heart disease. If you can clearly demonstrate the presence of A-V dissociation on your ECG, then the rhythm is certain to be of ventricular origin. If you are uncertain as to whether a wide complex tachycardia has a ventricular origin (vs a supraventricular origin with aberrant ventricular conduction), it would be helpful to compare your ECG to a baseline tracing to see whether the QRS complexes have similar morphologies. If the wider QRS appears to have the same morphology as the baseline QRS, then the rhythm is more likely to be an aberrantly conducted supraventricular rhythm. However, in older adults with established cardiovascular disease or major risk factors for it, a wide complex tachycardia is usually ventricular tachycardia. Whenever there is doubt, it is safer to treat for presumed VT in these patients.

Part 3. Cardiovascular Providers and Procedures Through my interactions with graduating medical students and newly minted interns over the years, I have come to realize that many young physicians have little idea about what a cardiologist actually is or does. To the outside observer, the management of patients with heart disease might seem utterly chaotic, and the specific roles of the patient’s primary care physician, consulting cardiologist, consulting surgeon, and other providers may not be entirely clear. This part of Section I focuses on the types of physicians who specifically train to care for patients with cardiovascular disease and highlights some of the more common procedures these individuals perform to facilitate that care. Three major topics are discussed here:

A. CARDIOVASCULAR PROVIDERS Medically Trained Providers In the United States all postgraduate medical training programs are required to maintain accreditation by the Accreditation Council for Graduate Medical Education (ACGME). The ACGME routinely audits individual training programs for their clinical, educational, and procedural quality and to ensure compliance with various safety standards. Completion of an accredited training program then allows one to sit for a board certification examination provided by an academic society such as the American Board of Internal Medicine (ABIM) or the American Board of Thoracic Surgery (ABTS). These groups set quality standards for their diplomates and ongoing maintenance of certification by periodic reexamination, and the

SECTION I: H O w TO Ap p r O ACH TH E CAr d IO l O g y p ATIENT

41

accumulation of continuing medical education (CME) credits is required over the course of one’s entire career. Adult cardiovascular disease is a subspecialty of the ABIM. Training in adult cardiovascular medicine requires successful completion of training in general internal medicine. Most cardiovascular medicine fellowship programs are 3 years long, and on successful completion of an accredited training program, a cardiologist is eligible to sit for the ABIM certification examination in adult cardiovascular medicine. Cardiologists who begin practice after general fellowship training are typically referred to as general cardiologists or clinical cardiologists. Core general cardiology training focuses on the diagnosis and medical management of heart and vascular disease in its entirety, using fundamental cardiovascular procedures such as diagnostic coronary angiography, invasive hemodynamic assessment, cardiac imaging modalities, and inpatient and outpatient clinical care. Additionally, procedure-specific certification for some imaging procedures such as nuclear cardiology and echocardiography can be obtained from academic societies such as the American Society of Echocardiography (ASE). After completion of training in adult cardiovascular disease, physicians may further advance their training in a specific area of cardiology by pursuit of a cardiology subspecialty fellowship. Subspecialty fellowship training programs are typically 1 or 2 years in length and often focus on acquisition of procedural expertise in a particular field. The ABIM offers subspecialty certification in interventional cardiology, clinical cardiac electrophysiology, advanced heart failure and transplantation, and adult congenital heart disease. Interventional cardiologists perform percutaneous interventions to treat certain forms of heart disease such as coronary and peripheral arterial disease. Many programs also offer training in the catheter-based management of structural heart disease. Most accredited fellowship programs are 2 years long and include training in coronary artery, peripheral vascular, and structural cardiac interventions. Cardiac electrophysiologists specialize in the medical and catheter-based management of cardiac arrhythmias. These 2-year training programs provide instruction in catheterbased diagnostic and therapeutic procedures for the treatment and management of rhythm disorders and the implantation of cardiac devices such as pacemakers and automated cardiac defibrillators. Heart failure specialists focus on the medical treatment of patients with advanced ventricular failure. These clinical training programs also allow one to acquire expertise in the evaluation and management of patients before and after cardiac transplantation, including the proper management of patients with implanted ventricular assist devices, evaluation and listing of potential transplantation candidates, and the management of chronic immune suppression after transplantation. Adult congenital heart disease is a newly accredited 1-year training program that arose to address the needs of our constantly increasing population of patients with congenital heart disease who survive into adulthood. Training in this area focuses on gaining balanced expertise in the procedural, medical, and imaging aspects of care for adult patients with congenital heart disease. Physicians who have successfully completed training in an accredited adult cardiovascular or pediatric disease fellowship program are eligible to apply.

42

CASE FIl ES: CAr d IO l Og y

In addition to the ABIM-accredited cardiovascular subspecialties mentioned above, there are other advanced cardiovascular medicine fellowships. Cardiac imaging fellowships allow one to focus on the diagnostic aspects of cardiology using modalities such as advanced echocardiography, computed tomography (CT), nuclear cardiology, and magnetic resonance imaging (MRI). Preventive cardiology focuses on the primary and secondary prevention of cardiovascular disease and the promotion of healthier lifestyles by integrating education with interventions such as supervised cardiac exercise. Vascular medicine fellowship can be pursued by physicians who have completed training in cardiology or internal medicine. These physicians focus on the medical and office-based procedural management of arterial and venous diseases. They also acquire experience in the diagnosis and treatment of hemostatic diseases. The scope of practice among cardiologists is quite variable. Some subspecialty cardiologists maintain their general cardiology skills and maintain a diverse practice that can include percutaneous coronary interventions, device implantation, imaging expertise, and the clinical management of a broad range of cardiovascular diseases. This is more common with cardiologists in private practice, particularly in geographic areas where broad subspecialty care may be lacking. Conversely, other cardiologists, particularly those in larger academic centers, may focus the majority of their efforts on a solitary aspect of cardiovascular medicine such as percutaneous valve interventions or catheter-based ablation of cardiac arrhythmias. A cardiologist can have a career that is entirely office-based, entirely procedure-based, and everywhere in between.

Surgically Trained Providers Surgeons also care for patients with cardiovascular disease and typically work in close partnership with cardiologists and other medicine-trained providers. The American Board of Thoracic Surgery certifies cardiothoracic surgeons after successful completion of an accredited general surgery residency (5 years) followed by a thoracic surgery residency (3 years), successful completion of an accredited joint general and thoracic surgery program (7 years total), or successful completion of a Thoracic Surgery Directors Association (TSDA)–sponsored and ACGME-accredited integrated thoracic surgery residency (6 years). Cardiothoracic surgeons are trained to perform surgical interventions on the heart, lungs, thoracic vessels, and foregut (mediastinum, esophagus, and stomach). Thoracic surgery training programs provide training in all of these disciplines, but in practice many surgeons focus on particular aspects of that training such as cardiac surgery, lung surgery, and/or foregut surgery. Vascular surgeons are trained to perform open surgical and percutaneous interventions on the aorta and peripheral blood vessels. The American Board of Surgery (ABS) offers certification to physicians who have completed training in an accredited general surgery program (5 years) and vascular surgery fellowship program (2 years) or in a combined general/vascular surgery residency program (5−6 years). In recent years the boundaries between cardiologists and surgeons have become less well defined. This is due largely to the advancement of catheter-based therapies such as percutaneous revascularization, transcatheter aortic repair, and transcatheter valve replacement. These procedures may be performed by either interventional

SECTION I: H O w TO Ap p r O ACH TH E CAr d IO l O g y p ATIENT

43

cardiologists or surgeons. In many centers this has increased the opportunity for collaboration between medical and surgical caregivers, allowing for hybrid, minimally invasive procedures for select patients. Collaborative opportunities such as these have influenced the organizational structure of many medical centers. With increasing frequency medical centers are moving away from a traditional departmental structure with separate departments of medicine and surgery in favor of an institute model that unites medical and surgical providers in the same administrative “home” that is centered around the management of organ system-based diseases. The institute model is designed to maximize collaboration between medical and surgical providers, promote clinical investigation, and eliminate waste or redundancy.

B. COMMON DIAGNOSTIC PROCEDURES Cardiologists employ a broad array of diagnostic tests and procedures to accurately diagnose and treat patients with cardiovascular disease. The following text provides an overview of some of the more commonly used diagnostic studies. Each modality described below has its own strengths and limitations. However, a few “universal truths” apply to all cardiovascular tests (in fact, to all tests in general). Prior to ordering any study, it is important to ask yourself “Will the results of this study change my management?” If the answer is “yes,” then the study is likely worth doing. If the answer is “no,” then one should question whether it needs to be done at all. For example, in a patient with pleuritic chest pain at low risk for venous thromboembolic disease, a normal D-dimer assay has a negative predictive value of nearly 100% for the diagnosis of pulmonary embolism. A contrast-enhanced CT of the chest to rule out pulmonary embolism is unlikely to influence your management and would expose the patient to needless risk from radiation and contrast. Another important concept to appreciate is the influence of probability of disease on the outcome of your test. Simply interpreted, Bayes’ theorem states that the probability that a patient with a positive test result actually has the disease being tested for is influenced by the prevalence of the disease in the study population. For example, an abnormal stress test in a 25-year-old healthy woman with no symptoms is most likely to be a false-positive finding, whereas the same abnormality in a 75-year-old male smoker with diabetes and classic angina symptoms is most likely a true positive. One must also weigh the risks involved in performing any test against the benefit provided by having the test result. Most cardiovascular tests carry little risk of physical harm; however, invasive studies, studies employing ionizing radiation, or those that require the use of potentially nephrotoxic contrast agents can pose risk to the patient.

Noninvasive Diagnostic Procedures Echocardiography employs ultrasound waves to visualize the heart and surrounding structures by exploiting the fact that different tissues reflect sound waves to differing degrees, based on tissue characteristics. Reflected waves returning to the ultrasound transducer are processed by the machine to convert these differences into spectral waveforms and two- or three-dimensional (2D and 3D) anatomic images of the heart (Figure I-20). Echocardiograms provide both static and real-time dynamic information regarding cardiac chamber size, systolic and diastolic function, valvular integrity, and the integrity of the surrounding tissues. Transthoracic

44

CASE FIl ES: CAr d IO l Og y

Fre q.: 1.7 MHz/3.3 MHz Proc.: 26.0/12.0/2.0/6.0/1.4 FP S : 61.5 De pth: 18.0 cm

V 5

10

LV RV

15

RA

LA

?? Figure I-20 T o- imensiona echoca io a hic ima in om the a ica ou -chambe vie (l A, e t at ium; l V, e t vent ic e; r A, i ht at ium r V, i ht vent ic e).

echocardiography (TTE) is performed with the ultrasound transducer placed on the surface of the chest. TTE provides accurate assessment of biventricular function, valve performance, and pericardial disease. Occasionally one needs to see posteriorly situated structures of the heart (mitral valve, left atrial appendage) with greater clarity than a transthoracic echocardiogram can provide. In these cases a transesophageal echocardiography (TEE) can be performed by having the patient swallow a long transducer probe with an ultrasound crystal at the tip. Echocardiography has several advantages, including its ubiquitous availability, general safety (although TEE does carry some small risk for esophageal and dental injury), and relatively low cost. Disadvantages include limited visualization of the heart in some patients due to technical issues such as obesity or advanced lung disease. Echocardiographic visualization can be enhanced in these cases with the use of echocardiographic contrast agents composed of noble gas−containing phospholipid “microbubbles” that are metabolized within the circulation to fatty acids within minutes of injection. Computed tomography (CT) scanning is also a useful tool in the diagnosis of cardiovascular disease. CT involves the use of ionizing radiation to create detailed 2D and 3D anatomic images of the heart and surrounding chest structures using computerized processing of multiple images obtained from an external single axial source of radiation. CT is useful for the assessment of coronary and pulmonary artery anatomy, aortic anatomy, cardiac tumors, and the relationship between the heart and adjacent structures in the chest (Figure I-21). Advantages of CT imaging include its relatively wide availability and high speed of image acquisition. Disadvantages include the exposure risk related to moderate-to high-dose ionizing radiation and nephrotoxic iodinated contrast. Magnetic resonance imaging (MRI) produces images of the heart by placing the patient within the field of a strong electromagnet. The magnetic field excites hydrogen ions, which then emit radiowaves that are collected and processed into viewable 2D and 3D anatomic images (Figure I-22). MRI images are occasionally enhanced via the use of intravenous chelated metal-containing contrast agents such

SECTION I: H O w TO Ap p r O ACH TH E CAr d IO l O g y p ATIENT

45

Cleve la nd clinic Brillia nce iCT P hilips -172C211 Rows : 512 Cols : 512

H

S VC PA

AO LAD RA

RV

A

P

CX LV

10.00 mm/div

%R-R: 75.0 120.000 kV 413 mA Tilt: 0.000 FOV: 178.000 mm Thickne s s : 0.900 mm

F

10.00 mm/div

Figure I-21 Ca iac CT ith 3d econst uction (l V, e t vent ic e; pA, u mona a te ; r A, i ht at ium; r V, i ht vent ic e; SVC, su e io vena cava). The e t ante io escen in (l Ad ) an ci cumex (CX) co ona a te ies a e sho n.

100

RA

Ao

LA

A R I

P L S

LV

A

B

Figure I-22 Ca iac Mr I in a atient ith m oca itis: (a) cont ast-enhance ima es om a on -axis vie ; (b) e a e cont ast enhancement in the inte o ate a a (a o s) consistent ith m oca ia in ammation (Ao, ao ta; l A, e t at ium; l V, e t vent ic e; r A, i ht at ium).

46

CASE FIl ES: CAr d IO l Og y

as gadolinium. Cardiac MRI has a number of uses, including the precise assessment of ventricular function, quantification of valvular regurgitation, visualization of the aorta and other chest structures, and differentiation between healthy (normal, viable) and unhealthy (ischemic, scarred, or inflamed) myocardial tissue. Advantages of cardiac MRI include the relative objectivity of its findings (compared with echocardiography and CT) and general safety. Disadvantages include its higher cost, limited availability, and lack of safe use in subjects with implanted metallic devices or claustrophobia. In rare circ*mstances the contrast agents used in MRI may result in nephrotoxicity. N uclear medicine imaging is also used for the diagnosis and management of patients with heart disease, particularly ischemic heart disease. In cardiac nuclear imaging studies patients are given intravenous radiopharmaceutical agents that emit detectable energy waves that are subsequently detected, processed, and formatted as 2D or 3D anatomic images (Figure I-23). Single-photon emission computed tomography (SPECT) studies utilize gamma-emitting moieties such as technetium-99 or thallium-201, whereas positron emission tomography (PET) relies on positronproducing compounds such as fludeoxyglucose (fluorine-18) and rubidium-82. Unlike CT scans, where the radiation passes through the patient from an external source prior to detection, in nuclear cardiology studies the radiation source comes from within the patient. Nuclear studies are used predominantly for the assessment of myocardial health, including the presence of ischemia, scarring, viability, and inflammation. Advantages of nuclear scans include their relative safety (radiation dose is extremely low) and relatively wide availability (SPECT in particular). Disadvantages include the limited spatial resolution and long scan times for SPECT and the increased cost and limited availability of PET.

Stress Testing Noninvasive assessment for the presence of myocardial ischemia can be achieved using a variety of studies, all of which are colloquially referred to as “stress tests.” These studies are the most clinically useful in subjects with symptoms that may be consistent with angina pectoris who are at moderately increased risk for underlying coronary artery disease. Every noninvasive ischemic study has two principal components: a stimulus to provoke ischemia and a surrogate marker to measure for it. The two stimuli employed for the provocation of myocardial ischemia are exercise (treadmill or stationary bicycle) or a pharmacologic agent that can either increase heart rate and contractility (dobutamine infusion) or induce coronary vasodilatation (adenosine or dipyridamole). Exercise is superior to pharmacologic agents for both the provocation and electrocardiographic assessment of ischemia, and an exercise study should always be arranged unless the patient cannot safely exercise. The presence of ischemia can be detected in several ways. All noninvasive ischemic evaluations employ continuous ECG monitoring as a means to assess for ischemia. ST segment depression is fairly sensitive for the detection of myocardial ischemia, but stress ECG alone lacks specificity for the detection of coronary disease, particularly in women. Still, a treadmill ECG is a reasonable study to perform in a low-to-moderate-risk patient with a normal resting ECG as the negative predictive

Pe rfus ion

Pe rfus ion ANT

S EP

Reve rs ibility ANT

21502.1

LAT

S EP

LAT

0.0

S EP

INF

B

50.0%

LAT

0.0

INF

A

ANT

19623.0

INF

0.0%

C

Figure I-23 r ubi ium-82 m oca ia pET ima es. These ima es e e taken u in est (a) an a te st ess (b) om a atient ith sus ecte co ona a te isease. The e t vent icu a e usion ma is o iente in the sho t axis ith the a ica se ments in the cente o the ma an the basa se ments on the e i he . Note the ma ke oss o e usion in the in e io an ate a e t vent icu a se ments ith st ess. This e esents ive se ments o mo e ate eve sib e ischemia (c); this atient as oun to have hi h- a e stenosis o a ominant ci cum ex co ona a te (ANT, ante io ; INF, inte e on; l AT, ate a ; SEp, se ta ).

48

CASE FIl ES: CAr d IO l Og y

value of a normal test is quite good. Pre- and postexercise echocardiographic imaging can be added to increase the specificity of the study, and this modality is commonly paired with dobutamine in patients who cannot exercise. Alternatively, one can perform nuclear imaging (SPECT or PET) to augment the specificity. Nuclear studies are commonly performed with vasodilator stimulation in patients who cannot exercise. The literature comparing the accuracy of echocardiography versus nuclear imaging for the assessment of ischemia has yielded some inconsistent conclusions regarding which modality is better; in general, one can consider them to be comparably accurate. Regional expertise, cost, availability, and patient-specific factors should influence the decision about which type of study to order. For example, patients with wheezing or known reactive airway disease may not tolerate adenosine or dipyridamole, while patients with atrial fibrillation can become quite tachycardic on dobutamine. Morbidly obese patients might be better suited for a PET imaging study, whereas patients with undiagnosed murmurs would be better suited for echocardiography, as it would also provide visualization of the cardiac valves.

Invasive Diagnostic Procedures Angiography is a procedure performed to diagnose intraluminal vascular disease, particularly arterial atherosclerosis and acute thromboembolic occlusion. During an angiogram the inner lumen of a blood vessel (typically an artery, but venous angiography is also performed) is opacified using an iodinated contrast agent that is directly injected into the vessel of choice via a hollow catheter. Images of the vessel lumen are obtained using a mobile radiographic image intensifier that is positioned by the operator via a mechanized gantry. Angiography can be performed in virtually any arterial bed. The most commonly studied arteries include the coronary arteries, carotid and cerebral arteries, distal lower extremity arteries, and pulmonary arteries (Figure I-24). Angiography is an invasive procedure that carries a small risk

RAO 1 CRAN 27

RAO 23 CAUD 24

LMCA LAD

RCA

CX

Figure I-24 Co ona an io am. r e ative no ma co ona an io am is sho n (CX, ci cum ex; l Ad , e t ante io escen in ; l MCA, e t main; co ona a te ; r CA, i ht co ona a te ).

SECTION I: H O w TO Ap p r O ACH TH E CAr d IO l O g y p ATIENT

49

for hemorrhage, infection, vessel rupture, myocardial infarction, stroke, and even death. Other risks include those related to contrast exposure and the use of moderate sedation. For diagnostic coronary angiography, the combined risk for a major adverse event (death, nonfatal myocardial infarction, stroke, or vascular injury requiring emergency surgery) is approximately 0.1%. Pulmonary artery catheterization (PAC), also referred to as right-heart catheterization or Swann-Ganz catheterization, can be useful for the diagnosis and management of such conditions as pulmonary hypertension, decompensated heart failure, and shock. During PAC a long, balloon-tipped hollow catheter is advanced from a central venous access point (typically the right internal jugular vein) through the right-heart chambers and into the branches of the pulmonary artery tree. The catheter has a pressure transducer distal to the balloon which is then “wedged” into a small pulmonary arteriole, allowing the catheter to read the pulmonary venous pressure that is reflected back to it through the capillary bed (Figure I-25). This pulmonary capillary wedge pressure (PCWP) serves as a surrogate for left atrial pressure and therefore left ventricular filling pressure. PAC can be performed as an isolated diagnostic procedure in the catheterization laboratory or as a bedside procedure in an intensive care setting. With the latter, the catheter typically remains in place so that management may be guided by the PAC data. Once the patient has been stabilized, the catheter is typically removed. PAC carries some risk for pulmonary arteriolar rupture, atrial and ventricular tachyarrhythmias, infection, and bleeding. Patients with an underlying left bundle branch block are also at risk for complete heart block because the catheter may irritate the right ventricular side of the interventricular septum and cause transient right bundle branch block.

Pulmona ry ve in

Pulmona ry a rte ry Ca the te r

Ba lloon

Pulmona ry ca pilla ry be d

Le ft a trium

mmHg 30 25 20 15 10

Pulmona ry a rte ry pre s s ure

Pulmona ry we dge pre s s ure

Le ft a tria l pre s s ure

Figure I-25 I ust ation o u mona a te ba oon occ usion to measu e the u mona (a te ) ca i a e e essu e (pCw p). w ith the ba oon in ate , the cathete ti eceives essu e e ecte back to it om the a te ia si e o the ca i a be . This is a e ecte ve sion to e t at ia essu e.

50

CASE FIl ES: CAr d IO l Og y

Electrophysiology (EP) testing is performed to evaluate and treat a wide spectrum of cardiac arrhythmias. In this procedure arterial and venous access is obtained (typically from the femoral position) and electrode-tipped catheters are advanced toward key electrophysiologic structures within the heart. These electrodes can both record and stimulate electrical conduction, allowing the electrophysiologist to create a 3D electrical map of the heart using sophisticated medical software. In this way the origins and pathways of various rhythm abnormalities are identified and even treated via radiofrequency ablation. EP testing carries an obvious risk for arrhythmia but also cardiac perforation, pericardial tamponade, blood loss, infection, and thermal damage to the esophagus, which lies quite near the posterior aspect of the left atrium.

C. THERAPEUTIC PROCEDURES Catheter-based procedures can be used for therapeutic interventions in addition to diagnosis. The advancement of catheter-based therapies in cardiovascular medicine since the early 1980s has been extensive, offering the option of endovascular treatment to an ever-growing list of cardiac disorders. The key feature of all percutaneous procedures is use of the modified Seldinger technique (named for Swedish radiologist Dr. Sven-Ivar Seldinger, 1921−1998). The Seldinger technique involves the use of a hollow-bore needle to introduce a flexible metal guidewire into an artery or vein. Over this guidewire one places a vascular sheath that consists of a relatively short, hollow catheter with a one-way diaphragm or valve that allows wires and catheters to be introduced into the vessel but prevents back bleeding. Through this sheath longer J-tipped guidewires wires can be negotiated through the vascular system to the point of interest using imaging (typically radiographic) guidance. Diagnostic and therapeutic catheters and equipment are then advanced over the wire like a monorail via a dedicated central lumen. The guidewire is removed and readvanced every time there is a catheter or equipment change. The Seldinger technique has made all percutaneous diagnostic and therapeutic interventions possible, from the simple placement of a central venous catheter to the percutaneous repair of the aorta.

Percutaneous Vascular Intervention Percutaneous coronary intervention (PCI) can be performed to treat stenosis of the coronary arteries. During these procedures a small, steerable guidewire is advanced across the narrowed area (visualized via angiography), a balloon-tipped catheter is advanced into place, and the balloon is inflated to a set pressure with contrasted saline using a handheld inflation device. The expanding balloon crushes the plaque into the vessel wall. Most PCIs also involve deployment of an intracoronary stent. A stent is a balloon-expandable wire mesh tube that retains its size and cylindrical shape after balloon expansion (Figure I-26). Stents are composed of stainless steel or a metallic alloy such as cobalt chromium and stents may be bare metal stents (BMSs) or drug-luting stents (DESs). DESs are coated with a biodegradable polymer that contains an antineoplastic agent such as everolimus, pacl*taxel, or zotarolimus. DES use is associated with a significant reduction in the risk for stent restenosis, but patients typically need a longer duration of dual antiplatelet therapy (eg, aspirin and

SECTION I: H O w TO Ap p r O ACH TH E CAr d IO l O g y p ATIENT

51

A

B

C

Figure I-26 I ust ation o co ona a te an io ast : (a) an occ usive co ona aque; (b) an ioast ba oon in ation ithin the stenosis. (Reprinted with permission, Cleveland Clinic Center for Medical Art & Photography © 2013. All rights reserved.)

a thienopyridine) to prevent stent thrombosis and allow endothelialization of the stent. Angioplasty and stenting may be performed in noncoronary arteries as well, including the carotid arteries, renal arteries, and the peripheral arteries of the lower extremities. Since an angiogram only provides information regarding the relative size and contour of the vessel lumen, other intraarterial imaging modalities have been developed to aid in the diagnosis and management of arterial disease, coronary artery disease in particular. Intravascular ultrasound (IVUS) involves the use of a guidewire with an ultrasound crystal near the tip. The wire is introduced to the vessel of choice and gradually withdrawn, providing a 2D image of the vessel lumen and wall in real time that can be correlated with the patient’s angiogram. IVUS is commonly used after PCI to assess the adequacy of stent deployment (Figure I-27). Fractional flow reserve (FFR) is another ultrasound-based intravascular modality that is commonly employed to assess the flow characteristics of a moderate angiographic stenosis. Using Doppler ultrasound the flow across the stenotic region is compared with flow in an angiographically normal segment of the same vessel after vasodilatation with adenosine. The FFR is a ratio of lesion flow to reference segment flow; an FFR of 0.8 or less is generally considered “flow-limiting.” FFR is most commonly used to guide PCI in patients with angiographic stenoses of questionable severity. Percutaneous technology is also used to replace the aortic valve during transcatheter aortic valve replacement (TAVR). During the TAVR procedure a large

52

CASE FIl ES: CAr d IO l Og y

B A

Figure I-27 I ust ation o int avascu a u t asoun (IVUS). The e is a stent e o e in a co ona a te to a ess a o - imitin stenosis. Fo o in stent acement the u t asoun i e is a vance into the stent to assess o a o iate e o ment an a osition (a). Acuta IVUS ima e is sho n in ane (b) ith the st uts o the stent (a o s). (Reprinted with permission of Cleveland Clinic Center for Medical Art & Photography © 2013. All rights reserved.)

introducer sheath is placed within a large artery (femoral, ascending aorta) by the Seldinger technique or via the left ventricular apex via a partial thoracotomy incision. A guidewire is advanced across the aortic valve, and this is followed by balloon valvuloplasty and deployment of a bioprosthetic valve situated within a balloonexpandable stent. During valve deployment the heart is never stopped, but the patient is subjected to rapid right ventricular pacing via a temporary right ventricular pacing wire to transiently halt cardiac output. Similar percutaneous interventions have also been performed to replace the pulmonic and tricuspid valves. In addition to valve replacement, one can also attempt percutaneous repair of the regurgitation mitral valve by deployment of a clip to hold the tips of the mitral leaflets together. Aortic aneurysms can also be repaired percutaneously via a procedure called endovascular aortic repair (EVAR). Employing the Seldinger technique, a surgeon or interventional cardiologist may introduce an expandable endovascular stent graft over a balloon in a manner analogous to percutaneous coronary intervention (Figure I-28). EVAR can be employed to treat abdominal or descending thoracic aortic disease, and its use has dramatically decreased the need for open aneurysm repair, a large and highly morbid surgery.

Electrophysiology Procedures As described previously, patients with rhythm disorders can be diagnosed using an invasive EP study. During an EP study an electrophysiologist may be able to offer

SECTION I: H O w TO Ap p r O ACH TH E CAr d IO l O g y p ATIENT

53

Endova s cula r s te nt gra ft

Conduit from ope n re pa ir

Figure I-28 I ust ation o an EVAr oce u e in a atient ith a evious o en e ai o an ascen in ao tic issection. The ista as ect o the su ica im ante con uit is sho n an se ves as an ancho o the en ovascu a stent a t. This t e o t o-sta e e ai is e e e to as an “e e hant t unk” e ai . (Reprinted with permission, Cleveland Clinic Center for Medical Art & Photography © 2013. All rights reserved.)

a therapeutic intervention such as catheter-based ablation of an abnormal focus of reentrant or automatic electrical activity. Catheter-based ablation is presently used to treat a number of rhythm disorders such as atrial flutter, atrial fibrillation, and ventricular tachycardia. In some patients with treatment-refractory atrial fibrillation with rapid ventricular heart rates, the atrioventricular node itself can be ablated and a pacemaker installed in order to control the heart rate. Electrophysiologists can also implant devices such as pacemakers and defibrillations for the management of conditions such as symptomatic bradyarrhythmia and heart failure and for the prevention of sudden cardiac death in patients with reduced left ventricular systolic function or a prior history of ventricular tachyarrhythmia. Device implantation involves the placement of electrode wires within the right atrium and right ventricle or on the epicardial surface of the left ventricle via the retrograde placement of an electrode in a cardiac vein via the coronary sinus (Figure I-29). The wires are typically introduced via the left subclavian vein and connected to a device generator placed in a subcutaneous pocket created in the left pectoral area.

Surgical Procedures Despite advancement of medical and percutaneous therapies for many forms of cardiovascular disease in recent years, a substantial number of patients require open cardiothoracic and vascular surgeries each year. These surgeries are invasive and

54

CASE FIl ES: CAr d IO l Og y

Ele ctrode s

Ge ne ra tor

Figure I-29 Ca iac es nch onization the a e ib i ato (Cr T-d ) evice. Th ee e ect o es a e int o uce om the e t subc avian vein an embe in the i ht at ium, i ht vent ic e, an a ca iac vein on the e ica ia su ace o the hea t nea the ate a a o the e t vent ic e. The e ica ia e ect o e is ace in a et o a e ashion om the i ht at ium via the co ona sinus. The i es a e connecte to a ene ato unit ace in the e t ecto a osition. (Reprinted with permission, Cleveland Clinic Center for Medical Art & Photography © 2013. All rights reserved.)

carry considerable risk related not only to the cardiac issue being addressed but also to the technical aspects of surgery that are required to create a stable, blood-free operative field such as full circulatory arrest, cross-clamping of the aorta, and the use of a mechanical cardiopulmonary bypass circuit. Nonetheless, with the passage of time and with ongoing refinement of surgical techniques, these surgeries are becoming increasingly safer and less morbid. Surgical coronary revascularization or coronary artery bypass grafting (CABG) is performed to treat advanced coronary artery disease. CABG is deemed superior to PCI in patients with severe multivessel disease that involves the left main coronary or proximal left anterior descending (LAD) coronary artery, particularly in patients with diabetes or left ventricular systolic dysfunction. CABG surgery is performed in most cases with a median sternotomy incision. Whenever possible, the left internal mammary artery is used as a bypass conduit because the use of the mammary conduit is associated with improved survival and better long-term bypass patency. The proximal end of the mammary artery remains in situ off the left subclavian artery, while the distal portion of the artery is removed from the interior chest wall and anastomosed to the LAD on a segment distal to the site of angiographic stenosis. In some patients the right internal mammary artery may also be used, although bilateral mammary use is generally avoided in diabetic patients because of risks for poor healing and mediastinal infection. Additional aortocoronary bypass grafts may

SECTION I: H O w TO Ap p r O ACH TH E CAr d IO l O g y p ATIENT

A

55

B

Figure I-30 Mit a va ve e ai : (a) qua an u a esection o a iece o e un ant ea et tissue in a atient ith seve e mit a e u itation; (b) the inishe e ai , com ete ith the acement o a oste io annu o ast ban . (Reprinted with permission, Cleveland Clinic Center for Medical Art & Photography © 2013. All rights reserved.)

be created using segments of harvested greater saphenous vein or radial artery; the proximal portion of the graft is anastomosed to the ascending aorta, and the distal segment of the graft is anastomosed to the target vessel distal to the site of stenosis. Valve surgery is indicated in patients with symptomatic severe valvular regurgitation or stenosis or in asymptomatic patients with high-risk features such as left ventricular dilatation or dysfunction. Surgical valve repair is most commonly performed on regurgitant atrioventricular valves, the mitral valve in particular. Valve repair involves debridement of redundant or damaged valvular tissue and occasional repair or replacement of damaged chordae tendonae. This surgery typically involves the placement of a flexible semicircular annuloplasty band to improve leaflet apposition and maintain the integrity of the repair (Figure I-30). Mitral repair is preferred to replacement because repair is less disruptive to left ventricular filling and it preserves the complex geometric and functional relationship between the valve and the ventricle. Valvular repair can often be performed via a smaller lateral thoracotomy incision or with robotic assistance, making the surgery much less invasive. When repair is not feasible, cardiac valves may be replaced with a valvular prosthesis. Prosthetic valves can be mechanical or bioprosthetic; the latter are composed of animal tissue or in some cases donated human tissue. Mechanical prostheses have the advantage of durability but the disadvantage of necessitating lifelong anticoagulation to prevent valve thrombosis. Bioprosthetic valves do not require anticoagulation, but they may be less durable than mechanical valves; younger patients who receive bioprosthetic valves may require another valve replacement later in life. Cardiac surgery may also help patients with advanced heart failure. Patients with progressive heart failure symptoms despite compliance with appropriate medical therapy may improve with support from a surgically implanted ventricular assist device (VAD). VADs involve the implantation of an inflow cannula at the ventricular apex attached to a continuous pump that directs flow through an outflow cannula placed in the ascending aorta. VAD therapy can be used to support the left or right ventricle, and a VAD may be used as a bridge to transplantation or as a permanent feature of therapy, termed “destination therapy.” Highly motivated patients with advanced pump failure who fail medical and device therapy may be considered candidates for cardiac transplantation.

This page intentionally left blank

SECTIO N II: Cl INICa l Ca SES

SECTION II

Clinical Cases

57

This page intentionally left blank

Ca SE 1 a 62-ye r-o d m n rrives to the emergency dep rtment comp ining o cute, severe precordi chest p in r di ting to his rm nd neck. He reports ee ing ike “ n e eph nt is st nding on my chest” nd st tes th t his symptoms re ccomp nied by n use . His chest p in beg n pproxim te y 30 minutes go whi e he w s w tching te evision nd it h s not comp ete y reso ved since onset. His medic history inc udes hypertension, hyper ipidemi , nd 50-p ck/ ye r history o cig rette smoking. On ex min tion the p tient is di phoretic nd in moder te distress with the o owing vit signs: b ood pressure 156/ 97 mmHg, pu se 113 bpm, respir tory r te 24 bre ths/ min, nd oxygen s tur tion 98% on room ir. He is t chyc rdic with norm S1 nd S2 nd without murmurs, rubs, or g ops. His jugu r venous pressure is not e ev ted; he h s e t c rotid bruit. Chest uscu t tion reve s int cr ck es t the e t b se but is otherwise c e r. His bdomen is protuber nt but so t nd without m sses. His ower extremities re without edem . He h s 2+ pu ses in his upper nd ower extremities. a n e ectroc rdiogr m (ECG) is perormed nd shown (Figure 1-1). St t bs re dr wn nd the nurse h s just sent them to the bor tory. C C C

Wh t is the most ike y di gnosis? Wh t is the most import nt e ture o his presenting history? Wh t is the most import nt initi ther peutic m neuver?

I

a VR

V1

V4

II

a VL

V2

V5

III

a VF

V3

V6

Fig re 1-1. C se ECG. M ke note o the ST segments in e ds V1–V4.

60

Ca SE FIl ES: Ca RDIO l OGY

a NSWER TO Ca SE 1: Anterior ST Elevation Myocardial Infarction (STEMI) Summary: This is a 62-year-old man who presents with a chest pain story that is classic for acute myocardial ischemia, including precordial discomfort radiating to the arm and neck. He has risk factors for coronary artery disease, including elevated cholesterol, high blood pressure, and an extensive smoking history. He has a carotid bruit on exam that suggests significant underlying atherosclerosis. An acute surge of catecholamines is responsible for the patient’s tachycardia, elevated blood pressure, and diaphoresis. His ECG is diagnostic. • M

l

l

ag

• M

m

a

• M

m

a

: Anterior ST segment elevation myocardial infarction. a

g al

a

ma

: The time since symptom onset. v : Prompt coronary revascularization.

ANALYSIS Objectives 1. Instant recognition of patients presenting with STEMI. 2. Understanding the importance of immediate reperfusion therapy for patients with STEMI. 3. Awareness of other diagnoses that should be considered in this patient group. 4. Knowledge of the potential electrical and mechanical complications of STEMI. 5. Knowledge of the evidence-based therapies that should be employed on discharge for this patient group.

Considerations ST elevation myocardial infarction is a true medical emergency that requires immediate recognition and prompt treatment. Time is the most important factor to consider at presentation because survival of myocardial tissue (as well as the patient) depends on prompt and early coronary revascularization. “Time is muscle” is a commonly used expression in emergency departments and catheterization laboratories around the world for good reason; the faster we recognize and treat STEMI, the more lives we save. The first priority in this patient is to establish the diagnosis so that definitive therapy can be rendered. His ECG and clinical scenario is classic for STEMI; however, it is important to consider other potential causes of ST segment elevation and chest pain such as acute aortic dissection and pericarditis as these conditions will certainly not improve and in fact may be exacerbated by the usual treatment for STEMI [eg, anticoagulation, percutaneous coronary intervention (PCI), or thrombolysis]. These diagnoses can often be excluded by clinical history alone, but if aortic dissection is strongly suspected, it may be necessary to arrange for a stat contrast-enhanced

SECTIO N II: Cl INICa l Ca SES

61

CT of the chest or transesophageal echocardiogram prior to definitive therapy. It is also important to carefully document this patient’s presenting physical exam so that potential complications of myocardial infarction can be readily identified should he decompensate during his hospitalization. Once the diagnosis of STEMI is secure, the patient should receive aspirin 325 mg, an ADP inhibitor, and parenteral anticoagulation while in the emergency department a decision is made regarding reperfusion strategies.

a PPROa CH TO: ST Elevation Myocardial Infarction DEFINITIONS Ac u t e c o r o n Ar y s y n d r o Me (Ac s ): A term that encompasses a range of ischemic heart diseases, including unstable angina (pain from myocardial ischemia at rest), non-ST-segment elevation myocardial infarction (myocardial ischemia with ST depression and/or T wave inversion and positive biomarkers), STEMI (myocardial ischemia due to complete occlusion of affected coronary artery resulting in positive biomarkers and ST elevation of > 0.1 mV in two or more adjacent leads). Ad e n o s i n e d i ph o s ph At e (Ad p) r e c e pt o r i n h i b i t o r : Any of the antiplatelet medications used to inhibit platelet activation and aggregation by blocking the ADP receptor on platelet cell membranes. The most widely used ADP inhibitors include clopidogrel, prasugrel, and ticagrelor. c Ar d i Ac b i o MAr k e r s : Any serum marker that is specific for necrosis of myocardial tissue such as troponin T, troponin I, and CK-MB (creatine kinase myocardial band). When these markers are positive in the appropriate clinical setting, a patient has “ruled in” for myocardial infarction. r e pe r f u s i o n t h e r Apy: The ultimate treatment goal in the management of acute myocardial infarction is to return adequate blood flow to the affected myocardium by establishing patency of the infarct-related coronary artery. r a refers to either direct mechanical disruption of the thrombus with percutaneous coronary intervention or through pharmacologic therapy with potent fibrinolytic medications that lyse thrombus such as streptokinase, alteplase, reteplase, and tenecteplase.

CLINICAL APPROACH Pathophysiology An ST segment elevation myocardial infarction (STEMI) is most commonly the result of atherosclerotic plaque rupture with subsequent acute thrombus formation and completion occlusion of the arterial lumen. Rupture of the fibrous cap reveals the highly thrombogenic extracellular lipid core, initiating platelet activation and aggregation as well as thrombin activation (Figure 1-2). Less frequently, an erosion

62

Ca SE FIl ES: Ca RDIO l OGY

Plate le ts

Ac tivate d plate le ts

Fibrino g e n

Fig re 1-2. P thophysio ogy o STEMI. a cute coron ry syndromes typic y begin with the rupture o vu ner b e therosc erotic p que ( ). Inter ction between the b oodstre m nd p que contents (p rticu r y oxidized l Dl cho estero ) resu ts in p te et ctiv tion nd ggreg tion (b). Fibrinogen is c e ved by thrombin to ibrin, which then comp exes with ctiv ted p te ets to orm thrombus (c) th t m y u tim te y prop g te nd occ ude the vesse umen, c using in rction (d). (Reprinted with permission, Cleveland Clinic Center for Medical Art & Photography © 2013. All rights reserved.)

of a coronary plaque can occur without plaque rupture but still resulting in thrombus generation. Other rare causes of STEMI include coronary artery spasm, coronary artery dissection, arteritis, cocaine abuse, and embolism to a coronary artery. All of these pathologic mechanisms have a common final pathway to inducing myocardial ischemia and ultimately infarction if artery patency is not established within approximately 30 minutes of occlusion. In other words, the lack of blood flow results in myocyte ATP depletion and subsequent coagulation necrosis of the myocardium.

Clinical Presentation The majority of patients presenting with STEMI will describe a crushing, heavy, pressure, or squeezing sensation. Radiation to the left arm or into the jaw is classic, although radiation to the back, right arm, shoulder, and epigastric region are

SECTIO N II: Cl INICa l Ca SES

63

also seen. The character can be similar to stable angina; however, the pain associated with STEMI is of longer duration (typically > 20–30 minutes), not relieved by nitroglycerin, and more intense. Associated symptoms include dyspnea, diaphoresis, palpitations, nausea, vomiting, extreme fatigue, or an impending sense of doom. Importantly, patients with diabetes, women, or the elderly may present without chest pain, only describing a nonspecific discomfort in combination with one of the aforementioned associated symptoms such as dyspnea or diaphoresis. One must maintain a high index of suspicion in these patient populations. The physical exam rarely contributes to the diagnosis of STEMI; however, it is very important to perform in order to exclude other diagnoses that may present in a similar fashion. In addition, the initial exam is important to monitor for complications that may occur as a result of STEMI such as pulmonary edema with pump failure or a new apical systolic murmur in patients with papillary muscle rupture. The most important action to take in any patient presenting with possible STEMI is to obtain an ECG. This should be performed and interpreted immediately on presentation so that emergent reperfusion therapy can be initiated if indicated. ECG criteria diagnostic of STEMI include ST elevation of 1 mm or greater in two or more contiguous leads. In leads V2–V3, at least 1.5 mm of ST elevation in women and 2 mm or greater in men are needed to make the diagnosis. The infarct-related artery can be identified according to which leads are involved (Table 1-1). In addition, any patient with symptoms suggesting ACS in the setting of a new left bundle branch block (LBBB) should be treated as diagnostic for STEMI. In patients with old LBBB, the criteria in Table 1-2 can be helpful in diagnosing STEMI. If the diagnosis remains unclear in patients with LBBB, emergent echocardiography is indicated to explore for regional wall motion abnormalities that may suggest underlying active myocardial injury. Wall motion assessment by echocardiography is also helpful in cases where ECG findings are not characteristic for STEMI although there is a high index of suspicion based on clinical presentation such as ongoing typical chest pain unrelieved with nitroglycerin administration. Complete occlusion of the left circumflex artery (LCx), for example, will occasionally present with minimal to no ECG changes. Echocardiography can reveal akinesis of the lateral wall, suggestive of STEMI and the need for emergent reperfusion therapy.

Table 1-1 • ECG CHANGES AND CORRELATION WITH INFARCT-RELATED ARTERY Category

Infarct-Related Coronary Artery

ECG Characteristics

a nterior

l e t nterior descending (l a D)

ST e ev tion in V1 – V4

l ter

l e t circum ex (l Cx)

ST e ev tion in V5 – V6 , I, Vl

In erior

Right coron ry rtery (RCa ) (80%) or l Cx (20%)

ST e ev tion in II, III, VF

Posterior

RCa or l Cx

T R w ve in V1–V3 , with ST depression in V1–V3

64

Ca SE FIl ES: Ca RDIO l OGY

Table 1-2 • ECG CRITERIA FOR DIAGNOSING STEMI IN PATIENTS WITH UNDERLYING LEFT BUNCLE BRANCH BLOCK (LBBB) Criterion

Score

ST segment e ev tion ≥ 1 mm concord nt with QRS

5

ST segment depression ≥ 1 mm in e ds V1, V2, or V3

3

ST segment e ev tion ≥ 5 mm discord nt with QRS

2

Note: ≥ 3 points = 90% specif city nd

88% positive predictive v ue.

Laboratory investigation is also important for the diagnosis of STEMI; however, there may be a significant delay between the time of vessel occlusion and the return of the first positive biomarker assay. Thus, one should not wait for laboratory data before making a decision regarding reperfusion therapy in patients with symptoms and ECG findings diagnostic for STEMI. Contemporary cardiac biomarkers include creatine kinase (CK), creatine kinase myocardial band (CK-MB), troponin I (TnI), and troponin T (TnT). CK is present throughout skeletal tissue, whereas CK-MB is more specific to cardiac muscle. Both are released during STEMI and are measureable within 4–6 hours, peak within the first 24 hours, and normalize by 48–72 hours. TnI and TnT are both specific and sensitive for acute myocardial infarction. They are released within 4–8 hours after symptom onset and generally peak by 12–36 hours. Troponins can remain elevated for up to a week after an acute event.

Differential Diagnosis The differential diagnosis for STEMI is extensive and includes other cardiovascular disorders, pulmonary pathology, and gastrointestinal (GI) disease. Perhaps the most important diagnosis to rule out is the presence of an aortic dissection. This is critical as fibrinolytics and anticoagulants are contraindicated in aortic dissection. One should suspect dissection in a patient with risk factors for dissection, including a history of uncontrolled blood pressure or Marfan’s disease. Classically, the patient will report sudden onset of tearing chest pain that radiates to the back. A chest x-ray can reveal a widened mediastinum, and the patient may have unequal blood pressure readings in each arm. Careful auscultation may reveal the presence of a new diastolic murmur consistent with aortic insufficiency. All of these findings, however, are nonspecific, and thus if there is any suspicion of aortic dissection, a gated CT is required for the diagnosis. Alternatively, if the patient has renal insufficiency that would be exacerbated by contrast administration, an emergent transesophageal echocardiogram (TEE) can be obtained. Occasionally, a dissection can be identified by transthoracic echocardiography (TTE) alone; however, if this is unrevealing, further imaging with CT or TEE is required. It is important to note that a patient can present with both aortic dissection and myocardial infarction should the dissection flap involve a coronary ostium. Emergent cardiothoracic surgery is required for definitive treatment in any dissection involving the ascending aorta. Patients with pericarditis classically present with chest pain that is improved with sitting forward or upright and exacerbated when supine. ECG findings usually

SECTIO N II: Cl INICa l Ca SES

65

show diffuse concave ST elevations with PR depression. Auscultation may reveal a friction rub. If the inflammation also involves myocardium, the patient will often have positive cardiac biomarkers. It can be associated with a viral infection and generally has a more subacute presentation compared to ACS. Another important diagnosis on the differential for sudden-onset chest pain is pulmonary embolism. This typically presents with acute onset of dyspnea and pleuritic chest pain. ECG will most often show sinus tachycardia, although it may reveal acute cor pulmonale with a S1Q3T3 pattern (prominent S wave in lead I along with a Q wave and inverted T wave in lead III). Echocardiography may reveal evidence of right ventricular strain and is helpful in ruling out regional wall motion abnormalities of the left ventricle. Gastrointestinal (GI) disorders, including nutcracker esophagus, diffuse esophageal spasm, GI reflux disease, and acute cholecystitis, can also mimic ACS. A careful history and physical exam (H&P) helps to exclude these potential diagnoses. Chest pain that is affected by food intake, is relieved by antacids, or does not radiate is more suggestive of noncardiac chest pain. Acute cholecystitis is generally associated with right upper quadrant tenderness, leukocytosis (which can also be seen in ACS), and fever. Diagnosis can usually be established with abdominal ultrasound. Given the life-threatening nature of ACS, one should fully rule out cardiac chest pain prior to attributing symptoms to a GI source.

TREATMENT OF STEMI After diagnosis of STEMI based on the above, all patients should be administered aspirin 325 mg unless a true aspirin allergy exists. Each patient should also receive an oral loading dose of an ADP inhibitor such as clopidogrel 600 mg, prasugrel 60 mg, or ticagrelor 180 mg. If the patient is selected for fibrinolytic therapy, then administration of clopidogrel 300 mg is recommended. All patients should also receive a parenteral anticoagulant. Traditionally, unfractionated heparin has been used. For those undergoing fibrinolytic therapy, enoxaparin has been shown to be superior to unfractionated heparin. Glycoprotein IIb/IIIa inhibitors should not be administered at first presentation in patients who have received dual antiplatelet therapy but generally are reserved for use by an interventional cardiologist if the patient is experiencing thrombotic complications. While the aforementioned therapies are important in the treatment of STEMI, the most important clinical decision in STEMI management pertains to the patient’s candidacy for reperfusion therapy. Time is of the essence to prevent irreversible myocardial damage as there is an inverse relationship between time to reperfusion and survival benefit. There are no absolute contraindications to PCI, and it is the recommended method of reperfusion when it can be performed in a timely manner. Specifically, the goal is to limit the “door to balloon” time at a PCI-capable facility to under 90 minutes; that is, angioplasty should be performed less than 90 minutes from the time the patient first presents for treatment. If the patient arrives to a nonPCI capable facility, an extra 30 minutes is permitted (for a total of 120 minutes) to allow time for transfer. If facilities for PCI are not available in a timely manner, then reperfusion with a fibrinolytic, unless contraindicated, should be employed within 30 minutes of

66

Ca SE FIl ES: Ca RDIO l OGY

Table 1-3 • ABSOLUTE AND RELATIVE CONTRAINDICATIONS TO FIBRINOLYTIC THERAPY Absol te Contraindications

Relative Contraindications

a ny prior ICH Known structur br in esion Known intr ncr ni neop sm Ischemic stroke within 3 months Suspected ortic dissection a ctive b eeding (not menses) C osed he d tr um within 3 months

History o chronic, severe hypertension SBP > 180 or DBP > 110 mmHg Tr um tic or pro onged CPR (> 10 min) Ischemic stroke > 3 months go Recent intern b eeding (within 4 weeks) Noncompressib e v scu r punctures Pregn ncy a ctive peptic u cer a ctive ntico gu tion

Abbreviations: CPR, c rdiopu mon ry resuscit tion; DBP, di sto ic b ood pressure; ICH, intr cr ni hemorrh ge; SBP, systo ic b ood pressure.

hospital arrival. Absolute contraindications to fibrinolysis include suspected aortic dissection, active bleeding or diathesis (not including menses), history of hemorrhagic stroke, intracranial neoplasm or cerebral vascular lesion, or a history of ischemic stroke, closed head, or facial trauma within the last 3 months (Table 1-3). After administration of a fibrinolytic agent, all patients, especially those who are at high risk (extensive ST segment elevation, history of myocardial infarction, recentonset LBBB, tachycardia, or hypotension), should be transferred to a PCI-capable facility as soon as possible so that PCI can be performed as needed. After these initial therapies, the patient should be monitored in a coronary intensive care unit so that any potential complications can be quickly identified and treated.

COMPLICATIONS OF STEMI Acute complications of STEMI include arrhythmias, pump failure, papillary muscle rupture or dysfunction, ventricular septal rupture, ventricular free wall rupture, or embolic complications. Late complications include postinfarct pericarditis (Dressler syndrome) and ventricular aneurysm formation. Within the first 24–48 hours of STEMI, patients are at increased risk of ventricular tachycardia (VT) or ventricular fibrillation (VF), particularly for those who are not revascularized in a timely manner or are incompletely revascularized. Patients with sustained VT or VF should be treated immediately with electrical defibrillation or cardioversion as well as intravenous amiodarone. Although premature ventricular complexes (PVCs) are frequently seen in these patients, prior studies have shown increased mortality when antiarrhythmics are used for PVC suppression; therefore, these agents should be avoided. Any electrolyte derangement such as hypokalemia or hypomagnesemia should be treated. One should not confuse VT with an accelerated idioventricular rhythm, which is a wide complex rhythm at a heart rate of 60–120 bpm that frequently accompanies myocardial reperfusion (Figure 1-3). Accelerated idioventricular rhythm is benign and almost always asymptomatic. Supraventricular tachycardias (SVTs) are less common, although they should be treated to prevent increased myocardial oxygen demand. If the SVT causes hemodynamic compromise, immediate

SECTIO N II: Cl INICa l Ca SES

67

II

Fig re 1-3. a cce er ted idioventricu r rhythm. Common y re erred to s reperfusion arrhythmia, this tr nsient wide comp ex rhythm is typic y symptom tic nd does not c use hemodyn mic inst bi ity.

cardioversion is indicated followed by amiodarone administration. Sinus bradycardia may be seen in patients who suffer an inferior STEMI as the RCA typically gives rise to the sinoatrial nodal artery that supplies the sinoatrial node. If this rhythm is not well tolerated, a temporary transvenous pacemaker may need to be inserted. Likewise, in patients who develop high-degree AV block, a temporary pacemaker may be required. Ideally, both atrial and ventricular leads are inserted to maintain synchrony between the atria and ventricles. Pump failure and cardiogenic shock occur in 5–8% of patients hospitalized with STEMI. c a g is defined as a systolic blood pressure (SBP) 90 mmHg, cardiac index < 2.2 L/min/m2, and a pulmonary capillary wedge pressure (PCWP) of >15 mmHg. In patients with extensive myocardial involvement, systolic function may be profoundly compromised with decreased cardiac output. Diastolic function can also be impaired, resulting in increased filling pressures, pulmonary congestion, and hypoxemia. The mainstay of treatment for cardiogenic shock is early revascularization. After this has been performed, care is mainly supportive and focuses on maintaining adequate ventilation, oxygenation, and perfusion. Therapy may include an intraaortic balloon pump, temporary pacing if bradycardic, as well as vasoactive agents. Hemodynamic monitoring with a pulmonary artery catheter is frequently employed. In particularly dire cases in the appropriate patient, a percutaneous ventricular assist device (eg, TandemHeart or Impella) or extracorporeal membrane oxygenation (ECMO) may be employed as a bridge to recovery, cardiac transplant, or left ventricular assist device (LVAD) implantation. Patients with inferior STEMI may experience right ventricular (RV) infarction and failure. These patients will typically present with hypotension and elevated jugular venous pressures, but with no pulmonary congestion. They are extremely volume sensitive and may require liters of fluid to maintain cardiac output and maintain a PCWP of 15 mmHg. Given their dependence on preload, nitrates and diuretics must be avoided. This cannot be overstated. Should IV fluids alone not maintain cardiac output, an inotrope such as dobutamine is indicated. In the majority of cases, RV function will improve with time and supportive care. Patients suffering from an inferior STEMI are also at increased risk for papillary muscle dysfunction or rupture. This is due to the fact that the posteromedial papillary muscle generally is supplied by a single artery, namely, the posterior descending artery (PDA), which typically arises from the RCA. Severe papillary muscle dysfunction or, in particular, papillary muscle rupture, is an indication for emergent surgery. In the case of rupture, rapid initiation of after load reduction with vasoactive medications as well as intraaortic balloon pump (IABP) is virtually always required as a bridge to surgery.

68

Ca SE FIl ES: Ca RDIO l OGY

Ventricular septal rupture (VSR) is more common in patients who are older, are female, and have no history of prior infarct. Frequently, it manifests as a sudden deterioration in clinical status with hypotension, biventricular failure, and a new pansystolic murmur at the left lower sternal border. Importantly, in patients with a large defect or those that are in cardiogenic shock, the murmur may be subtle or even inaudible. The patient should be stabilized with expeditious initiation of nitroprusside and IABP to reduce afterload. Emergent consultation with cardiothoracic surgery is indicated. Even with surgery, prognosis is still grim. In patients who are at particularly high risk, percutaneous closure may be performed by an interventional cardiologist using a ventricular septal defect closure device. Rupture of the ventricular free wall is also more common in females, the elderly, and those who have no prior history of myocardial infarction. It generally presents with sudden onset of chest pain and is rapidly fatal unless emergent surgery is performed. As blood enters the pericardial space, the patient will develop signs of tamponade, including jugular venous distension, muffled heart sounds, and pulsus paradoxus. In dire cases, pericardiocentesis can be considered while arrangements are being made for the operating room. This is a stopgap measure only as blood will continue to fill the pericardial space; definitive surgical therapy is uniformly required for survival. In addition to the above, one must be watchful for potential embolic complications. Shortly after the patient develops an acute occlusive coronary lesion, the patient’s ventricular function is impaired along the artery’s territory. Blood flow becomes sluggish, and the risk of a mural thrombus increases. This is particularly true in patients who suffer an anterior wall myocardial infarction or in those with large infarcts. Should this thrombus embolize, the patient may suffer stroke, acute limb ischemia, intestinal ischemia, or renal infarction. In the acute setting, the appropriate consultant should be contacted immediately to prevent permanent tissue damage. Intravenous anticoagulants followed by oral anticoagulants on an outpatient basis should also be employed. Dressler’s syndrome or late pericarditis is a late complication of STEMI that can present up to 2 months postinfarct. Patients typically describe progressively worsening chest pain that can last for hours. The discomfort is exacerbated with the patient lying flat and alleviated when the patient leans forward. A friction rub may be heard on exam. Treatment is with aspirin 650 mg every 4–6 hours along with colchicine. Nonsteroidal anti-inflammatory medications (NSAIDs) should be avoided in the postinfarct patient given their increased cardiovascular risk. Ventricular aneurysms are another potential late complication of STEMI. They are more commonly seen in patients who do not receive reperfusion therapy. Symptoms vary widely, and patients may be asymptomatic or present with arrhythmias and acute decompensate heart failure. If a mural thrombus is present, patients should be anticoagulated for 3–6 months. Anticoagulation should also be considered in patients with an aneurysm in combination with a low ejection fraction, as these patients have higher rates of stroke. Surgical resection should be considered in patients who develop refractory heart failure or arrhythmias.

SECTIO N II: Cl INICa l Ca SES

69

SECONDARY PREVENTION Because of the extensive toxic cardiovascular effects of smoking, any patient who smokes should stop immediately. For many patients, experiencing a STEMI is a profoundly life-altering experience that renders them particularly amenable to smoking cessation efforts. This is, in fact, the single most important action a patient can take to prevent subsequent morbidity and mortality and should be pursued aggressively. Other risk factors, including hypertension and diabetes, should be optimized. Prior to discharge, all patients should leave the hospital on an evidence-based medical regimen to help prevent recurrent events and death. High-dose statins, such as atorvastatin 80 mg daily, have been shown to be more effective than low-dose statins, even if their LDL (low-density lipoprotein) cholesterol is already low as statins have beneficial pleiotropic effects aside from lowering cholesterol. All patients with a history of ACS should have a lifelong LDL goal of 20 minutes). Pain from U A/N STEMI is more severe and of longer duration when compared to stable angina. NSTEMI is differentiated from UA by the presence of positive cardiac biomarkers. In general, patients with UA/NSTEMI are older and have more cardiac risk factors such as hypertension, hyperlipidemia, or diabetes when compared to patients with ST elevation myocardial infarction (STEMI). They are also more likely to have a prior history of myocardial infarction or to have undergone revascularization with PCI or coronary artery bypass grafting (CABG). Associated symptoms include dyspnea, diaphoresis, palpitations, nausea, or vomiting. Diabetic patients, females, or the elderly may present without chest pain, describing only a nonspecific discomfort in combination with one of the aforementioned associated symptoms such as dyspnea or diaphoresis. The physical exam seldom adds to the diagnosis of UA/NSTEMI. Patients who present with signs of heart failure on exam (elevated jugular venous pressure, S3 gallop, or rales) are at increased risk for in-hospital morbidity and mortality. Documentation of the baseline physical exam is very important to recognize potential complications that may arise during their hospital stay, particularly the presence or absence of cardiac murmurs as a new murmur may indicate a mechanical complication related to myocardial infarction. An ECG should be performed within 10 minutes of presentation and is helpful in risk-stratifying patients. Patients with ST segment depression of ≥0.5 mm or transient ST segment elevation as well as those with a preexisting left bundle branch block are at increased risk of death or myocardial infarction within 1 year. Other patients may present with new T wave inversions alone, which are less specific, although new, deep T wave inversions of ≥2 mm across the precordium usually are associated with ischemia, generally due to a severe lesion of the proximal left anterior descending artery (Wellens’ syndrome). Importantly, many patients do not present with any ischemic ECG changes, so lack of ECG changes is insufficient to rule out a diagnosis of UA/NSTEMI. In patients with ongoing chest pain suspicious for acute coronary syndrome (ACS), serial ECGs should be obtained every 15–30 minutes to monitor for the development of ST segment deviation. Laboratory investigation is critical to differentiate between UA and NSTEMI. Clinically, U A and N STEMI are indistinguishable. A diagnosis of NSTEMI is confirmed by the presence of positive cardiac biomarkers. Importantly, there is a time delay between when symptoms first start and the development of positive biomarkers. Contemporary cardiac biomarkers include creatine kinase (CK), creatine kinase myocardial band (CK-MB), troponin I (TnI), and troponin T (TnT). CK is present throughout skeletal tissue, whereas CK-MB is more specific to cardiac muscle. Both are released during NSTEMI and are measureable within 4–6 hours, peak within the first 24 hours, and normalize by 48–72 hours. TnI and TnT are both specific and sensitive for acute myocardial infarction. They are released within 4–8 hours after symptom onset and generally peak by 12–36 hours. Troponins may stay elevated for up to a week after an acute event. Given the aforementioned time delay, patients with negative cardiac biomarkers within 6 hours of symptom onset should have biomarkers remeasured within 8–12 hours after symptom onset. It is

SECTIO N II: CLINICAL CASES

83

also common to trend biomarkers at 6–8-hour intervals 3 times or until levels have peaked to provide a measure of infarct size.

Differential Diagnosis The differential for UA/NSTEMI is similar to STEMI that was discussed in Case 1. However, in this patient population, it is also important to explore for causes of secondary UA/NSTEMI, that is, disease that precipitates myocardial ischemia due to instigating a mismatch between myocardial oxygen supply and demand. For example, patients with an underlying high-grade coronary artery stenosis that is generally asymptomatic may present with anginal chest pain if they become tachycardic with increasing oxygen demand. This can be observed in patients who present with a supraventricular tachycardia or in patients who are septic with relative hypovolemia and compensatory tachycardia. Likewise, a patient with significant acute blood loss with resultant anemia may also develop supply demand mismatch due to hypovolemia with tachycardia as well as decreased oxygen-carrying capacity from loss of red blood cells. Other potential diagnoses that can cause supply demand mismatch include thyrotoxicosis, hypotension, or hypoxemia. In these cases of secondary UA/ NSTEMI, the primary goal is treatment of the underlying disease state. Once the patient has recovered from the acute illness, a search for significant coronary artery disease can be considered.

TREATMENT OF UA/ NSTEMI After diagnosis of UA/NSTEMI with the aforementioned criteria, the initial treatment goals include alleviation of ischemic pain, optimization of the patient’s hemodynamics, cardiac risk stratification, choosing a management strategy, and initiation of antithrombotic therapy. For relief of ischemic pain, sublingual nitroglycerin should be administered, and if pain continues after three tablets, intravenous nitroglycerin can be infused and titrated for pain relief. Intravenous nitroglycerin can also be administered to treat any concomitant hypertension. Likewise, beta-blockers can be used for those with suboptimal hemodynamics, such as hypertension or tachycardia, to decrease myocardial oxygen demand. Beta-blockers are also helpful for those with ongoing chest pain; however, they should be avoided in the acute setting when signs and symptoms of cardiogenic shock or hemodynamic compromise are present. Oxygen can be given to those with hypoxia or respiratory distress. Various risk stratification models have been developed to identify patients who are at high risk and thus may benefit from an early invasive approach with cardiac catheterization. The “thrombolysis in myocardial infarction” (TIMI) risk score (see Table 2-1) is frequently used to help identify patients at high risk. Importantly, some factors in this risk score carry more weight than others. For example, patients with ST segment depression or elevated cardiac biomarkers are considered high risk regardless of the patient’s TIMI score. Another important decision to be made when the patient initially presents is whether to pursue early invasive therapy versus an initially conservative approach. This decision depends on various factors and is summarized in Table 2-2. Patients undergoing an early invasive approach should undergo cardiac catheterization

84

CASE FILES: CAr d IO LOGy

Table 2-2 • SELECTION OF INITIAL INVASIVE VERSUS CONSERVATIVE STRATEGY Invasive

Conservative

r ecu ent angina o ischemia espite optimize me ical the ap

Low isk sco e (eg, TIMI)

Elevate ca iac bioma ke s

Patient o ph sician p efe ence in the absence of high- isk featu es

New ST segment ep ession Signs o s mptoms of hea t failu e Hemo namic instabilit Sustaine vent icula tach ca ia P io PCI within 6 months o histo

of CABG

New o wo sening mit al egu gitation High- isk sco e (eg, TIMI) Left vent icula function < 40% Patients with an of these featu es shoul be efe e fo imme iate co ona angiog aph . [Data from Jneid H, Anderson JL, Wright RS, et al. 2012 ACCF/ AHA focused update incorporated into the ACCF/ AHA 2007 guidelines for the management of patients with unstable angina/ non-ST-elevation myocardial infarction. J Am Coll Cardiol. 2012;61(23):e179–e347.]

within 24 hours of presentation. Patients for whom a conservative strategy is chosen should be optimized medically and undergo catheterization only should they develop recurrent ischemia or symptoms on optimized therapy. Low- and intermediate-risk patients who are initially managed conservatively should also undergo cardiac stress testing. If high-risk features are present on functional testing, cardiac catheterization should be considered. With regard to antithrombotic therapy, all patients should be loaded with aspirin 325 mg. Patients who undergo an initially conservative approach should be loaded with clopidogrel 600 mg or ticagrelor 180 mg (preferred). Anticoagulation with low-molecular-weight heparin or fondaparinux is preferred, although unfractionated heparin can also be employed if needed. Patients treated with fondaparinux have a higher incidence of catheter- associated thrombosis, so if a patient treated with this agent ultimately undergoes catheterization, another anticoagulant should be used. In patients who are at higher risk (recurrent ischemic discomfort, dynamic ECG changes, or hemodynamic instability), a 48–72-hour GP IIb/IIIa (eptifibatide or tirofiban) infusion can be considered. It should be noted that most of these higherrisk patients should then proceed to catheterization. Patients in whom an initial invasive approach is chosen should receive anticoagulation with unfractionated heparin, enoxaparin, or bivalirudin (preferred because of lower bleeding risk). They should also receive ADP inhibition with clopidogrel, ticagrelor, or prasugrel. GP IIb/IIIa inhibitors can be reserved for use during catheterization, although if bivalirudin is used as the anticoagulant, they are not necessary. Finally, it is important to note that there is no role for fibrinolytic agents in patients with UA/NSTEMI.

SECTIO N II: CLINICAL CASES

85

In addition to antiplatelet therapy described previously, a high-dose HMG-CoA (3-hydroxy-3-methylglutaryl-coenzyme A) reductase inhibitor (statin), such as atorvastatin 80 mg daily, should be prescribed on presentation even if lipids are the goal. Beta-blockers should be started as soon as feasible and maintained indefinitely. An angiotensin-converting enzyme inhibitor (ACEI) should also be prescribed indefinitely in all patients with UA/NSTEMI and heart failure, an ejection fraction 10 mmHg, SVO 2 ≥70%, MAP ≥65 mmHg, and urine output >0.5 mL/kg/h. Management includes intravenous crystalloid or colloid volume resuscitation, along with vasopressor in usions such as norepinephrine or dopamine,

100

CASE FILES: CARd IO LOg y

which should be titrated to the lowest possible dose to achieve the target MAP. Although dopamine is commonly used as a vasopressor in patients with shock, there are data supporting improved outcomes with norepinephrine in both distributive and cardiogenic shock. In cases where sepsis is suspected, blood and urine cultures are obtained prior to the use o broad-spectrum antimicrobial agents to cover in ective etiologies. Laboratory data to exclude other causes o distributive shock such as pancreatitis, adrenal ailure, toxin ingestions, and thyroid disease should also be obtained. Management of Hy ovolemic Shock Hypovolemic shock is managed in a ashion similar to distributive shock, namely, goal-directed volume resuscitation (or trans usion in the setting o known or suspected bleeding) and judicious vasopressor support. Addressing the underlying cause or the hypovolemia is also key.

CASE CORRELATION • See also Case 1 (acute coronary syndrome/STEMI) and Case 2 (acute coronary syndrome/NSTEMI).

COMpREHENSION Qu ESTIONS 3.1 A 75-year-old man with a history o hypertension, diabetes, and complete heart block that was treated with a permanent pacemaker several years ago arrives at the ICU rom the orthopedic surgery service or shock management. He is one day removed rom an elective right total hip replacement, and his surgery was reportedly uncomplicated. His preoperative dobutamine echocardiogram revealed normal ventricular unction and no evidence o ischemia. On arrival he is pale and somnolent but arousable. Vital signs: blood pressure (BP) 80/40 mmHg, heart rate (HR) 50 bpm, respiratory rate (RR) 18/min, temperature 37.9°C, and oxygen saturation 96% on room air. On examination his neck veins are lat at 45°. His cardiopulmonary examination is normal. His extremities are cool and without edema. Laboratory rate includes the ollowing: BUN 50, creatinine 1.8 mg/dL, hemoglobin 6.0 g/dL, mean corpuscular volume (MCV) 89, and white blood cells (WBC) 12,000 with normal di erential. Cardiac biomarkers are normal. ECG reveals a paced ventricular rhythm. Two units o blood are ordered, and 2 L o normal saline is in used rapidly. A ter luid resuscitation his examination is unchanged and his blood pressure is now 84/45 mmHg. Which o the ollowing is the most appropriate next step? A. In usion o sodium nitroprusside B. In usion o norepinephrine C. In usion o milrinone D. Pacemaker reprogramming to a heart rate o 90 bpm E. Placement o an intraaortic balloon pump

SECTIO N II: CLINICAL CASES

101

3.2–3.6 Match the hemodynamic pro ile with the most appropriate patient. Each answer (A–E) can be used more than once or none at all. BP (mmHg)

HR (bpm)

RAP (mmHg)

mPAP (mmHg)

PCWP (mmHg)

CI (L/ min/ m2)

SVO2 (%)

SVR (dyn)

A

70/ 30

130

7

15

10

5.5

80

250

B

70/ 55

110

20

40

35

1.5

45

2000

C

70/ 30

140

1

8

5

1.8

55

2000

D

70/ 40

110

15

35

30

1.8

60

400

E

70/ 50

110

20

55

10

1.5

45

2000

3.2 A 55-year-old woman with breast cancer who presents to the emergency department (ED) with dyspnea and syncope ollowing an episode o sharp right-sided chest pain. 3.3 A 19-year-old man with no medical problems who was ound unconscious ollowing a gunshot wound to the abdomen. 3.4 A 77-year-old man with ischemic heart disease and an indwelling Foley catheter admitted rom his nursing home with 3 days o con usion and hypoxemia. 3.5 A 17-year-old woman presenting to the ED with con usion, ever, and abdominal pain attributed initially to menstrual cramps. She has a di use erythrodermic rash. 3.6 A 65-year-old man presents rom the regular nursing loor 3 days a ter undergoing PCI to treat an anterior STEMI. 3.7 A 67-year-old woman with advanced systolic heart ailure is admitted or altered mental status and dyspnea a ter running out o her heart ailure medications one week ago. Vital signs are BP 92/70 mmHg (MAP 77 mmHg), HR 100 bpm, RR 18/min, O 2 saturation 92% on 4 L nasal cannula (NC) oxygen. She has 20 cm JVP at 90°, S3 gallop, wet bibasilar pulmonary rales, and cold lower extremities with 1+ edema. All o the ollowing would be expected to improve her per usion pressure except… A. Intra-aortic balloon pump placement B. Intravenous urosemide C. Intravenous normal saline D. Intravenous milrinone E. Intravenous nitroprusside

102

CASE FILES: CARd IO LOg y

ANSWERS 3.1 D. This patient has evidence or hypovolemic shock that has not responded to initial volume resuscitation. His clinical picture is most consistent with acute blood loss ollowing surgery. We would expect him to mount a compensatory tachycardia, but he is likely pacemaker-dependent and pacing in a programmed backup rate since surgery (device sensing is o ten programmed o or surgeries). Thus, increasing his device rate to 90 bpm would be the most reasonable next step. Pressor support with norepinephrine (B) is not an unreasonable choice, but it would be more appropriate to see i aster pacing solves the problem irst. Dopamine at moderate doses (5–10 µg/kg/min) is not a bad choice in patients with bradycardia as the β1 e ects can increase heart rate, although that may not help in this pacemaker-dependent man. The remaining options are all reasonable or patients with pure cardiogenic shock, but this patient’s pro ile is more consistent with hypovolemic shock that cannot be compensated because o his ixed slow heart rate. Improving the heart rate should eliminate the “cardiogenic component” o his shock. 3.2 E. This patient likely has cardiogenic shock and right ventricular ailure due to acute pulmonary embolism. 3.3 C. This patient has hypovolemic shock due to blood loss rom a gunshot wound. 3.4 D. Mixed distributive and cardiogenic shock triggered by severe urinary tract in ection–related sepsis. 3.5 A. This patient has distributive shock secondary to toxic shock syndrome. 3.6 B. This patient has cardiogenic shock ollowing an acute myocardial in arction. 3.7 C. This patient has examination evidence o lorid central volume overload. Although she is hypotensive, additional luid resuscitation will not help because her preload is suboptimal (overloaded; on the ar right o his Starling curve). Reducing preload with urosemide would actually improve ventricular loading conditions with expected improvement in stroke volume and per usion pressure (remember, ΔP = MAP – RAP). IABP therapy and nitroprusside both improve per usion pressure by reducing a terload and optimizing cardiac output. Milrinone improves per usion pressure by increasing stroke volume due to increased contractility.

SECTIO N II: CLINICAL CASES

103

CLINICAL PEARLS C

Most patients with shock present with sinus tach car ia. If ou see a patient with unexplaine sinus tach car ia an h potension, think of shock. If ou see a patient with shock who has bra car ia or a lowerthan-expecte heart rate, consi er wa s to increase the heart rate.

C

Check arterial bloo ases an lactate levels earl an often in patients with suspecte shock. It will ive ou important pro nostic information an help ou au e response to therap .

C

Exclu e acute m ocar ial infarction imme iatel in patients presentin with car io enic shock; earl revascularization is associate with better outcomes.

REFERENCES De Backer D, Biston P, Devriendt J, et al. SOAP II Investigators. Comparison o dopamine and norepinephrine in the treatment o shock. N Engl J Med. 2010;362(9):779. Dellinger RP, Levy MM, Rhodes A, et al.. Surviving Sepsis Campaign Guidelines Committee including the Pediatric Subgroup. Crit Care Med. 2013;41(2):580. Hochman JS, Sleeper LA, Webb JG, et al. Early revascularization in acute myocardial in arction complicated by acute cardiogenic shock. SHOCK Investigators. Should we emergently revascularize occluded coronaries or cardiogenic shock. N Engl J Med. 1999;341(9):625. Mueller HS, Chatterjee K, Davis KB, et al.. ACC expert consensus document. Present use o bedside right heart catheterization in patients with cardiac disease. American College o Cardiology. J A m Coll Cardiol. 1998;32(3):840. Thiele H, Zeymer U, Neumann FJ, et al. Intraaortic balloon support or myocardial in arction with cardiogenic shock. IABP-SHOCK II Trial Investigators. N Engl J Med. 2012;367(14):1287.

This page intentionally left blank

CASE 4 A 68- ear-o man presents to his new primar care provi er or an initia patient visit. His so e active comp aint is mi retrosterna chest pressure that he experiences on a ter signi icant exertion, an the iscom ort reso ves within minutes o cessation o the provoking activit . The pattern an severit o his s mptoms have not change signi icant over the ast 3 ears, an he enies an episo es o chest pain at rest, izziness, s ncope, spnea, pa pitations, or ower extremit e ema. He has a histor o coronar arter isease requiring percutaneous coronar intervention (PCI) with p acement o a bare-meta stent to the right coronar arter 5 ears ago. He was to that his other coronar vesse s were mi isease an there were no other esions suitab e or PCI. Other pertinent me ica histor inc u es h percho estero emia an ongstan ing h pertension. His current me ications inc u e aspirin 81 mg, metopro o succinate 25 mg ai , simvastatin 10 mg, an h roch orothiazi e 12.5 mg ai . He is a i e ong nonsmoker, an oes not consume a coho or use i icit rugs. On examination the patient is comortab e an in no acute istress. The heart rate is 90 bpm an regu ar, an the b oo pressure is 155/ 85 mmHg in the e t arm. There are no murmurs, rubs, ga ops, or c icks on car iac auscu tation, an the secon heart soun is ph sio ogica sp it. There are no apparent extra heart soun s. The remain er o the ph sica examination is unremarkab e. A resting 12- ea e ectrocar iogram (ECG) is obtaine , an is shown in Figure 4-1. C C C

What is the most ike iagnosis? What is the best next iagnostic step? What is the best next step in therap ?

I

a VR

V1

V4

II

a VL

V2

V5

III

a VF

V3

V6

V1

II

V5 25 mm/s 10 mm/mV 150 Hz 7.1.1

12S L 239 CID: 67

EID: Ne wly a cquire d EDT: ORDE

Figure 4-1. Resting 12- ea ECG. (Reproduced, with permission, from Donald Underwood, MD)

SECTIO N II: Cl INICAl CASES

107

ANSWER TO CASE 4: Chronic Stable Coronary Artery Disease Summary: A 68-year-old man is establishing care with a new primary care provider. He is currently experiencing typical chest pain symptoms with heavy exertion, and these symptoms have been stable or the last 3 years. He denies chest pain at rest, worsening o his angina over the last 3 months, and has no symptoms suggestive o congestive heart ailure. He has a known history o coronary artery disease and is on aspirin, a long-acting beta-blocker, simvastatin, and hydrochlorothiazide. His physical examination is unremarkable, and he does not smoke or drink alcohol. The patient has a normal resting ECG and is able to exercise. Importantly, his blood pressure remains elevated, and his resting heart rate does not suggest therapeutic β-adrenergic blockade. • Most likely diagnosis: Chronic stable coronary artery disease (CAD). • N ext diagnostic step: (1) assess biochemical markers o risk or CAD and MI (total, LDL, and HDL cholesterol, serum creatinine, asting blood glucose, or hemoglobin A1c) and (2) per orm exercise ECG stress testing to evaluate risk or subsequent non atal MI or death rom cardiovascular causes • N ext step in therapy: Optimize guideline-based medical therapy to reach a target blood pressure o 40 mmHg. In symptomatic patients, the peak and mean pressure criteria are each lowered by 10 mmHg (>50 mmHg peak, > 30 mmHg mean). Surgical therapy is recommended in patients with associated severe pulmonary regurgitation, hyoplastic pulmonary annulus, or sub- or supravalvular PS.

Tricuspid Stenosis o v rvi w Tricuspid stenosis (TS) is uncommon, and is typically associated with other valvular lesions, carcinoid syndrome, or rheumatic heart disease. It is infrequently seen in adults. The murmur of and overall presentation of TS is similar to MS—there is an opening snap followed by a low-pitched diastolic murmur, but unlike MS, it is heard at the lower left sternal border and increases with inspiration. Obstruction of flow across the tricuspid valve leads to an increase in RA pressure, and jugular venous distention with a prominent A wave and slow y descent. The predominant symptoms of TS are right-sided heart failure. Like the other valvular lesions discussed, the TTE is utilized for diagnosis. A tricuspid valve area of < 1.0 cm2 is indicative of severe TS. Similar to MS, most cases of TS can be treated with balloon valvuloplasty, with surgery reserved for cases refractory to the percutaneous approach.

CASe Co RReLAt Io N • See also Case 8 (hypertrophic obstructive cardiomyopathy).

Co MPReHeNSIo N QUeSt Io NS 10.1 A 78-year-old man presents with occasional syncope with exertion for the last 2 months. Physical exam demonstrates a III/VI late-peaking systolic murmur at the right upper sternal border. The second heart sound is dampened, and the carotid upstroke is delayed. On the basis of the examination, what is the most likely diagnosis? A. Aortic sclerosis B. Mild to moderate aortic stenosis C. Severe aortic stenosis D. Hypertrophic obstructive cardiomyopathy E. Severe mitral regurgitation

174

CASE FILES: CARd IO LOg y

10.2 An 80-year-old woman with a history of coronary artery disease (treated with three-vessel CABG), severe COPD [forced expiratory volume in 1 second (FEV1) 0.7 L], chronic kidney disease, and type 2 diabetes presents for evaluation of critical AS. She is presently hemodynamically stable but describes progressive NYHA class 3 dyspnea that limits her active lifestyle. Her echocardiogram shows that her AVA is 0.5 cm2 with peak and mean transaortic pressure gradients of 84 mmHg and 56 mmHg, respectively. Which of the following is the best treatment option for this patient? A. Medical management with beta-blockers and afterload reduction B. Percutaneous balloon valvuloplasty C. Transcatheter aortic valve replacement (TAVR) D. Redo sternotomy and open aortic valve replacement E. Referral to a hospice 10.3 A 24-year-old woman is admitted to the hospital with acute pulmonary edema and atrial fibrillation with rapid ventricular rate. Examination reveals an early diastolic murmur most audible at the apex. What is the most likely diagnosis? A. Aortic regurgitation B. Congenital aortic stenosis C. Mitral regurgitation D. Mitral stenosis 10.4 A 36-year-old female is admitted to the hospital with flushing, abdominal pain, and diarrhea for the past week. A CT of the abdomen shows diffuse abdominal lymphadadenopathy, a 2-cm mass in the cecum, and a 1-cm liver mass of uncertain etiology. Physical examination reveals BP 90/54, HR 112 and regular, a II/VI systolic murmur at the left upper sternal border, and right upper quadrant abdominal pain. What is the most likely diagnosis? A. Hepatocellular carcinoma B. Carcinoid syndrome C. Intestinal lymphoma D. Gastrinoma

ANSWeRS 10.1 C. The examination findings are consistent with severe AS. As the stenosis worsens, the murmur becomes late-peaking, obliterating S2, and there may be pulsus parvus et tardus. None of the other valvular abnormalities are described by this patient’s murmur.

SECTIO N II: CLINICAL CASES

175

10.2 C. TAVR has emerged as the treatment of choice for patients with severe AS who are at high risk for complications related to open-valve replacement. Medical therapy does not improve survival or improve symptoms much in patients with severe AS. Balloon valvuloplasty does not provide complete or durable hemodynamic improvement and is generally used as a palliative procedure or as a “bail out” procedure for critically ill patients who are expected to recover enough to potentially undergo TAVR or open-valve replacement. 10.3 D. This question describes the classic clinical scenario of MS exacerbated by the physiologic changes of pregnancy, including increased preload and pulse. This patient should be diuresed carefully, and monitored closely throughout pregnancy given the high association of morbidity and mortality with MS and pregnancy. 10.4 B. This patient’s symptoms are most consistent with carcinoid syndrome. The symptoms of carcinoid syndrome are due to an excess secretion of seritonin, which causes bronchospasm, diarrhea, sweating, and flushing. As seritonin is metabolized by the liver, these symptoms do not typically present unless carcinoid is metastatic to the liver. Carcinoid is associated with right-sided valvular heart lesions, including pulmonary stenosis, as in this patient.

CLINCIAL Pe ARLS C

Aortic stenosis is classifie as severe once AVA < 1.0 cm 2, mean ra ient > 40 mmH , an peak ra ient > 60 mmH . Mortalit from AS t picall occurs within 5 ears of the onset of an ina, 3 ears of s ncope, an 2 ears of heart failure s mptoms.

C

The TAVR proce ure has been shown to be superior to me ical therap in inoperable patients an noninferior to sur er in hi h-risk patients with severe AS.

C

Percutaneous balloon valvuloplast ma provi e temporar relief, but results are not urable in AS, with avera e uration of proce ural success approximatel 3–6 months. In contrast, valvuloplast is the treatment of choice for MS, as results are t picall much more successful an urable.

C

Rheumatic fever is the major cause of MS, an ma be associate with AS an ri ht-si e valvular stenoses, as well.

C

Carcinoi is the most common cause of ri ht-si e valvular stenoses in a ults, an t picall is not s mptomatic unless metastatic to the liver.

176

CASE FILES: CARd IO LOg y

Ref eReNCeS Baumgartner H, et al. Echocardiographic assessment of valve stenosis: EAE/ASE recommendations for clinical practice. Eur J Echocardiogr. 2009;10:1–25. Leon MB, et al. Transcatheter aortic-valve implantation for aortic stenosis in patients who cannot undergo surgery. N Engl J Med. 2010;363:1597–1607. Smith CR, et al. Transcatheter versus surgical aortic-valve replacement in high-risk patients. N Engl J Med. 2011;2187–2198.

CAs e 11 A 54- a -ol man com to th m nc oom with a 3- a hi to of palpitation , fati u , an ho tn of b ath. H ha ha imila mptom in th pa t, but th w alwa ho t-liv an h i not p viou l k m ical att ntion. H ni ch t pain o ncop . Hi m ical hi to i notabl fo ob it , t at h p t n ion, an ob t uctiv l p apn a. Hi ocial hi to v al pi o ic h av alcohol con umption but no illicit u u . Famil hi to i un ma kabl . On ph ical xamination, th pati nt i comfo tabl . Hi pul i i ula l i ula with a h a t at of 110 bpm an hi bloo p u i 126/ 87 mmH . Th i no ju ula v nou i t n ion. Ca iac xamination i no mal apa t f om th i ula h thm. Th lun fi l a cl a , an th i no p iph al ma. C C C

What i th mo t lik l ia no i ? What i th b t n xt ia no tic t p? What i th b t n xt t p in th ap ?

178

CAs e FILe s : CAr d IO LOg y

ANs Wer TO CAs e 11: Atrial Fibrillation Summary: A 54-year-old man with obesity, hypertension, obstructive sleep apnea, and excessive alcohol consumption comes to the emergency room with a 3-day history of palpitations, fatigue, and shortness of breath. He has had similar symptoms in the past, but these were always short-lived and he did not previously seek medical attention. On physical examination, his heart rate is 110 bpm irregularly irregular, and blood pressure is 126/87 mmHg. Cardiopulmonary examination is normal apart from the irregular rhythm. • Most likely diagnosis: Paroxysmal atrial fibrillation. • N ext diagnostic step: Electrocardiogram as shown in Figure 11-1. • N ext step in therapy: Anticoagulation and rate control.

ANALYSIS Objectives 1. Know the diagnostic approach to supraventricular tachycardia. 2. Describe the difference between the mechanisms of atrial fibrillation and atrial flutter. 3. Understand the importance of anticoagulation in atrial fibrillation and atrial flutter. 4. Be familiar with the role of rate and rhythm control strategies in the management of atrial fibrillation.

Considerations This 54-year-old patient has several risk factors for the development of atrial fibrillation. Hypertension, obesity, sleep apnea, and heavy alcohol consumption–especially binge drinking–are all associated with a greater risk for atrial fibrillation. Thus, while other supraventricular tachycardias are possible, atrial fibrillation or atrial flutter is the most likely diagnosis. The irregularly irregular pulse is also very suggestive. An ECG should confirm the diagnosis and differentiate between atrial fibrillation and atrial flutter (Figures 11-1 and 11-2). The first priority should be prevention of thromboembolic stroke, which is perhaps the direct consequence of atrial fibrillation or flutter. For most patients and for patients in whom an early rhythm control strategy is planned, stroke prevention requires systemic anticoagulation with warfarin or a novel oral anticoagulant (NOAC). Control of the ventricular rate is also important to limit symptoms and to avoid the deleterious effects of persistent tachycardia on left ventricular function. Rate control is generally achieved with negatively chronotropic drugs such as beta-blockers and nondihydropyridine calcium channel blockers such as diltiazem. Acute cardioversion of atrial fibrillation

I

a VR

V1

V4

II

a VL

V2

V5

III

a VF

V3

V6

V1

II

V5 Fi re 11-1. At ial fib illation. Not th ab nc of P wav v nt icula at .

an th fin “f wav ” which

p

nt fib illato

activation of th at ia, alon with th i

ula l i

ula

I

a VR

V1

V4

II

a VL

V2

V5

III

a VF

V3

V6

V1

II

V5 Fi re 11-2. At ial flutt . Not th ula , o aniz at ial activit , in cont a t to at ial fib illation. Th flutt wav an a con i t nt th ou hout th h thm t ip. In thi ca , th v nt icula pon i va iabl with a ula l i compl x i a multipl of th at ial flutt c cl l n th.

a b t n in th inf io l a (II, III, an aVF) ula patt n wh th int val b tw n ach Qr s

s e CTIO N II: CLINICAL CAs e s

181

or atrial flutter episodes with a definite onset within the last 48 hours can be considered, although in this case the onset of symptoms is outside this timeframe. If spontaneous conversion to sinus rhythm does not occur, cardioversion can be undertaken after either at least 3 weeks of therapeutic anticoagulation or a transesophageal echocardiogram demonstrating the absence of left atrial appendage thrombus. Underlying potentially reversible causes of atrial fibrillation such as hyperthyroidism should be excluded, and an echocardiogram to assess for structural heart disease– especially mitral valve disease and left ventricular dysfunction–should be ordered. Patients with symptoms or signs suggestive of coronary artery disease may require noninvasive testing to exclude active ischemia.

APPr OACH TO: Atrial Fibrillation and Atrial Fl tter DEFINITIONS ATRIAL FIBRILLATION : A chaotic atrial rhythm characterized by very rapid and uncoordinated depolarization (fibrillation) of the atria with resultant loss of atrial mechanical contraction and an irregular, often rapid, ventricular rate. ATRIAL FLU TTER: A macroreentrant atrial tachycardia around structural or functional barriers in the atria and characterized by rapid atrial contraction at 250–300 bpm with ventricular response that is often fast and regular because of 2:1 conduction over the AV node. PAROXYSMAL: Episodes of atrial fibrillation lasting 7 days, typically until electrical or pharmacologic cardioversion. PERMAN EN T: Continuous atrial fibrillation present for >1 year, or when a decision has been made not to attempt rhythm control. LON E ATRIAL FIBRILLATION : Atrial fibrillation in patients younger than 60 years and without cardiopulmonary disease. RATE CON TROL: A strategy to alleviate the symptoms of atrial fibrillation by controlling the ventricular rate with medications and occasionally ablation of the AV node along with pacemaker implant, without attempts to convert the rhythm to sinus rhythm. RHYTHM CON TROL: A strategy to reestablish and maintain sinus rhythm through the use of antiarrhythmic drugs, cardioversion, and catheter or surgical ablation.

182

CAs e FILe s : CAr d IO LOg y

CLINICAL APPROACH Etiologies Atrial fibrillation and flutter do not have a single unifying cause. Instead, a number of predisposing conditions are described, especially for atrial fibrillation (Table 11-1). Paroxysmal atrial fibrillation is assumed to arise from rapid repetitive atrial depolarizations originating near the pulmonary vein ostia, while persistent atrial fibrillation requires additional abnormalities, especially in the left atrium, for its maintenance. Atrial flutter is less common but also occurs in conditions that cause atrial dilatation, such as pulmonary embolus, mitral or tricuspid valve disease, and congestive heart failure. Typical atrial flutter results from macroreentry around the tricuspid valve and variable amounts of the right atrium. Atypical atrial flutter occurs usually after cardiac surgery or ablation, and is due to reentry around atrial incisions or scars.

Table 11-1 • CONDITIONS ASSOCIATED WITH AN INCREASED RISK OF ATRIAL FIBRILLATION CONg ENITAL

Familial Con

nital h a t i a

Wolff-Pa kin on-Whit DEg ENERATIvE

A vancin a s ick inu

INCREASED ATRIAL STRETCH

n om

H p t n ion Pulmona Con

mbolu

tiv h a t failu

Valvula h a t i a ISCHEmIA AND HYPOxIA

( p ciall mit al an t icu pi )

M oca ial infa ction Co ona

at

Pulmona INFLAmmATORY

n om

i a i a

P ica iti Po top ativ ( p ciall ca iac an tho acic u Inf ction

ENDOCRINE

H p th oi i m d iab t

m llitu

Ph och omoc toma Ob it NEu ROLOg IC

s uba achnoi h mo ha s t ok

DRu g S

Alcohol Th oph llin

)

s e CTIO N II: CLINICAL CAs e s

183

Clinical Presentation The most common symptoms of atrial fibrillation are fatigue, reduced exercise tolerance, and palpitations. For younger patients in whom ventricular rates are generally higher, palpitations predominate. In older patients and in those in whom rates are better, controlled fatigue and exercise intolerance, which can be insidious, are more prominent. Patients with coronary artery disease may present with angina due to rate-related ischemia. Those with heart failure often present with acute decompensation due to loss of atrioventricular synchrony and tachycardia. Most patients have some symptoms that may be manifest as an improvement in exercise capacity after restoration of sinus rhythm, but some are truly asymptomatic. Some patients present with stroke or transient ischemic attack (TIA), or with heart failure due to prolonged tachycardia.

Treatm ent The first line of treatment of atrial fibrillation is anticoagulation to reduce the risk of stroke or systemic embolism. The risk for stroke is approximately 5 times higher in patients with atrial fibrillation than in the general population, but that risk can be reduced by nearly two-thirds with appropriate anticoagulation. Heparin or lowmolecular-weight heparin are used in recent-onset atrial fibrillation in hospitalized patients. The decision to prescribe long-term anticoagulation is based on risk stratification using clinical risk assessment tools such as the CHADS2 (where CHADS is Congestive heart failure, Hypertension, Age ≥75 years, Diabetes mellitus, Stroke) or CHA2DS2-VASc (vascular disease history) score (Table 11-2) and should be weighed against the risk for major bleeding due to anticoagulation using similar risk assessment tools such as HAS-BLED [Hypertension, abnormal kidney or liver function, stroke history, bleeding predisposition, Labile INR, Elderly (age 65 or older),

Table 11-2 • CHADS2 AND CHA2DS2-VASc SCORES CHADS2

CHA2DS2-VASc

Con

Con

tiv h a t failu

tiv h a t failu

H p t n ion

H p t n ion

A

A

≥75

≥75

d iab t

d iab t

s t ok / TIA/ p iph al mboli m

s t ok / TIA/ p iph al mboli m Va cula i a (p io m oca ial infa ction, p iph al at i a , ao tic plaqu ) A

65–74

s x cat

o

(f mal

n

)

Note: Each risk factor is given 1 point except stroke/ TIA/ peripheral em bolism , which is given 2 in both system s, and age ≥75 in the CHA2DS2-VASc score. A score of ≥ 2 is considered an indication for anticoagulation, whereas either aspirin or anticoagulation is recom m ended for a score of ≥ 1, and aspirin for a score of 0.

184

CAs e FILe s : CAr d IO LOg y

drug or alcohol abuse]. In patients with one or more CHADS2 risk factors, anticoagulation with warfarin (with an INR of 2.0–3.0), dabigatran, rivaroxaban, or apixaban is preferred. Aspirin is used for low-risk patients, although its role is less clear. The risk of stroke and the benefit of anticoagulation is similar in patients who have paroxysmal and persistent atrial fibrillation. The role of anticoagulation is less well studied in patients with atrial flutter, but as this arrhythmia frequently coexists with atrial fibrillation, the same guidelines are used. Two strategies can be used to control the arrhythmia itself: (1) rhythm control, where attempts are made to restore and maintain sinus rhythm through the use of antiarrhythmic drugs, cardioversion, and catheter-based or surgical ablation; and (2) rate control, where atrial fibrillation is accepted as the rhythm but the ventricular rate is controlled with drugs, or occasionally pacemaker implantation with ablation of the AV node in patients with refractory tachycardia. Mortality rates are equivalent between these two approaches, and as most strokes occur when anticoagulation is stopped, this should be continued in patients with risk factors even when sinus rhythm is restored, because of the possibility of asymptomatic recurrences. Studies of drug-treated patients using long-term ambulatory monitoring have demonstrated that asymptomatic paroxysms of atrial fibrillation are common even when office ECGs consistently reveal sinus rhythm during follow-up. Quality of life and functional status is consistently better in patients who successfully maintain sinus rhythm; however, current antiarrhythmic drug therapy is characterized by limited efficacy and toxicity. Nonetheless, this approach is favored in patients who have significant symptoms as a consequence of their atrial fibrillation. Rate control is achieved primarily with beta-blockers and nondihydropyridine calcium channel blockers (eg, diltiazem or verapamil). Digoxin has a limited role as its vagotonic effect is quickly overcome by adrenergic states such as exercise and should rarely be used as the sole rate control agent. Atrial flutter is often more symptomatic and is generally more difficult to rate-control than atrial fibrillation, due to the rapid organized atrial depolarizations. In cases of refractory symptoms or hemodynamic compromise due to rapidly conducted atrial fibrillation, ablation of the AV node with implantation of a pacemaker is a useful strategy. The disadvantage of this approach is that it renders the patient permanently pacemaker-dependent. In patients with left ventricular dysfunction, biventricular pacing should be considered. Class 1C or class 3 antiarrhythmic drugs are used to establish and maintain sinus rhythm in the rhythm control strategy. Class 1C drugs (flecainide or propafenone) are used in patients without structural heart disease. The class 3 drugs sotalol and dofetilide are options in patients with coronary artery disease, but are contraindicated in those with significant renal impairment. Dofetilide and another class 3 drug amiodarone are the only reasonable antiarrhythmic options for patients with left ventricular dysfunction as the 1C agents are associated with an increased risk for dangerous arrhythmia in patients with reduced pump function, and sotalol has significant negative inotropic properties. Amiodarone is perhaps the most effective antiarrhythmic for the treatment of atrial fibrillation but is limited by significant dose-dependent toxicities, especially thyroid and pulmonary. Dronedarone is an amiodarone analog that is contraindicated in patients with heart failure and those with permanent atrial fibrillation.

s e CTIO N II: CLINICAL CAs e s

185

Cardioversion is used in patients with persistent atrial fibrillation. Therapeutic anticoagulation for at least 3 weeks prior to this is needed to ensure the absence of thrombus in the left atrial appendage, and for 4 weeks after, regardless of the baseline risk of thromboembolism. With the return of left atrial contractile function in sinus rhythm, the postcardioversion period presents a higher risk as any such thrombi may be dislodged. In patients who have not been anticoagulated. a transesophageal echo can be performed to exclude the presence of left atrial appendage thrombi; however, therapeutic anticoagulation after cardioversion is still required. Pharmacologic cardioversion can be achieved with oral flecainide or propafenone, or with intravenous ibutilide or amiodarone. Electrical cardioversion is performed by a cardiologist under anesthesia via an external defibrillator. A biphasic shock is delivered through the chest that alters transmembrane gradients via an incompletely understood mechanism, leading to the cessation of atrial fibrillation and the resumption of sinus rhythm. The use of antiarrhythmic drugs after cardioversion delays recurrence of atrial fibrillation and should be strongly considered. Catheter-based ablation uses radiofrequency energy to isolate the pulmonary veins from the left atrium, thereby preventing the rapid discharges that form the electrophysiologic basis of paroxysmal atrial fibrillation from reaching the atrium. Catheter-based ablation is superior to antiarrhythmic drug therapy in the maintenance of sinus rhythm. Success rates are highest in patients with paroxysmal atrial fibrillation, normal or mildly dilated left atria, and without structural heart disease. The procedure is time-consuming and technically challenging, and effects on longterm stroke and mortality rates remain under investigation. Surgical ablation of atrial fibrillation is often combined with concomitant heart surgery (valve surgery or coronary artery bypass), although it can also be performed as a standalone procedure. Results may be comparable to catheter ablation, although recovery time and complication rates are higher, and standalone surgical ablation is generally reserved for those who have failed at or been turned down for catheter ablation. Ablation of atrial flutter is technically more straightforward and associated with higher success rates (95%) and is superior to antiarrhythmic drugs, even as a firstline approach. Patients with symptomatic atrial flutter should be evaluated for catheter ablation rather than repeated cardioversions and extended antiarrhythmic drug therapy. The risk of developing atrial fibrillation in a patient with atrial flutter is approximately 10% per year, so continuation of anticoagulation in patients with risk factors for thromboembolism, even after successful ablation of atrial flutter, should be carefully considered.

186

CAs e FILe s : CAr d IO LOg y

COmPREHENSION Qu ESTIONS Choose the correct answer (A–E) to the following statements: 11.1 An 83-year-old man with a history of hypertension and diabetes is referred to you for management after presenting to the emergency department last week with several hours of recent-onset palpitations. He was found to be in atrial fibrillation with a rapid ventricular rate and underwent successful electrical cardioversion. He was discharged to home from the ED with prescriptions for metoprolol and a limited supply of low-molecular-weight heparin to use while he began warfarin therapy. His point-of-care INR in the office today is 2.2 on warfarin 5 mg daily. The patient is questioning his need for anticoagulation, citing the fact that he is now back in normal rhythm after cardioversion and is not in atrial fibrillation permanently. He reports that his 50-year-old nephew has a diagnosis of atrial flutter and takes aspirin only for stroke prevention. He considers himself healthier than the average 83-year-old man and feels that his risk for stroke must be relatively low. He would like to know more about atrial fibrillation and his risk for stroke. You should tell him which of the following? A. His stroke risk is similar regardless of whether his atrial fibrillation is paroxysmal, persistent, or permanent. B. His stroke risk would be considerably lower if he had atrial flutter like his nephew. C. His stroke risk is similar to that of the age-matched general population. D. His stroke risk 1 week after cardioversion is lower than it was prior to cardioversion E. His stroke risk should be reduced to a similar extent by aspirin and warfarin.

s e CTIO N II: CLINICAL CAs e s

187

11.2 A 75-year-old woman with rheumatic mitral valve disease treated with a mitral bioprosthesis 10 years ago is referred to you for management of her permanent atrial fibrillation. She has been in atrial fibrillation for the past 4 years and was initially asymptomatic, but over the past year she has developed progressive exertional dyspnea, ankle swelling, and fatigue. This correlates with a gradual increase in her resting average heart rate from 89 bpm 1 year ago to 128 bpm as assessed by a recent 48-hour ambulatory monitor. Her primary care physician has attempted to treat her with metoprolol, but her dose has been limited by hypotension. Digoxin was also added, but this did not significantly improve her heart rate. She takes dose-adjusted warfarin for stroke prevention. She has failed numerous electrical cardioversions even in the setting of previous antiarrhythmic therapy with amiodarone and sotalol. Her examination is noteworthy for distended jugular veins, an irregularly irregular tachycardia, faint bibasilar rales, and bilateral pitting edema of the legs. Her ECG reveals atrial fibrillation with a rate of 115 bpm and ST segment findings consistent with digoxin effect. Her QTc interval is 520 ms. You arrange for an office echocardiogram, and this reveals a mildly dilated left ventricle with an ejection fraction of 35%, a dilated right ventricle with mild systolic dysfunction, an intact mitral prosthesis with an average mean gradient of 7 mmHg and trivial regurgitation, and a markedly enlarged left atrium. Her ejection fraction by echocardiogram last year was 55%. In addition to proper medical management of her newly discovered heart failure, what is the most appropriate next step? A. Treatment with dofetilide for rhythm control. B. Treatment with propafenone for rhythm control. C. Referral for catheter-based ablation of her atrial fibrillation. D. Referral for catheter-based ablation of the AV node and pacemaker implantation. E. Referral for redo mitral valve replacement and surgical MAZE procedure. 11.3 Which of the following statements regarding atrial flutter is true? A. Ablation of atrial flutter is more difficult than that of atrial fibrillation. B. Atrial flutter presents a risk of thromboembolism similar to that for atrial fibrillation. C. Rate control in atrial flutter is easier than rate control in atrial fibrillation. D. Atrial flutter is associated with fewer symptoms than atrial fibrillation. E. Atrial flutter is caused by chaotic irregular atrial depolarizations.

188

CAs e FILe s : CAr d IO LOg y

ANSWERS 11.1 A. The stroke risk associated with atrial fibrillation is similar regardless of whether the atrial fibrillation is considered paroxysmal, persistent, or permanent. This is because a large percentage of patients with paroxysmal atrial fibrillation have clinically silent recurrences and some of these episodes can exceed 24 hours in length. Mechanical atrial contraction can remain impaired for days to weeks after cardioversion before normalizing, and for this reason patients are considered to be at increased risk for stroke in the weeks immediately after cardioversion. Atrial flutter is felt to confer approximately the same stroke risk as atrial fibrillation. Given this patient’s elevated CHA2DS2VASc score, his stroke risk would be expected to exceed that of the general population and he would be expected to derive a greater benefit from warfarin than aspirin. 11.2 D. This patient has failed reasonable attempts at both rate and rhythm control, and AV nodal ablation with pacing is the most definitive treatment option. This patient’s symptoms are most likely due to heart failure, which is likely the result of a tachycardia-mediated cardiomyopathy. Antiarrhythmic therapy with dofetilide or propafenone is unlikely to succeed, and both of these agents are contraindicated in this patient (for QT prolongation and LV dysfunction, respectively). With her longstanding atrial fibrillation and marked atrial enlargement, atrial fibrillation ablation would also be unlikely to succeed. Warfarin is the most appropriate choice for her anticoagulation as newer agents such as rivaroxaban have not been studied for use in patients with rheumatic mitral valve disease. This patient has a functional mitral prosthesis and does not require redo open-heart surgery. 11.3 B. Atrial flutter is a fast, organized rhythm that follows a rather predictable circuit involving the right atrium and tricuspid valve isthmus in most cases. Atrial flutter is generally more difficult to rate-control and less responsive to antiarrhythmic therapy than atrial fibrillation, but its predictable pathway makes it quite amenable to successful ablation. The stroke risk related to atrial fibrillation and atrial flutter is considered to be similar.

CASE CORRELATION • See also Case 11 (atrial fibrillation and flutter).

s e CTIO N II: CLINICAL CAs e s

189

CLINICAL PEARLS C

Fati u an x ci intol anc a tion, p ciall in th l l .

common

C

Th i k of th ombo mboli m i at ial fib illation.

imila in pa ox mal an p

C

Anticoa ulation houl b con i t ok .

C

Anticoa ulation houl b continu in pati nt with a ia no i of at ial fib illation who a in inu h thm, a cu nc can b il nt o p nt lat .

C

d i oxin i a thi -lin a th ol th ap .

C

At iov nt icula no ablation an pac mak cont ol if f acto to m ical th ap .

C

Th choic of antia h thmic u to achi v h thm cont ol i compl x an houl b initiat un xp t up vi ion.

C

Cath t ablation of at ial fib illation i in th maint nanc of inu h thm.

at cont ol m

mptom of at ial fib illai t nt

in all pati nt with i k facto

ication an

houl

fo

a l b u

implant i u ful fo

up io to antia h thmic

at

u

REFERENCES Atrial Fibrillation Investigators. Risk factors for stroke and efficacy of antithrombotic therapy in atrial fibrillation. Analysis of pooled data from five randomized controlled trials. A rch Intern Med. 1994;154(13):1449–1457. Da Costa A, Thévenin J, Roche F, et al. Results from the Loire-Ardèche-Drôme-Isère-Puy-de-Dôme (LADIP) trial on atrial flutter, a multicentric prospective randomized study comparing amiodarone and radiofrequency ablation after the first episode of symptomatic atrial flutter. Circulation. 2006;114(16):1676–1681. Fuster V, Rydén LE, Cannom DS, et al. ACC/AHA/ESC 2006 Guidelines for the Management of Patients with Atrial Fibrillation: a report of the American College of Cardiology/American Heart Association Task Force on Practice Guidelines and the European Society of Cardiology Committee for Practice Guidelines (Writing Committee to Revise the 2001 Guidelines for the Management of Patients with Atrial Fibrillation): developed in collaboration with the European Heart Rhythm Association and the Heart Rhythm Society. Circulation. 2006;114(7):e257–e354. Haïssaguerre M, Jaïs P, Shah DC, et al. Spontaneous initiation of atrial fibrillation by ectopic beats originating in the pulmonary veins. N Engl J Med. 1998;339(10):659–666. Lip GY. Implications of the CHA2DS2-VASc and HAS-BLED scores for thromboprophylaxis in atrial fibrillation. A m J Med. 2011;124:111–114. Wilber DJ, Pappone C, Neuzil P, et al. Comparison of antiarrhythmic drug therapy and radiofrequency catheter ablation in patients with paroxysmal atrial fibrillation: a randomized controlled trial. JA MA . 2010;303(4):333–340.

190

CAs e FILe s : CAr d IO LOg y

Wolf PA, Abbott RD, Kannel WB. Atrial fibrillation as an independent risk factor for stroke: The Framingham Study. Stroke. 1991;22(8):983–988. Wyse DG, Waldo AL, DiMarco JP, et al. Atrial Fibrillation Follow-up Investigation of Rhythm Management (AFFIRM) Investigators. A comparison of rate control and rhythm control in patients with atrial fibrillation. N Engl J Med. 2002;347(23):1825–1833.

CAs E 12 A 75- ear-ol man with a hi tor of h perten ion, h perlipi emia, an coronar arter i ea e with rug-eluting tent placement 5 ear ago pre ent with a complaint of lighthea e ne an fatigue, ongoing for the pa t 24 hour . He ha not experience an epi o e of ncope, che t pain, hortne of breath, or palpitation . He wa able to ambulate to our office, an i full conver ant with ou; however, he oe tate that the e activitie were ignificantl more ifficult to perform than u ual, an he ha to top multiple time to re t coming from the parking lot to our office. He appear fatigue in general. He i afebrile with a bloo pre ure of 110/ 60 mmHg, heart rate of 30 bpm, an breathing rate of 14 time per minute. On examination, hi lung are clear, an he oe not have an lower extremit e ema. Hi car iac examination i ignificant for bra car ia without murmur , rub , or gallop . Neck examination how intermittent large jugular venou pul ation . Peripheral pul e are trong. Laborator evaluation how normal bloo count, renal function, electrol te , th roi function te t , an no elevation in erum car iac biomarker . An ECG i obtaine in office an i hown in Figure 12-1. C C C

What i the mo t likel iagno i ? What el e o ou nee to know about hi me ical hi tor ? What i the be t next tep in management?

I

II

III

a VR

V1

V4

a VL

V2

V5

a VF

V3

V6

V1

II

V5 Figure 12-1. An ECG for the main ubject of thi ca e.

s ECTIO N II: CLINICAL CAs Es

193

ANs WER TO CAs E 12: Bradycardia Summary: A 75-year-old man with little comorbid history presents with a 1-week history of fatigue and lightheadedness, with no other significant symptoms. Examination is significant for severe bradycardia. There is no laboratory evidence of renal impairment or acute or ongoing myocardial ischemia or infarction. ECG shows abnormal conduction through the AV node, with dissociation of the atrial (P waves) and ventricular (QRS) activation. • Most likely diagnosis: Complete heart block (third-degree AV block). • What else is needed to know about his medical history? Medications taken by patient or recent changes in his regimen. • N ext step in management: Ventricular pacing.

ANALYSIS Objectives 1. Know how to interpret an electrocardiogram (ECG) and identify the different types of AV block. 2. Recognize key factors that need to be identified and evaluated as part of management of patients with conduction abnormalities. 3. Understand key physical exam findings in patients with complete heart block. 4. Understand different maneuvers that can be employed in the evaluation of patients with second-degree AV block to differentiate Mobitz 1 from Mobitz 2 block.

Considerations This 75-year-old man presents with severe bradycardia for 24 hours, with associated symptoms (lightheadedness and fatigue). The first, and most important, step in diagnosis of a patient presenting with these symptoms and exam finding of bradycardia is electrocardiogram (ECG). The ECG will provide the diagnosis of what the conduction abnormality is (sinus bradycardia vs. second-degree AV block type 1 or type 2 vs. complete heart block), where the ventricular activation is coming from if complete heart block is present (narrow complex suggesting junctional escape, wide complex suggesting ventricular escape), as well as providing other pertinent information such as the presence of active myocardial ischemia or infarction, or interventricular conduction abnormalities (right or left bundle branch block). This ECG shows complete heart block, identified by dissociation of the atrial and ventricular activation, with more P waves than QRS complexes. Further, the ventricular rate is very slow, and in the presence of symptoms, is not ultimately sustainable. The next immediate step is to stabilize the patient with temporary ventricular pacing. While this patient ultimately may require permanent pacemaker implantation, this process would not occur likely for hours at a minimum, and therefore more immediate intervention is warranted.

194

CAs E FILEs : CARd IO LOGy

Transcutaneous pacing with external pads is an option, but not an ideal one as they are often uncomfortable for the patient and are typically unreliable with either complete inability to pace, or intermittent loss of capture. Transvenous temporary pacing provides more reliability, better pacing options, and aside from initial venous access, is more comfortable for the patient while further workup and planning is done; this can be performed by a trained cardiologist or intensivist. Finally, after stabilization, baseline workup can commence. Careful review of the patient’s medication list, as well as recent changes, is needed as many medications (beta-blockers, calcium channel blockers, digoxin, antiarrhythmics) may cause AV block that can be resolved with decreasing dose, cessation, or pharmacologic reversal of the agent. Lab evaluation with complete blood count (CBC), comprehensive metabolic panel (CMP; a standardized group of 14 blood tests), and thyroid-stimulating hormone (TSH) blood tests are important to rule out end-organ dysfunction, which may result from hypoperfusion due to bradycardia, as well as ensure that electrolyte or thyroid dysfunction are not culprits in the presenting situation. Cardiac evaluation should include exclusion of myocardial infarction with serial CK, CK-MB, and troponin T or I, as active ischemia or infarct may cause AV conduction abnormalities; and an echocardiogram should be performed to evaluate for baseline LV function, wall motion, and valvular abnormalities. In patients with known severe coronary artery disease, or suspicion of disease due to concomitant symptoms (chest pain, shortness of breath, heart failure symptoms), evaluation for ischemia or severe coronary disease may be necessary with noninvasive stress testing or cardiac catheterization.

APPROACH TO: Bradycardia DEFINITIONS BRADYCARDIA: Ventricular rate 200 ms on ECG. Second-degree AV block comes in two varieties: Mobitz type 1 (or Wenckebach) and Mobitz type 2. Mobitz type 1 AV block is defined by a progressive PR prolongation with eventual “dropped” QRS complex (Figure 12-2a). The subsequent PR interval after the dropped beat is short, and again shows progressive prolongation in subsequent beats. Mobitz type 1 block typically occurs within the AV node; therefore, maneuvers that improve AV conduction will improve the block (exercise, atropine), whereas maneuvers which decrease AV conduction will worsen the degree of block (carotid sinus massage). Mobitz 1 block does not typically progress to highgrade block (Mobitz II or complete heart block). Mobitz type 2 second-degree AV block is defined by a stable PR interval in consecutive beats, with nonconduction or “dropped QRS” occurring at either regular or irregular intervals (Figure 12-2b). Mobitz 2 block typically occurs below the AV node, in the His-Purkinje system. As a result of the disease at this level, Mobitz 2 is more often associated with bundle branch block (but not the rule). Maneuvers that improve conduction through the AV node actually worsen Mobitz 2 block, as this decreases recovery time for the His-Purkinje system (exercise, atropine); by contrast, carotid sinus massage will improve Mobitz 2 block, as this slows AV nodal conduction enough to allow the His-Purkinje system to recover and conduct more impulses. Because of instability of the His-Purkinje system in Mobitz 2 block, there is a high rate of progression to complete heart block. Differentiation between Mobitz types 1 and 2 second-degree AV block is difficult when the ratio of AV conduction is 2:1 (Figure 12-2c). This is clinically relevant

196

CAs E FILEs : CARd IO LOGy

A

B

II C

II D Figure 12-2. T pe of atrioventricular (AV) block: (A) econ - egree AV block, Mobitz t pe 1 (Wenckebach); (B) econ - egree AV block, Mobitz t pe 2; (C) econ - egree AV block with 2:1 AV con uction; (d ) thir - egree (complete) AV block.

because the likelihood of progression to complete heart block is higher with Mobitz type 2 AV block and knowing the mechanism may influence management (such as the decision to place a transvenous pacing wire). In this circ*mstance there are some characteristics that may help one distinguish between types 1and 2 (Table 12-1). Third-degree block (complete heart block) is defined as complete dissociation of the atria and ventricles (Figure 12-2d). Typically, in complete heart block, the PP interval and the RR interval are consistent from beat to beat, but are not the same. PR interval will vary as well due to the dissociation. Typically, atrial rate will be higher than the junctional or ventricular escape rate, and more P waves will be noted than QRS complexes.

Clinical Presentation Patients presenting with complete heart block, or even second-degree AV block, can have various symptoms at presentation. Patients typically experience symptoms consistent with decreased perfusion to end organs. Specifically, lightheadedness, dizziness, fatigue, confusion, and syncope are common. Also associated are shortness of Table 12-1 • CHARACTERISTICS OF TYPE 1 (AV NODAL) VS. TYPE 2 (HIS-PURKINJE) BLOCK IN 2:1 BLOCK Parameter

AV Node (Mobitz 1)

His-Purkinje (Mobitz 2)

T pical QRs complex

Narrow

Wi e (bun le branch block)

PR uration

Long, lengthen

Normal

Wor en

Improve

Re pon e to exerci e

Improve

Wor en

Re pon e to atropine

Improve

Wor en

Re pon e to caroti

inu ma age

s ECTIO N II: CLINICAL CAs Es

197

breath with exertion and angina-like pain. Block due to acute myocardial infarction may present with chest pain or referred pain. Onset of the symptoms may be gradual over hours, or abrupt; in some cases symptoms are not noted until the patient begins physical activity, at which point they cannot increase cardiac output enough to match oxygen demand. The history taken should be detailed, and specifically focused on when the symptoms started, as that will likely indicate the duration of their bradyarrhythmia. Associated concerning symptoms such as chest pain should elicit evaluation for ischemic heart disease. Detailed review of medications, particularly in those with multiple comorbidities, is necessary; medications with AV nodal blocking activity need to be identified, and dosing, frequency, compliance, and changes in regimen of these medications needs to be discussed. The physical examination should first note vitals, particularly heart rate (before pacing) and blood pressure. Some patients tolerate bradycardia well with normal blood pressure, whereas others may experience significant hypotension as well. Neck examination may reveal “cannon A waves” of the jugular venous pulsation, consistent with right atrial contraction against a closed tricuspid valve. Lung examination is typically normal, but findings consistent with heart failure such as crackles need to be addressed quickly and portend a more significant disease process. Cardiac exam will identify bradycardia, but also may note variable intensity of S1. An S3 may also identify those with volume overload due to uncoordinated atrial and ventricular conduction. Murmurs should be noted as well, as valvular endocarditis may cause regurgitant disease with infiltration of the conduction system.

Treatm ent The first step in management of a patient presenting with bradycardia is determination of the rhythm. In those with symptoms consistent with their bradycardia, or high-degree block, immediate action should be taken to increase the ventricular rate, typically with temporary pacing. Once stabilized, determination of the etiology is the next step. Evaluation for electrolyte disturbance, thyroid or renal dysfunction, and myocardial ischemia should all be undertaken with initial laboratory tests. Any of these abnormalities should be treated aggressively, as correction of the underlying problem (hyperkalemia, thyroid dysfunction, etc) may improve the conduction and obviate the need for permanent pacing. Evidence of myocardial ischemia or active infarction should be addressed with cardiac catheterization and intervention if necessary. Recent cardiac surgery may cause sinus node dysfunction and subsequent bradycardia, particularly junctional bradycardia, but this often improves with time. Careful medication review is of extreme importance in the assessment of patients with bradycardia, particularly elderly patients in whom polypharmacy is quite common. Medications that can cause AV block (and medications that may potentiate their effect) should be of particular concern. Changes in regimen and compliance (double dosing) should be noted, or changes in organ function that may cause elevated drug levels. Monitoring and withholding of the medication is often all that is required, and once levels decrease, AV function may improve. However, in extreme cases, medication reversal may be necessary with appropriate reversal agents.

198

CAs E FILEs : CARd IO LOGy

Finally, in cases of persistent symptomatic bradycardia, permanent pacing may be required. Consultation with a cardiac electrophysiologist is necessary, for implant of the device, as well as selection of the appropriate type of device (single-chamber, dual-chamber, etc).

COMPREHENSION QUESTIONS 12.1 A 45-year-old woman misunderstood her physician’s instructions and took 10 pills of metoprolol instead of 1 tablet. She presents with some lethargy and a heart rate of 44 bpm and BP of 100/60 mmHg. Which of the following is the best treatment? A. Digoxin B. Glucagon C. Glucose D. Propranolol E. Warfarin 12.2 An 86-year-old man with chlorthalidone-treated hypertension presents for evaluation of his 6-month complaint of progressive exercise intolerance. He reports a steady decrease in stamina while walking and as of late has noted dyspnea and lightheadedness after walking for a few minutes. He saw his primary care physician, who arranged for a treadmill ECG study that was nondiagnostic for myocardial ischemia because of failure to achieve target heart rate (he achieved 60% of his maximum predicted heart rate) with no ST segment deviation despite symptoms of shortness of breath and dizziness. His ECG reveals sinus bradycardia with a ventricular rate of 45 bpm and a nonspecific intraventricular conduction abnormality with leftward axis deviation and QRS duration of 122 ms. Basic laboratory data including TSH are normal, and a resting echocardiogram reveals preserved biventricular function with no significant valvular disease and stage 1 diastolic dysfunction. What is the most appropriate next step? A. Refer for pacemaker implantation to treat symptomatic bradycardia B. Refer for coronary angiography to exclude coronary artery disease C. Refer for phase 2 cardiac exercise to treat deconditioning D. Discontinue chlorthalidone because of a possible medication side effect E. Begin beta-blockers for diastolic heart failure

s ECTIO N II: CLINICAL CAs Es

199

12.3 Which of the following statements about second-degree AV block is most accurate? A. Mobitz type 1 block is likely to progress to complete AV block B. Mobitz type 2 block tends to improve when the heart rate increases C. Mobitz type 1 block occurs within the AV node D. Mobitz type 2 block tends to worsen with carotid sinus massage E. Mobitz type 1 block features a normal PR interval

ANSWERS 12.1 B. Glucagon is the best antidote to beta-blocker overdose. Digoxin and propranolol are both AV nodal blocking agents that would be expected to worsen bradycardia. Glucose should have little effect on the heart rate in the absence of profound hypoglycemia, and warfarin is an anticoagulant that should have no effect on the AV node. 12.2 A. This patient has symptomatic bradycardia manifesting as chronotropic incompetence or failure of the heart rate to increase to meet physical demands. He also has conduction disease beyond the AV node, and the most appropriate treatment choice would be the implantation of a rate-responsive permanent pacemaker. Coronary angiography is not indicated as this patient does not have symptoms strongly suggestive of coronary disease. Cardiac exercise would be unlikely to help, given that his exercise capacity is limited by his slow heart rate. A thiazide-type diuretic such as chlorthalidone should not cause bradycardia, but the introduction of a beta-blocker such as metoprolol would almost certainly worsen his symptoms. 12.3 C. Mobitz type 1 block occurs within the AV node itself and is often due to dynamic inhibition of the node by drugs or increased vagal tone. The PR interval gradually increases until a QRS drops and the QRS width is normal (provided that there is no underlying distal conduction disease). Vagal maneuvers such as carotid massage tend to worsen the degree of block, whereas sympathetic stimulation or vagolytic medications such as atropine lessen the block. Mobitz type 2 block occurs just distal to the AV node, often resulting in subtle widening of the QRS. The block can handle only a fixed rate of impulses from above, so increasing the heart rate produces more block and slowing the heart rate results in less block. Mobitz type 2 is more likely to progress to complete heart block than Mobitz type 1.

200

CAs E FILEs : CARd IO LOGy

CLINICAL PEARLS C

Complete heart block ma pre ent with multiple mptom , inclu ing lighthea e ne , fatigue, ncope, an hortne of breath.

C

The mo t common cau e of AV block i i iopathic fibro i an clero i of the con uction tem. The econ mo t common cau e i i chemic heart i ea e.

C

s econ - egree AV block with 2:1 con uction can complicate ifferentiation between t pe 1 (Wenckebach) an t pe 2 Mobitz. Maneuver that improve AV con uction (exerci e, atropine) ten to improve t pe 1 block while wor ening t pe 2 block.

C

Maneuver that ecrea e AV con uction (caroti inu ma age) ten to wor en Mobitz 1 block but will improve t pe 2 block (a a re ult of lowe AV no al con uction, allowing the Hi -Purkinje tem to recover for the ub equent impul e).

C

In patient with complete heart block an low rh thm , initial management after ECG i obtaine houl be irecte at provi ing ventricular pacing, preferabl with tran venou temporar pacemaker wire , until po ible etiolog can be etermine , with arrangement for long-term pacing (permanent pacemaker implant).

REFERENCES Mangrum JM, DiMarco JP. The evaluation and management of bradycardia. N Engl J Med. 2000;342: 703–709. Olgin JE, Zipes DP. Specific arrhythmias: Diagnosis and treatment. In: Libby P, et al. Braunwald’s Heart Disease. 8th ed. Philadelphia, PA: Saunders, Elsevier; 2008:913–920. Sauer WH. Etiology of atrioventricular block. UpToDate 2014. Leonard I. Ganz, Brian C. Downey, Editors. http://www.uptodate.com.

CASE 13 A 24- ear-o d woman with no prior medica histor presents to our c inic with a comp aint of sporadic episodes of intermittent pa pitations and mi d shortness of breath asting 1–2 hours at a time over the past 6–8 months, about twice per week. She has not experienced an episodes of s ncope, chest pain, ower extremit edema, orthopnea, or d spnea with exertion; she reports that the mi d shortness of breath occurs on when she is experiencing her pa pitations. She exercises for 30 minutes, 4 times per week, without imitations. With regard to her pa pitations, she states that the occur spontaneous without c ear provocation. Once started, the pa pitations are constant unti the abrupt stop, usua about 1–2 hours ater; when most severe, she ies down and tries to re ax, but the pa pitations continue. During a few of her episodes, she or one of her friends pa pated her pu se, with a consistent reading at approximate 150–160 bpm. She has not presented ear ier for eva uation because the episodes reso ve before she can present for eva uation, and she has been “ver bus ” with her graduate schoo work. Her genera appearance is norma and she is in no distress. Vita signs inc ude b ood pressure 110/ 60 mmHg and heart rate of 65 bpm. Ph sica exam is unrevea ing; no JVP is noted, and there are norma ung findings and no cardiac murmurs, ga ops, or rubs. Heart sounds are norma , with regu ar rate and rh thm. A base ine ECG is obtained in office and shows norma sinus rh thm. After discussion with her, further workup is schedu ed for her return visit to the c inic at a ater date. However, 2 da s ater, ou get a ca from her that she just began having pa pitations. She comes to the c inic urgent , and a repeat ECG is obtained for review (Figure 13-1). Whi e tach cardic, she has subjective pa pitations, but remains hemod namica stab e with s sto ic b ood pressure of 105 mmHg. C C C

What is the differentia diagnosis of this patient’s condition? What is the most ike diagnosis? What is the best next step in management?

I

a VR

V1

V4

II

a VL

V2

V5

III

a VF

V3

V6

V1

II

V5 Figure 13-1. ECG for the main subject of this case.

SECTIO N II: Cl INICAl CASES

203

ANSWER TO CASE 13: AV Nodal Reentrant Tachycardia Summary: A 24-year-old woman with no medical history presents with intermittent episodes of palpitations lasting 1–2 hours at a time, about 2 days per week, for several months. Initial assessment was negative, but before completion of further testing, she had another episode with documented ECG showing a narrow complex tachycardia. The tachycardia is regular, with a short RP interval, and a pseudo-R′ appearance in lead V1, which is actually the retrograde P wave (atrial activation from the AV node) occurring nearly simultaneously with the QRS (ventricular activation). • Differential diagnosis: AVNRT, AVRT, atrial fibrillation, atrial flutter, atrial tachycardia. • Most likely diagnosis: Atrioventricular nodal reentrant tachycardia (AVNRT). • N ext step in management: Maneuvers to confirm rhythm (carotid sinus massage, adenosine).

ANALYSIS Objectives 1. Know how to interpret an electrocardiogram (ECG) to classify narrow complex tachycardia (NCT) to help form a differential diagnosis. 2. Learn maneuvers–both physical and with medications–to help definitively diagnose the arrhythmia. 3. Learn treatment strategies for NCT, including catheter ablation.

Considerations This 24-year-old woman presents with intermittent, symptomatic tachycardia with palpitations. As is often the case, on her first presentation to your office she is asymptomatic and in normal rhythm. But on her return, while experiencing palpitations, she is tachycardic, with ECG showing a NCT with rate of ~150 beats per minute. The first step in her management is to diagnose her arrhythmia. Immediate conversion (either medical or electrical) to sinus rhythm is not necessary here as she is tolerating her tachycardia well and is hemodynamically stable. While ECG is necessary for diagnosis, often the multiple different types of NCT can appear similar. Differentiation of NCT starts with separation into long-RP-interval (atrial tachycardia, sinus tachycardia, AVRT, atrial flutter) and short-RP-interval (AVNRT most common) tachycardias, although considerable overlap is present (atrial tachycardia, AVRT, and atrial flutter can be either). A pseudo-R′ is identified in lead V1 (short RP interval), making AVNRT the most likely arrhythmia; however, it is still possible that it could be AVRT, atrial tachycardia, or atrial flutter. Therefore, maneuvers, both physical and medical, can be applied to help differentiate her rhythm.

204

CASE FIl ES: CARDIO l OGy

The purpose of vagal maneuvers is to slow AV nodal conduction, thereby allowing a slower ventricular rhythm with ongoing atrial arrhythmia, or to ascertain whether the rhythm is dependent on the AV node as part of a reentrant loop, in which case the arrhythmia will terminate. The first of these steps consists in performing vagal maneuvers. Valsalva or coughing can be employed but can interfere with ECG if the patient moves or strains too much. Therefore carotid sinus massage (CSM) may be best, and should be performed while running a continuous rhythm strip or ECG. If CSM is not feasible (carotid bruit present, patient body habitus, central venous catheters in the inpatient setting, etc), then pharmacologic assessment can be achieved with adenosine. Adenosine acts to slow or block AV nodal conduction, and provides the same diagnostic answers as vagal maneuvers. Comparison with other AV nodal blocking agents (beta-blockers, calcium channel blockers), reveals adenosine to have the advantage of an extremely short half-life. (Its t1/2 is so short, in fact, that if it is injected into an IV in the patient’s hand, the drug may be metabolized before it reaches the heart!) If the arrhythmia is dependent on the AV node to continue (reentrant loop that includes the AV node), then these maneuvers will terminate the rhythm (AVNRT, AVRT). If the arrhythmia is not dependent on the AV node, these maneuvers will only serve to slow ventricular rate by decreasing the number of impulses conducting through the AV node while the atrial arrhythmia continues. Therefore, in atrial flutter, multifocal atrial tachycardia, and atrial tachycardia, this allows for further evaluation of the atrial signal and identification of the rhythm. The next step is to complete workup of the patient, which includes laboratory evaluation with CBC, basic metabolic panel (BMP), and TSH. In some instances, atrial arrhythmias (any type) can be due to coronary ischemia or active infarction, and evaluation for coronary artery disease can be pursued in specific populations, particularly those who have chest pain with the tachycardia. Also, underlying processes (lung disease) may lead to these arrhythmias, and in select populations should be investigated. After initial evaluation and identification of the rhythm, the final step is rhythm management, with short- and long-term therapy. If adenosine fails to terminate the rhythm during evaluation, then other agents (beta-blockers, calcium channel blockers) can be used. These are usually very effective in suppression of atrial arrhythmias. Long-term management can be strictly medication-based; however, most forms of non-AV tachycardia are very amenable to catheter-based ablation, with success rates over 95% in the electrophysiology (EP) lab.

APPROACH TO: Narrow Complex Tachycardia DEFINITIONS N ARROW COMPLEX TACHYCARDIA (N CT): Fast arrhythmia with heart rates >100 bpm, and QRS duration 55%). This led to an approximately 0.25–1 relative third reduction in mortality over medium-term follow-up in several randomized clinical trials. In the current era, about 10% of patients with heart failure and a primary prevention ICD receive appropriate ICD therapy in the first year after implant, with slightly fewer receiving an inappropriate shock (a shock delivered for a rhythm other than VT/VF). Implantation within 40 days

SECTIO N II: Cl INICAl CASES

219

of myocardial infarction or 3 months of surgical or percutaneous revascularization is seldom performed, to allow for improvement in LVEF and as clinical trials have failed to show benefit within these timeframes. The prognosis of sudden cardiac arrest remains grim. Of the just over half of events in which resuscitation is attempted, fewer than 10% survive to hospital discharge. There is significant regional variation within the United States, likely due to the efforts of some communities to train the general population in CPR and improve and coordinate emergency medical service (EMS) assets. Prognosis is improved if the initial rhythm is VT or VF (therefore treated with a defibrillation shock and termed “shockable rhythms”), if the SCA is witnessed, and if bystander CPR is administered. Future directions in the prevention and treatment of SCA lie in “primordial prevention,” which refers to interventions to reduce the prevalence of risk factors for coronary artery disease in the population, and refinement of the current LVEF-centered approach to risk stratification in those with established risk factors for SCA.

CASE CORRELATION • See also Case 1 (acute coronary syndrome/STEMI), Case 2 (acute coronary syndrome/NSTEMI), and Case 8 (hypertrophic obstructive cardiomyoopathy).

COMPREHENSION QUESTIONS 14.1 A 55-year-old man presents via the emergency department for management following a witnessed cardiac arrest that was preceded by chest pain and successfully aborted by a bystander who used an automated external defibrillator (AED) at the scene. His ECG following his resuscitation revealed 3-mm ST segment elevation in the inferior leads, and he was sent for emergent coronary angiography that revealed acute thrombotic occlusion of the proximal right coronary artery. He was treated with aspiration thrombectomy and PCI with stenting, and his remaining hospital course was unremarkable. His predischarge echocardiogram revealed preserved left ventricular systolic function with mild inferior hypokinesis and an ejection fraction of 55%. He was discharged on metoprolol, lisinopril, atorvastatin, aspirin, and ticagrelor. Which of the following statements is true regarding the secondary prevention of cardiac arrest in this patient? A. This patient should undergo ICD implantation prior to discharge. B. This patient should undergo ICD implantation 3 months after his event. C. This patient should have an electrophysiology study to assess his need for an ICD. D. This patient should be treated with an antiarrhythmic drug. E. This patient should require no additional management for arrhythmia.

220

CASE FIl ES: CARd IO l OGy

14.2 Which of the following statements regarding out-of-hospital sudden cardiac arrest (SCA) is true? A. Bystander CPR is associated with reduced survival in SCA. B. An initial rhythm of VT/VF is associated with reduced survival in SCA. C. Interruption of CPR does not impact outcome in SCA. D. The provision of AEDs in public places is associated with reduced survival in SCA. E. Therapeutic hypothermia improves neurological outcome after SCA in patients with depressed mental status after resuscitation. 14.3 A 30-year-old woman with a peripartum cardiomyopathy sees you for a scheduled follow-up visit. She was diagnosed with dilated cardiomyopathy 10 months ago when she was hospitalized with decompensated heart failure, and her ejection fraction was found to be approximately 10%. She was treated with medical therapy, which presently includes carvedilol, ramipril, spironolactone, and furosemide, and over time her ejection fraction increased to 25%, but it has remained there in the face of maximal medical therapy. She presently feels well and describes mild dyspnea with moderate physical activity. Her physical examination today is unremarkable. Her ECG reveals sinus rhythm with a heart rate of 60 bpm and a narrow QRS (94 ms). At her last office visit you discussed the possible role of an implantable cardiac defibrillator (ICD) in her care. Her echocardiogram today reveals a dilated left ventricle with an ejection fraction of 23%. Which of the following statements regarding ICD therapy for this patient is true? A. ICD therapy will improve her exertional symptoms. B. ICD therapy will reduce her mortality. C. ICD therapy will allow her to discontinue some of her current medications. D. ICD therapy would be equivalent to treatment with amiodarone in terms of survival. E ICD therapy would likely result in at least one shock the 12 months following implantation.

ANSWERS 14.1 E. This patient should not require additional management for arrhythmia. Although he presented with a shockable (VT/VF) cardiac arrhythmia, this occurred in the context of an acute inferior myocardial infarction that was promptly and successfully revascularized. In the absence of recurrent ventricular arrhythmias and in the presence of normal systolic function, this patient would be considered to be at acceptably low risk for recurrent arrhythmia and would not require additional assessment with an EP study or consideration for ICD implantation. Antiarrhythmic therapy has not conclusively reduced mortality in patients who survival cardiac arrest.

SECTIO N II: Cl INICAl CASES

221

14.2 E. Therapeutic hypothermia for 24–36 hours following revival after cardiac arrest in patients with altered mental status. Survival following out-of-hospital cardiac arrest has improved in recent years in part because of better awareness of heart disease in the general population and the availability of AED therapy in public places. Immediate, uninterrupted bystander CPR also improves outcome, as does presenting initially with a “shockable” rhythm such as VT or VF. 14.3 B. This patient would be expected to derive a survival benefit from ICD implantation. ICD implantation does not improve heart failure symptoms unless the patient undergoes implantation of a cardiac resynchronization therapy (CRT) device; our patient has mild symptoms and a normal QRS duration, which would likely preclude CRT therapy. In the first year after implantation approximately 10% of heart failure patients receiving an ICD for primary prevention will receive an appropriate shock from their devices.

CLINICAL PEARLS C

The term sudden cardiac death (SCd ) app ies if the patient has ie – otherwise, sudden cardiac arrest (SCA) is use .

C

Most episo es of SCd occur in patients with coronar arter isease; such patients shou un ergo risk assessment with echocar iograph to etermine l VEF.

C

Continuous high-qua it CPR, fo owe b a vance car iac ife support (ACl S) a gorithms an further iagnostic an therapeutic proce ures as ictate b the in ivi ua circ*mstances, are the priorities in the management of SCA.

C

An ICd shou be offere to survivors of car iac arrest in whom a c ear transient an reversib e cause cannot be proven.

C

Primar prevention ICd s are offere to patients with heart fai ure an l VEF ≤35% on optima me ica therap .

C

Primar prevention ICd s ea to a 0.25–0.33 re uction in morta it in se ecte patients.

REFERENCES Buxton AE. Risk stratification for sudden death in patients with coronary artery disease. Heart Rhythm. 2009;6:836–847. Connolly SJ, Hallstrom AP, Cappato R, et al. Meta-analysis of the implantable cardioverter defibrillator secondary prevention trials. Eur Heart J. 2000;21(24):2071–2078. Epstein AE, DiMarco JP, Ellenbogen KA, et al. American College of Cardiology Foundation; American Heart Association Task Force on Practice Guidelines; Heart Rhythm Society. 2012 ACCF/AHA/ HRS focused update incorporated into the ACCF/AHA/HRS 2008 guidelines for device-based therapy of cardiac rhythm abnormalities: a report of the American College of Cardiology Foundation/ American Heart Association Task Force on Practice Guidelines and the Heart Rhythm Society. J A m Coll Cardiol. 2013;61:e6–e75.

222

CASE FIl ES: CARd IO l OGy

Hazinski MF, Nolan JP, Billi JE, et al. Part 1: Executive summary: 2010 international consensus on cardiopulmonary resuscitation and emergency cardiovascular care science with treatment recommendations. Circulation. 2010;122(16 Suppl 2):S250–S2575. The Hypothermia After Cardiac Arrest Study Group. Mild therapeutic hypothermia to improve the neurologic outcome after cardiac arrest. N Engl J Med. 2002;346:549–556. Myerburg RJ, Castellanos A. Cardiac arrest and sudden cardiac death. In: Libby P, Bonow RO, Mann DL, Zipes DP, eds. Braunwald’s Heart Disease: A Textbook of Cardiovascular Medicine. 9th ed. Elsevier, Sauders, Philadelphia, PA; 2011:chap 41. The Public Access Defibrillation Trial Investigators. Public-access defibrillation and survival after out-ofhospital cardiac arrest. N Engl J Med. 2004;351:637–646.

CASE 15 A 54- ea -ol man p esents to the eme genc epa tment complaining of tach palpitations an lighthea e ness. He enies s ncope, chest pain, exe tional spnea, o thopnea, lowe ext emit e ema, nausea, vomiting, o iapho esis. His s mptoms have been inte mittent fo seve al weeks, usuall lasting less than 1 minute. Howeve , to a his s mptoms have pe siste fo seve al hou s. His me ical histo inclu es co ona a te isease with a emote infe io m oca ial infa ction equi ing stenting, h pe tension, h pe lipi emia, iabetes mellitus, an a 40-pack/ ea histo of smoking with mo e ate to seve e ch onic obst uctive lung isease. His me ications a e aspi in, ato vastatin, metop olol succinate, lisinop il, metfo min, an an inhale long-acting β-agonist an anticholine gic b oncho ilato . On exam, he is mentating app op iatel but appea s anxious. His bloo p essu e is 105/ 68 mmHg, his pulse is 175 bpm, his ox gen satu ation is 93%, an he is afeb ile. The e is no jugula venous istention. Pulmona exam emonst ates a few scatte e expi ato wheezes bilate all but no ales. Ca iac exam is pe tinent fo tach ca ia but no mu mu s o ubs. The ab ominal, musculoskeletal, neu ologic, an skin exams a e benign. Labs a e no mal, inclu ing potassium, magnesium, an a point-of-ca e t oponin T (TpT). Baseline ECG (Figu e 15-1) an ECG obtaine on p esentation (Figu e 15-2) a e shown. C C C

What is the most likel iagnosis? What is the best next iagnostic step? What is the best next step in the ap ?

I

a VR

V1

V4

II

a VL

V2

V5

III

a VF

V3

V1 Figure 15-1. Baseline ECG fo the main subject of this case.

V6

I

a VR

V1

V4

II

a VL

V2

V5

a VF

V3

III

V1 Figure 15-2. P esenting ECG fo the main subject of this case.

V6

226

CASE FILES: CAr d IO LOGy

ANSWEr TO CASE 15: Wide Complex Tachycardia Summary: A 54-year-old-man with a history of coronary artery disease and myocardial infarction presents to the emergency room with sustained tachypalpitations. He has been having intermittent symptoms of the same palpitations over the preceding several weeks, but this is the first time it has lasted more than a minute. He denies symptoms of presyncope, syncope, chest pain at rest or with exertion, dyspnea on exertion, orthopnea, lower extremity edema, nausea, vomiting, or diaphoresis. His blood pressure is 105/68 mmHg, his pulse is 175 bpm, his oxygen saturation is 93%, and he is afebrile. His exam demonstrates tachycardia and is without signs of heart failure. Lab work is normal. His ECG is shown in Figure 15-1. • Most likely diagnosis: Ventricular tachycardia. • N ext diagnostic step: Review ECG for Brugada’s criteria. • N ext step in therapy: If the patient is unstable in any manner, perform emergent synchronized cardioversion. If the patient is stable, consider administration of an IV antiarrhythmic drug such as amiodarone and/or urgent cardioversion.

ANALYSIS Objectives 1. Be able to identify a wide complex tachycardia (WCT) and know the next immediate steps in treatment. 2. Become familiar with the differential diagnosis for wide complex tachycardia, and understand the criteria used to differentiate ventricular tachycardia (VT) from supraventricular tachycardia (SVT) with aberrancy. 3. Understand the criteria for placement of an implantable defibrillator for primary and secondary prevention.

Considerations This patient’s ECG shows a wide complex tachycardia (WCT). While there is a differential diagnosis that needs to be considered, the clinical status of patients in a WCT can vary from stable to cardiac arrest, and status may change rapidly. Therefore, the first priority for any patient in a WCT is to assess their clinical status. If there is no pulse, there should be immediate initiation of CPR along with advanced cardiovascular life support (ACLS) or basic life support (BLS) algorithms, including cardioversion. Even in the setting where a pulse is present, if there are signs of clinical instability, defined as hypotension (SBP < 90 mmHg), altered mental status, angina, or heart failure, the patient requires immediate cardioversion. In both cases, cardioversion should be synchronized if possible, reserving unsynchronized cardioversion for ventricular fibrillation (VF). In settings where the patient is stable, such as the one described in this case, the etiology of the WCT should be assessed through an evaluation of the ECG and history taking.

SECTIO N II: CLINICAL CASES

227

In an unselected patient population presenting with a WCT, the rhythm is ultimately determined to be ventricular tachycardia (VT) in 80% of cases. This number increases to >90% in patients with a prior myocardial infarction. VT is also more common in adult patients, compared to pediatric patients. Therefore, in an adult patient presenting with a WCT, if the rhythm cannot be determined, it is reasonable to treat them as if it is VT. This is especially true because treating a SVT as a VT is safe, while treating a VT as SVT can be harmful.

APPr OACH TO: Wide Complex Tachycardias DEFINITIONS WIDE COMPLEX TACHYCARDIA (WCT): A rhythm with a rate >150 bpm and a QRS duration of >120 ms. VEN TRICU LAR TACHYCARDIA (VT): A rhythm originating below the level of the AV node. Most commonly, this rhythm originates within the ventricular tissue itself and is associated with scarring in the ventricle. On the ECG this generally produces a wide QRS as the electrical signal propagates cell to cell through the ventricles without the aid of the His-Purkinje system. SU PRAVEN TRICU LAR TACHYCARDIA (SVT): A rhythm originating at the level of the AV node or higher, namely, the SA node, atria, or the AV node itself. These rhythms tend to produce a narrow QRS complex as the electrical signal from the AV node propagates via the His-Purkinje system. However, the QRS can be wide if there is an accessory pathway or aberrant conduction. ACCESSORY PATHWAY: A connection between the atrium and the ventricle that is not part of the AV node. Accessory pathways can result in preexcitation of ventricular tissue, characterized by a delta wave as is seen in Wolff-ParkinsonWhite (WPW) syndrome. An accessory pathway can also result in atrioventricular reentrant tachycardia (AVRT) in which a circuit is created as the signal from the SA node travels down the accessory pathway, up the AV node, and back down the accessory pathway–or the reverse. ABERRAN T CON DU CTION : Conduction of a supraventricular impulse into the ventricle in an abnormal manner, such as is seen with a right or left bundle branch block. This abnormal conduction produces a wide QRS.

CLINICAL APPROACH Etiologies Wide complex tachycardias are the result of a rhythm originating in the ventricle or a SVT conducting aberrantly or via an accessory pathway. However, by far the most common WCT is VT. VT is described as either sustained (episodes lasting >30 seconds) or nonsustained VT. VT is further divided into monomorphic VT (MMVT)

228

CASE FILES: CAr d IO LOGy

and polymorphic VT (PMVT). In MMVT all the beats look the same because they are all coming from the same source (Figure 15-3a). This source is either an area within the ventricle with increased automaticity or, more typically, from a reentrant circuit. The reentrant circuit is similar to those found in atrial flutter or AVNRT, and classically occurs around an area of scar from a myocardial infarction. This is why WCTs are more likely to be VT in patients with known CAD, as they are the patients with the substrate to form these circuits. Patients with cardiomyopathy and decreased left ventricular ejection fractions are also at high risk for VT. Polymorphic VT is a WCT where the QRS changes morphology, axis, or both (Figure 15.3b). Unlike the scenario in MMVT, where there is one source initiating the rhythm, in PMVT there are multiple simultaneous circuits. The changes in morphology seen on the ECG are the summed vectors of these multiple electric circuits within the ventricle. PMVT is often categorized into those without prolonged QT and those with prolonged QT. The QT interval is measured from the onset of the Q wave to the end of the T wave, and represents ventricular repolarization. It is affected by heart rate, and is corrected using Bazett’s formula, dividing the QT interval by the square root of the RR interval (QTc = QT/√RR), where QTc is the corrected QT interval. The QTc is prolonged for men if it lasts for >450 ms and for females, for >470 ms. QTc prolongation can be inherited or acquired through electrolyte abnormalities or medication use. Hypokalemia and hypomagnesemia can cause QTc prolongation, as can the use of medications such as quinolone antibiotics (ciprofloxacin), antipsychotics (Haldol), antiarrhythmic drugs (amiodarone, flecanide, sotalol), and many other medications. In patients with a normal QTc and PMVT, most cases are related to myocardial ischemia (Figure 15-3b). In patients with a prolonged QTc and PMVT, it is called torsades de pointes (TdP), which translates to “twisting of the points.” This description is derived from the “twisting” of the QRS complex around the isoelectric baseline. Long QT is a predisposition for an R-on-T phenomenon, which occurs when a premature ventricular contraction lands on the T wave. At this point in the cardiac cycle the ventricle is in a relative refractory period, and the extra stimuli from the PVC can initiate TdP (Figure 15-3c). Ventricular fibrillation (VF) occurs when there is uncoordinated contraction of the ventricle. This is always a nonperfusing rhythm and a dire medical emergency (Figure 15-3d). This should be treated with immediate initiation of CPR along with advanced cardiovascular life support (ACLS) or basic life support (BLS) algorithms, including defibrillation. VF is seen in cardiac ischemia, severe electrolyte disturbances, and overdoses of cardiotoxic medications. Supraventricular tachycardias are rhythms originating at the level of the AV node or higher, namely, the SA node, atria, or the AV node itself. These rhythms tend to produce a narrow QRS complex as the electrical signal from the AV node propagates via the His-Purkinje system. However, in two specific circ*mstances a SVT can cause a WC: when there is an accessory pathway, or when there is aberrant conduction. If there is an accessory pathway, the signal can reach the ventricle through this access point before the signal from the AV node. These two eventually meet and form the QRS, but the signal from the accessory pathway comes earlier

SECTIO N II: CLINICAL CASES

Monomorphic ve ntricula r ta chyca rdia

A Polymorphic ve ntricula r ta chyca rdia

B Tors a de s de pointe s

C Ve ntricula r fibrilla tion

D Atria l fibrilla tion with a Wolf-Pa rkins on-White a cce s s ory pa thway

E Antidromic AVRT Atria l flutte r with a be rra nt conduction

F Atria l fibrilla tion with a be rra nt conduction

G Figure 15-3. Examples of wi e complex tach ca ias.

229

230

CASE FILES: CAr d IO LOGy

than that from the AV node, resulting in a slurring of the QRS called a delta wave. When a patient with an accessory pathway develops an atrial arrhythmia such as atrial fibrillation or atrial flutter, it can appear as a WCT (Figure 15-3e). Accessory pathways can also result in AVRT. AVRT is classified as antidromic or orthodromic depending on the pathway that initially depolarizes the ventricle. Antidromic AVRT is a rhythm where a signal from the SA node travels down the accessory pathway into the ventricle, up the AV node, and back down the accessory pathway. Because the ventricle depolarization is being initiated via the accessory pathway, and not the AV node and His-Purkinje system, the QRS will be wide. Alternatively, in orthodromic AVRT the signal initially propagates normally through the AV node and down the His-Purkinje system, but returns through the accessory pathway and back down the AV node. This will produce a narrow complex tachycardia because the His-Purkinje system is involved in the initial ventricular depolarization. Another instance when a SVT can produce a WCT is in the presence of aberrancy. Patients with a bundle branch block who develop atrial flutter or atrial fibrillation will have WCT (Figure 15-3 f, g). In these cases, an ECG obtained prior to the tachycardia is very helpful. It is important to know that aberrant conduction can either develop or become more pronounced heart rate increases. For example, a patient with an incomplete right bundle branch block and a QRS of 100 at a heart rate of 70 bpm may have a complete bundle branch and a QRS of 130 at a rate of 120 bpm.

Diagnosis The diagnosis of VT can be difficult. Several algorithms have been developed to help differentiate VT from SVT, the most common of which is the Brugada criteria (Figure 15-4). This is a four-step decision tree in which if any of the criteria are met, the rhythm is VT. Overall, the algorithm’s sensitivity and specificity are 98.7% and 96.5%, respectively. Step 1 is to evaluate the morphology of the precordial leads (V1–V6) for concordance. If the QRS complex in all these leads is monophasic with the same polarity, or if there is a QS complex, they are said to be concordant. They are not concordant if there is a QR or RS complex in any of the leads. If concordance is present, the rhythm is VT; if not, continue to step 2. The next step is to evaluate the duration of the RS interval. If the RS interval is >100 ms in any of the precordial leads, then the rhythm is VT; if not, continue to the next step. Step 3 requires evaluating for evidence of AV dissociation. If P waves can be identified, and march out separately from the QRS complexes, there is AV dissociation. Also, if fusion beats are present, there is AV dissociation and the rhythm is VT. If there is no AV dissociation, then continue to step 4 and evaluate the morphology of the QRS complexes. In this step the QRS is first determined to be either a left bundle branch block (LBBB) morphology or a right bundle branch block (RBBB) morphology. Once this is determined, the criteria outlined in Figure 15-3 are applied, and if the QRS meets the criteria, the rhythm is VT; if not, it is an SVT.

Clinical Presentation Patients with a WCT can be in a relatively benign rhythm, like atrial flutter with aberrancy, or life-threatening rhythm, like VT or VF. Likewise, the presentation of

SECTIO N II: CLINICAL CASES

231

Brug ada c rite ria fo r ve ntric ular tac hyc ardia S te p 1 Is the re abs e nce of a RS or QR complex in a ll pre cordia l le a ds , ie, is the re concorda nce of the pre cordia l le a ds ? No

V1

V4

V2

V5

Ye s

VT

S te p 2 Is the RS inte rva l >100 ms in a ny pre cordia l le a d? No

V6

V3

> 100 mS e c

Ye s V1 VT

S te p 3 Is the re AV dis s ocia tion or fus ion be a ts (arrows indica te P wave s, a s te ris k a t the fus ion be a t)? No

*

Ye s

VT S te p 4 Are the morphology crite ria for VT me t? No

SVT

Ye s

VT

RBBB Mo rpho lo gy

LBBB Mo rpho lo gy

• Initia l R > 40 mS e c Le ad V1 o r V2

Le ad V6

> 40 mS e c

• Notche d S wave • Ons e t of the QRS to na dir or Q or S >60 mS e c • Pre s e nce of a Q or QS

Notch

• Monopha s ic R or QR • R ta lle r tha n R′

> 60 mS e c

• Pre s e nce of a RS complex

Figure 15-4. B uga a c ite ia fo the iagnosis of vent icula tach ca ia.

patients with a WCT can vary dramatically from stable to pulseless. The majority of patients who present with, or develop, a WCT will need immediate intervention to save their lives. These interventions, along with treatments for more stable patients, and prophylactic measures will be discussed in the next section. Here we discuss historical, exam, and laboratory information that is pertinent in the assessment of a WCT patient. In an unselected patient population presenting with a WCT, the rhythm will ultimately be determined to be VT in 80% of cases. This number increases to more than 90% in patients with a prior myocardial infarction. Therefore it is helpful to know whether the patient has a history of coronary artery disease. Patients with structural heart disease, particularly those with a decreased ejection fraction, are also more likely to have VT, making a history of congestive heart failure important. Other comorbidities, such as renal disease, can predispose to electrolyte abnormalities and subsequent arrhythmias. Recent medication changes can also cause electrolyte abnormalities, such as hypokalemia and hypomagnesiema from new or increased diuretic dose, or hyperkalemia from ACE inhibitors or aldosterone antagonists.

232

CASE FILES: CAr d IO LOGy

Long QT can be induced by new medications, including quinolone antibiotics (ciprofloxacin), macrolide antibiotics (erythromycin), antipsychotics, antiarrhythmic drugs (amiodarone, flecanide, sotalol), antiemetics (ondansetron, promethazine), and many others. Additionally, a history of WPW, atrial fibrillation, or atrial flutter may be helpful in determining the etiology of the patient’s WCT, but a history of these does not exclude VT. Symptoms experienced by a patient in a WCT are variable. Excluding those with cardiac arrest, symptoms can include palpitations, fatigue, presyncope, or syncope. People may also have angina, either as the provoking stimulus for their arrhythmia or from the increased myocardial oxygen demand in the setting of the tachycardia. The tachyarrhythmia can also result in heart failure and symptoms of shortness of breath, dyspnea on exertion, orthopnea, paroxysmal nocturnal dyspnea, weight gain, and lower extremity edema. The exam of a patient with WCT, depending on the situation, may be extremely cursory. However, despite limited time, certain obvious findings can be helpful. A sternotomy scar likely indicates prior cardiac surgery and probable coronary artery disease. Similarly, amputations (particularly lower extremity) are most commonly related to peripheral vascular disease, and greatly increase the likelihood of coronary artery disease. Finally, the presence of an arteriovenous fistula (generally on the upper extremity) indicates end-stage renal disease and hence a predisposition to electrolyte abnormalities. Lab work should be obtained on all patients with a WCT and should be evaluated for electrolyte abnormalities and cardiac ischemia. One should also consider checking thyroid studies.

Treatm ent The first priority for any patient in a WCT is to assess their clinical status. If there is no pulse, there should be immediate initiation of CPR along with advanced cardiovascular life support (ACLS) or basic life support (BLS) algorithms, including cardioversion. Even in the setting where a pulse is present, if there are signs of clinical instability–defined as: hypotension, altered mental status, angina, or heart failure–the patient requires immediate cardioversion. In both cases, cardioversion should be synchronized if possible, reserving unsynchronized cardioversion for VF. If possible, an ECG should be obtained and evaluated using the Brugada criteria. A prior ECG can be very helpful in evaluating for a history of atrial arrhythmia, preexcitation, or aberrant conduction. However, even in a stable patient with VT, urgent cardioversion is reasonable. The immediate medical treatment for VT generally consists in correcting any electrolyte abnormalities and administering IV antiarrhythmic drugs such as amiodarone or lidocaine. Amiodarone is effective for both VT and SVTs, and is administered 150 mg IV over 10 minutes followed by an infusion of 1 mg/min for 6 hours, then 0.5 mg/min. Lidocaine is particularly effective in treating VT related to cardiac ischemia and is administered as a bolus of 1–1.5 mg/kg over 2–3 minutes. Chronic medical therapy for VT should be performed with guidance of a specialist. For patients in whom VT reoccurs despite medical therapy, or in whom medical therapy is not tolerated, a catheter-based VT ablation can be considered.

SECTIO N II: CLINICAL CASES

233

Patients who survive an episode of VT or VF may warrant placement of an implantable cardioverter defibrillator (ICD). The indications for a secondary prevention ICD are (1) patients who are resuscitated from a VT/VF arrest in whom a completely reversible cause cannot be identified–patients with VT/VF limited to the first 48 hours after an acute myocardial infarction do not qualify for an ICD and (2) patients with spontaneous VT in the setting of hypertrophic, valvular, ischemic, or infiltrative heart disease. The indications for a primary prevention ICD are patients with (1) a prior myocardial infarction (>40 days prior) and an ejection fraction ≤30%; (2) nonischemic cardiomyopathy, NYHA class II or III symptoms, and an ejection fraction persistently ≤ 35% despite guideline-directed medical management for 3 months; (3) structural heart disease, syncope, and VT induced during an electrophysiology study; and (4) patients underlying disorders at high risk for VT/ VF such as congenital long QT or arrhythmogenic right ventricular cardiomyopathy. An ICD is contraindicated in patients with a life expectancy of less than 1 year, even if they otherwise meet ICD implantation criteria.

CASE CORRELATION • See also Case 13 (narrow complex tachycardias), and Case 14 (sudden cardiac death).

COMPREHENSION QUESTIONS 15.1 A 70-year-old woman with a history of Wolff-Parkinson-White (WPW) pattern noted on numerous prior ECGs presents to the emergency department with palpitations and near-syncope. Her ECG on presentation reveals a wide complex irregular tachycardia with a rate of 200 bpm. What is the most appropriate therapy for this patient? A. Metoprolol B. Diltiazem C. Digoxin D. Cardioversion E. Adenosine 15.2 A 25-year-old man presents from a group home for schizophrenic patients after an episode of syncope. His ECG on presentation reveals QT prolongation to 630 ms. Which of the following arrhythmias most likely caused his syncope? A. Monomorphic VT B. AV nodal reentrant tachycardia C. Atrial fibrillation with aberrancy D. Ventricular fibrillation E. Torsades de pointes (TdP)

234

CASE FILES: CAr d IO LOGy

15.3 An ICD is indicated in which of the following situations? A. A patient with ischemic cardiomyopathy, NYHA class II symptoms, and an ejection fraction of 45% B. A patient who experienced VF arrest at the time of a myocardial infarction, but has not had any arrhythmias since C. A patient with a nonischemic cardiomyopathy, on guideline-directed medical therapy for the last 3 months, whose ejection fraction has persistently been 30% D. A patient who developed QTc prolongation and TdP after starting ciprofloxacin, and whose QTc returned to normal once the medication was stopped

ANSWERS 15.1 D. Prompt cardioversion would be the best move here. This patient has atrial fibrillation in the setting of a known accessory pathway. The wide complex appearance is due to conduction down the accessory pathway into the ventricle, where it conducts cell to cell and produces a wide QRS. Cardioversion is required to abort conduction down the bypass tract, which is generally quite rapid. Sodium channel blockers can also do this. The remaining choices are all drugs that can block the AV node, which can be dangerous in atrial fibrillation with an accessory pathway because it can force all conduction down the accessory pathway and precipitate ventricular fibrillation. 15.2 E. A prolonged QTc predisposes to an R-on-T phenomenon and TdP. QT prolongation can be caused by a number of medications, and antipsychotic drugs are common offenders. Although all of the rhythm disorders listed can potentially cause syncope, the one most closely associated with prolonged QT intervals is TdP. 15.3 C. A patient with a nonischemic cardiomyopathy, on guideline-directed medical therapy for the last 3 months, whose ejection fraction has persistently been ≤35%, qualifies for an ICD. Patients with an ischemic cardiomyopathies do not qualify for ICDs unless their EFs are persistently ≤30% and they are on guideline-directed medical therapy for at least 3 months. Patients who experience VT/VF at the time of myocardial infarction are not candidates for an ICD unless the arrhythmia reoccurs >48 hours from the time of the infarction. An ICD is not indicated if a reversible cause can be identified.

SECTIO N II: CLINICAL CASES

235

CLINICAL PEARLS C

Most WCTs a e VT.

C

VT is mo e common in patients with a histo a ec ease ejection f action.

C

d iffe entiating VT f om SVT can be ifficult, an B uga a’s c ite ia can be helpful.

C

T eatment of WCTs often equi es eme gent inte vention, inclu ing ca iove sion.

C

The e a e gui elines outlining the in ications fo ICd use in p ima secon a p evention.

of m oca ial infa ction o

an

REFERENCE Epstein AE, DiMarco JP, Ellenbogen KA, et al. ACC/AHA/HRS 2008 Guidelines for Device-Based Therapy of Cardiac Rhythm Abnormalities: a report of the American College of Cardiology/American Heart Association Task Force on Practice Guidelines (Writing Committee to Revise the ACC/ AHA/NASPE 2002 Guideline Update for Implantation of Cardiac Pacemakers and Antiarrhythmia Devices): developed in collaboration with the American Association for Thoracic Surgery and Society of Thoracic Surgeons. Circulation. 2008;117(21):e350–e408.

This page intentionally left blank

CASE 16 A 54- ea -ol woman p esents to the eme enc epa tment complainin of seve al a s of p o essive sho tness of b eath an fati ue. She was in he usual state of health until 2 weeks a o, when she fell an inju e he left knee. As a esult, she ha been takin ibup ofen at home a oun the clock fo pain elief. Since then she has note a 10-lb wei ht ain, ab ominal bloatin , an swellin in he le s. He s mptoms have wo sene api l ove the past a . She was unable to lie comfo tabl in be an woke up seve al times at ni ht aspin fo ai . She is now sho t of b eath at est, unable to speak in full sentences, an has an unp ouctive cou h. On ph sical examination, the patient is afeb ile, tach pneic with a espi ato ate of 32 b eaths/ min, an he hea t ate is 110 bpm, bloo p essu e is 160/ 110 mmH , an ox en satu ation on oom ai is 82%. The e is elevate ju ula venous p essu e. Hea t exam is notable fo an au ible S3. Lun s have ales bilate all . She is usin accesso muscles to b eathe. Ab ominal exam eveals a pulsatile live . Ext emities a e wa m with oo pulses an pittin e ema bilate all to the knees. Labo ato evaluation eveals an elevate c eatinine an an elevate b ain nat iu etic pepti e level. An a te ial bloo as eveals h poxia an h pe capnia. A chest x- a shows ca iome al , iffuse bilate al infilt ates, an small bilate al pleu al effusions. C C C

What is the most likel ia nosis? What is the best next ia nostic step? What is the best next step in the ap ?

238

CASE FILES: CAr d IO LOg y

ANSWEr TO CASE 16: Acute Decompensated Heart Failure Summary: A 54-year-old woman presents to the ED with progressive respiratory distress, exertional dyspnea, orthopnea, and paroxysmal nocturnal dyspnea in the setting of heavy nonsteroidal anti-inflammatory drug (NSAID) use. She is tachycardiac, hypertensive, and hypoxemic. Her physical exam is notable for elevated jugular venous pressure, an audible S3, bilateral rales, a pulsatile liver, and lower extremity edema. Laboratory evaluation reveals acute kidney injury and elevated brain natriuretic peptide. Arterial blood gas reveals hypoxia and hypercapnia. A chest x-ray shows cardiomegaly, diffuse bilateral infiltrates, and small bilateral pleural effusions. • Most likely diagnosis: Acute decompensated heart failure. • Best diagnostic step: Determine hemodynamic profile and identify a precipitant of the acute decompensation. • Best therapy: Administer an intravenous diuretic and a vasodilator. Start noninvasive positive pressure ventilation.

ANALYSIS Objectives 1. Know the common presenting signs and symptoms of acute decompensated heart failure. 2. Know the common treatment goals for acute heart failure based on hemodynamic profile. 3. Understand the importance of a systematic search for precipitating causes of recent-onset or worsening heart failure.

Considerations This 54-year-old woman presents in acutely decompensated heart failure with respiratory distress, volume overload, and pulmonary edema. An important first step in evaluating a patient with suspected heart failure is to determine a patient’s hemodynamic profile based on volume status (wet or dry) and overall perfusion (warm or cold). This patient presents wet and warm, the most common hemodynamic profile in decompensated heart failure. She has no clear evidence of organ hypoperfusion, which could manifest as cool extremities, weak pulses, altered mental status, or liver abnormalities. The most immediate problems are severe congestion and hypoxia, which should be treated by volume reduction with diuretics, the administration of vasodilators, and noninvasive positive-pressure ventilation. Once measures to stabilize the patient’s clinical condition and provide symptom relief have been initiated, it is important to identify the specific factors that led to this particular acute decompensation. In this particular case, the likely culprit is heavy NSAID use. NSAIDs inhibit renal prostaglandins, and as a result can cause sodium and water retention

SECTIO N II: CLINICAL CASES

239

and diminish the effects of diuretics. It is critically important, however, to rule out other precipitating factors in the initial evaluation of the patient.

APPr OACH TO: Acute Decompensated Heart Failure DEFINITIONS HEART FAILU RE: A syndrome in which the heart cannot meet the metabolic demands of the body or fails to maintain adequate cardiac output in the setting of rising filling pressures. ACU TE DECOMPEN SATED HEART FAILU RE: An episode in which a patient’s heart failure signs and symptoms change, resulting in a need for urgent therapy or hospitalization. This is a possibly life-threatening condition that requires immediate medical attention. CARDIOMYOPATHY: A disease of the heart muscle that can result in heart failure. FLASH PU LMON ARY EDEMA: A feature of decompensated heart failure where acute increases in left ventricular diastolic pressure cause rapid fluid accumulation in the pulmonary interstitium and alveolar spaces.

CLINICAL APPROACH Etiologies In heart failure, the heart cannot meet the metabolic demands of the body and fails to maintain adequate cardiac output. There are many medical conditions that can adversely affect myocardial performance and promote heart failure (Table 16.1). Heart failure can be classified as heart failure with low EF (impaired systolic function) or heart failure with preserved EF (impaired diastolic function). Patients with heart failure with preserved EF tend to be older, female, and more likely to have atrial fibrillation and hypertension. In the United States, coronary artery disease is the most common cause of heart failure. Table 16-1 • ETIOLOGIES OF HEART FAILURE Ischemia hea t isease H pe tension g enetic (familial o he e ita ) d iabetes mellitus M oca itis Infilt ative isease (hemoch omatosis, am loi osis, sa coi osis) Substance abuse (alcohol, cocaine) Connective tissue isease Chemothe ap -in uce ca iotoxicit Valvula hea t isease Pe ica ial isease Pe ipa tum ca iom opath

240

CASE FILES: CAr d IO LOg y

Patients with systolic heart failure experience progressive remodeling of the left ventricle with dilatation and impaired contractility. Clinically, patients develop worsening symptoms, decreased exercise tolerance, and reduced quality of life. The progressive decline is characterized by increasingly frequent episodes of acute decompensation that require hospitalization. Acute heart failure typically arises as a result of deterioration in patients with a previous diagnosis of heart failure. However, acute heart failure may also be the first presentation of a heart failure syndrome. Acute heart failure is associated with the accumulation of fluid within the lungs’ interstitial and alveolar spaces, as a result of acutely elevated cardiac filling pressures.

Clinical Presentation Dyspnea is the most common presenting symptom of acutely decompensated heart failure. Other typical symptoms include orthopnea, paroxysmal nocturnal dyspnea, reduced exercise tolerance, fatigue, and ankle swelling. Patients may also describe a nocturnal cough, wheezing, weight gain, bloating, loss of appetite, and confusion. The clinician should attempt to quantify symptoms in order to gauge disease severity and monitor both the degree of acute decompensation and the response to treatment. A thorough physical exam can secure the diagnosis, and should focus on a careful determination of volume status. Signs of heart failure include hypoxia, pulmonary rales, pleural effusions, an accentuated third heart sound, a pulsatile liver, an elevated jugular venous pressure, sustained hepatojugular reflux, lower extremity edema, and abdominal ascites. Signs of low perfusion include cool skin, a narrow pulse pressure, and faint pulses. On the basis of presenting signs and symptoms, the clinician can determine a patient’s hemodynamic profile (Figure 16-1). This is an important step in evaluating a patient with suspected heart failure because it will guide treatment.

NO

YES

NO

Wa rm a nd Dry

Wa rm a nd We t

YES

Cold a nd Dry

Cold a nd We t

L

O

E

V

W

I

D

P

E

E

N

R

F

C

E

U

S

O

I

F

O

N

EVIDENCE OF CONGESTION

Figure 16-1. Hemo namic p ofiles in hea t failu e.

SECTIO N II: CLINICAL CASES

241

Table 16-2 • COMMON PRECIPITANTS OF ACUTE HEART FAILURE Acute m oca ial ischemia o infa ction Infection (pneumonia) A h thmias Uncont olle h pe tension d ieta in isc etion (excessive salt an flui intake) Me ication noncompliance Initiation of u s that inc ease salt etention (ste oi s, thiazoli ine iones, NSAId s) Pulmona embolus En oc ine abno malities (h pe th oi ism) Acute ca iovascula iso e s (en oca itis, m ope ica itis, ao tic issection)

Diagnosis The identification of a precipitant of acute heart failure (HF) is another important aspect of evaluating an acutely decompensated patient. Patients with preexisting HF often have a clear precipitant for an acute decompensation (Table 16.2). Medical and dietary noncompliance are common precipitants of acute decompensation. Coronary ischemia and uncontrolled hypertension can also contribute to the sudden rise in left-sided intracardiac pressures that can trigger episodes of flash pulmonary edema. NSAIDs, and calcium channel blockers, alcohol, and illicit drugs such as cocaine can also precipitate acute heart failure. In addition to a thorough history and physical exam, the following workup can facilitate the identification of a precipitant and rule out alternative causes for the presenting signs and symptoms such as chronic lung disease, anemia, or kidney failure. Laboratory evaluation should include complete metabolic panel, complete blood count, thyroid function, cardiac enzymes, and coagulation studies. Measurement of brain natriuretic peptide (BN P) can be useful in a patient presenting with suspected heart failure. A normal BNP in this setting can effectively rule out heart failure. Laboratory abnormalities identified on this initial evaluation can confirm end-organ hypoperfusion consistent with a low output state. All patients presenting with suspected acute decompensated heart failure should receive an electrocardiogram to identify signs of ischemia, old infarction, conduction abnormalities, arrhythmia, pericarditis, or chamber enlargement. A chest x-ray should be obtained to exclude lung disease and evaluate for cardiomegaly, diffuse bilateral infiltrates, focal consolidations, or pleural effusions. A repeat echocardiogram is not required for the evaluation of an acute episode in a patient with known HF. However, an echocardiogram is essential in cases of recent-onset heart failure, to evaluate cardiac structure and function. Newly diagnosed HF patients generally undergo coronary angiography to definitively exclude ischemic heart disease as the cause for their pump dysfunction. A pulmonary artery (right heart) catheterization can provide information about rightand left-sided filling pressures, pulmonary pressures, and cardiac output. In select cases of unexplained heart failure, endomyocardial biopsy can be helpful in providing a diagnosis and guiding management.

242

CASE FILES: CAr d IO LOg y

Treatm ent The immediate goals of treatment of acute decompensated heart failure are to provide rapid symptom relief, improve organ perfusion, and return the patient to a warm and dry hemodynamic profile. Therapy should also be directed at the underlying precipitating cause of the acute decompensation of heart failure, if it is known and can be treated. Specific therapy should be guided by the patient’s hemodynamic profile. Most acutely decompensated heart failure patients are warm and wet, with elevated filling pressures and volume overload. In these cases, administration of an intravenous loop diuretic is the mainstay of therapy and can provide rapid symptom relief with the removal of fluid and reducing central venous and pulmonary capillary wedge pressures. Patients often need a higher dose of loop diuretics in the acute setting. Studies have shown that a single intravenous bolus regimen and a continuous infusion of diuretic have similar efficacy in acutely decompensated heart failure. In cases where substantial congestion persists, severe kidney injury exists, and adequate diuresis cannot be achieved, ultrafiltration or dialysis may be considered. Several randomized trials have examined the efficacy of ultrafiltration in the setting of acutely decompensated heart failure, and ultrafiltration may be indicated for patients with obvious volume overload who are not responding to medical therapy. Supplemental oxygen therapy should be provided as needed to treat hypoxia. For patients in respiratory distress or persistent hypoxia, noninvasive positive-pressure ventilation (NPPV) should be utilized. In the absence of contraindications, the use of NPPV in patients with cardiac pulmonary edema can decrease the need for intubation and improve dyspnea, hypoxia, hypercapnia, and heart rate. Morphine can also provide symptomatic relief in cases of acute pulmonary edema Vasodilators such as nitroglycerin or nitroprusside can be useful if blood pressure tolerates them. These medications reduce preload and afterload, and can provide rapid symptom relief in patients with acute pulmonary edema or severe hypertension when used in combination with diuretics. It is very important to identify those patients who have a cold and wet hemodynamic profile, since they will require inotropes and vasodilators in a monitored, critical care setting. Beta-blockers should not be started in acute decompensated heart failure. In patients already on beta-blockers, beta-blocker dosages should be reduced or entirely discontinued until the patient becomes clinically euvolemic and hemodynamically stable. Accurate weight, fluid intake, urine output, volume status, electrolyte levels, and vitals should be followed closely once treatment has been initiated. After stabilization of the patient, low doses of ACE inhibitors should be introduced with careful attention to creatinine, blood pressure, and serum electrolytes.

CASE CORRELATION • See also Case 1 (acute coronary syndrome/STEMI), Case 2 (acute coronary syndrome/NSTEMI), and Case 3 (cardiogenic shock).

SECTIO N II: CLINICAL CASES

243

COMPREHENSION QUESTIONS 16.1 A 55-year-old man complains of increasing dyspnea on exertion and orthopnea. His physical examination reveals an S3 heart sound, pulmonary rales, jugular venous distension, and lower extremity edema. He is normotensive, and his extremities are warm to touch. An echocardiogram confirms an ejection fraction of 25% and a dilated left ventricle. What is the most appropriate next step in management? A. Oral furosemide B. Intravenous furosemide C. Oral beta blocker D. Sublingual nitroglycerin 16.2 Signs of low perfusion on physical exam include all of the following except A. Cool skin B. Narrow pulse pressure C. Presence of a third heart sound D. Faint pulses 16.3 Which of the following medications should not be initiated during an episode of acute decompensated heart failure? A. Beta-blocker B. Ace inhibitor C. Potassium supplementation D. Digoxin

ANSWERS 16.1 B. Intravenous furosemide is indicated in this patient with warm/wet profile decompensated heart failure. Intravenous dosing results in more rapid drug delivery and effect than oral dosing. Beta-blocker initiation or titration is not advisable in patients with acutely decompensated heart failure. Nitroglycerin may cause some venodilatation, which may reduce preload, but it will not affect this patient’s volume overload. 16.2 C. A third heart sound is a marker of left ventricular volume overload. Cool extremities, narrow pulse pressure, and faint pulses are markers of poor distal perfusion. 16.3 A. Chronic beta-blocker use is associated with significantly improved survival in patients with reduced LV systolic function. However, acutely their negative inotropic effects can worsen stroke volume and therefore worsen heart failure symptoms. Beta-blocker initiation should be deferred until the patient is euvolemic and well compensated on exam after treatment with appropriate diuresis and afterload reduction.

244

CASE FILES: CAr d IO LOg y

CLINICAL PEARLS C

The vast majo it of acute hea t failu e exace bations have p ecipitatin facto s. It is impo tant to i entif the eason fo ecompensation.

C

d ete mination of the hemo namic p ofile (volume an pe fusion status) of an acutel ecompensate HF patient is a helpful fi st step in mana ement.

C

A tho ou h ph sical exam can secu e a hea t failu e.

C

Int avenous iu etics a e the mainsta of the api t eatment fo acute ecompensate hea t failu e.

ia nosis of ecompensate

REFERENCES Bart BA, Boyle A, Bank AJ, et al. Ultrafiltration versus usual care for hospitalized patients with heart failure: the relief from acutely fluid-overloaded patients with decompensated congested heart failure (RAPID-CHF) trial. J A m Coll Cardiol. 2005; 46:2043. Bart BA, Goldsmith SR, Lee KL, et al. Ultrafiltration in decompensated heart failure with cardiorenal syndrome. N Engl J Med. 2012;367:2296. Costanzo MR, Guglin ME, Saltzberg MT, et al. Ultrafiltration versus intravenous diuretics for patients hospitalized for acute decompensated heart failure. J A m Coll Cardiol. 2007;49:675. Felker GM, Lee KL, Bull DA, et al. Diuretic strategies in patients with acute decompensated heart failure. N Engl J Med. 2011;364(8):797–805. Lindenfeld J, Albert NM, Boehmer JP, et al. HFSA 2010 Comprehensive Heart Failure Practice Guideline. Heart Failure Society of America. J Card Fail. 2010;16(6):e1–e194. McMurray JJ, Anker SD, Auricchio A, et. al., ESC guidelines for the diagnosis and treatment of acute and chronic heart failure 2012: The Task Force for the Diagnosis and Treatment of Acute and Chronic Heart Failure 2012 of the European Society of Cardiology. Developed in collaboration with the Heart Failure Association (HFA) of the ESC. Eur J Heart Fail. 2012;14(8):803–869. Weng CL, Zhao YT, Liu QH, et al. Meta-analysis: noninvasive ventilation in acute cardiogenic pulmonary edema. A nn Intern Med. 2010;152(9):590. Yancy CW, Jessup M, Bozkurt B, et al. 2013 ACCF/AHA guideline for the management of heart failure: a report of the American College of Cardiology Foundation/American Heart Association Task Force on Practice Guidelines. Circulation. 2013;128(16):1810–1852.

CASE 17 A 50- ear- ld w man asks t see her cardi l ist 3 m nths pri r t her scheduled clinic app intment because she “isn’t d in well.” Three ears earlier she was dia n sed with an idi pathic n nischemic cardi m path [ejecti n fracti n (EF) 15%]. Since then her medicati n re imen has ev lved t include a beta-bl cker, an ACE inhibit r, an ald ster ne anta nist, and a diuretic. Tw ears a she received an implantable cardi verter defibrillat r and cardiac res nchr nizati n therap defibrillat r (CRT-D). Initiall she felt better and returned t her j b as a cashier, which required her t stand and take the stairs n ccasi n. H wever, 3 m nths a she rep rted frequent li htheadedness and was n ted t have a l w bl d pressure f 90/ 60 mmH with a heart rate f 70 bpm durin a re ular scheduled visit. Her ACE inhibit r d se was reduced, but despite the chan e in medicati n, she c ntinued t feel unwell. The patient n w experiences sh rtness f breath and fati ue n m st da s. She n l n er had the ener t perf rm the re ular activities f dail livin such as sh werin and dressin with ut needin t rest. Because f these s mpt ms, she quit her j b as a cashier and ver the past m nth has spent m st da s l in n the c uch. In the cardi l ists’ ffice she is n ted a ain t have a bl d pressure f 90/ 60 mmH and a heart rate f 110 bpm. Her ju ular ven us pressure (JVP) is elevated t the an le f the jaw. Her peripheral extremities are c l t the t uch. C C C

What is the m st likel dia n sis? What is the best next dia n stic step? What is the best next step in therap ?

246

CASE FILES: CARDIo Lo g y

ANSWER To CASE 17: Advanced Heart Failure and Transplantation Summary: A 50-year-old woman with a 3-year history of nonischemic cardiomyopathy (EF 15%) has received appropriate guideline-based therapy for advanced heart failure. Her functional status has recently deteriorated, and she is no longer able to carry out her activities of daily living without symptoms of fatigue, shortness of breath, and presyncope. On physical examination she is noted to have low blood pressure, an elevated JVP, and cool extremities in keeping with low cardiac output (cardiogenic shock). On the basis of her symptom chronology, her low-output state has likely been subacute to chronic. • Most likely diagnosis: Advanced cardiomyopathy, end-stage heart failure. • N ext diagnostic step: Hemodynamic evaluation–right-heart catheterization. • N ext step in therapy: Initiate evaluation for advanced therapies, including cardiac transplant and/or mechanical circulatory support.

ANALYSIS Objectives 1. Recognize the spectrum of patients with chronic heart failure and apply the American College of Cardiology (ACC) staging classification to identify patients with advanced or end-stage heart failure. 2. Understand the definition of cardiogenic shock and the physical exam findings consistent with this clinical state. 3. Understand the importance of medical therapy in the treatment of chronic heart failure and know the criteria for ICD and CRT implantation. 4. Become familiar with the process involved in an advanced therapy workup when a patient is considered for cardiac transplant and/or mechanical circulatory support.

Considerations This is a 50-year-old woman with a 3-year history of nonischemic cardiomyopathy. She has a persistent low EF (15%) despite optimal medical and pacing therapy, and now presents with deterioration in her functional status over a 3-month period. The first priority should be to recognize the “red flags” associated with her clinical presentation as a sign of end-stage heart failure. These include symptomatic low blood pressure, medication intolerance, and most importantly a functional decline from NYHA II symptoms to NYHA class III/IV symptoms (Table17-1). It is important to highlight the fact that her low EF of 15% is less relevant here than her clinical symptoms. It is a common misconception that if symptoms worsen, the EF must also deteriorate; however, this is not always true. Her EF has not changed in 3 years and cannot explain the whole picture. The syndrome of

SECTIo N II: CLINICAL CASES

247

Table 17-1 • CLASSIFICATION AND STAGING OF HEART FAILURE Heart Failure Stage (ACC/ AHA) (Reference)

NYHA Stage (Reference)

A

Risk fact rs f r heart failure are present, such as h pertensi n, c r nar arter disease, and diabetes; h wever, n structural heart disease is identified

N applicable equivalent

B

Structural heart disease is identified, but n heart failure s mpt ms are present

1

As mpt matic

C

Structural heart disease is identified and heart failure s mpt ms are currentl present, r have been present in the past

2

S mpt ms with m derate exerti n (e , climbin ne fli ht f stairs) S mpt ms with minimal exerti n (e , perf rmin activities f dail livin )

D

3

Structural heart disease is identified and s mp- 4 t ms f heart failure are refract r t treatment and require specialized interventi ns

S mpt ms at rest

Abbreviations: ACC, American C lle e f Cardi l ; AHA, American Heart Ass ciati n; NyHA, New y rk Heart Ass ciati n. (Data from American College of Cardiology.)

advanced heart failure is far more complex than what is reflected in a single variable of ejection fraction. The patient is noted to have a low systemic blood pressure, resting tachycardia, raised JVP, and cool peripheries, in keeping with a clinical picture of cardiogenic shock. There are four hemodynamic profiles of heart failure pertaining to the cardiac physical examination. This patient’s physical examination is consistent with an elevated filling pressure (WET) and low cardiac output (COLD) (Figure 17-1). Before this patient’s heart failure progressed toward an end stage, she received appropriate guideline-based medical therapy. One can refer to the original landmark

Volume S ta tus

m r a W

i m

o

e

i

r

s

xt e , e

u

Cold a nd We t *

t

f

Cold a nd Dry

a

r

Wa rm a nd We t

t

e

Wa rm a nd Dry

C

o

l

d

L

a

P

We t

c

n

t

i

e

s

Dry

J VP, P CWP

Figure 17-1. Characterizati n f heart failure based n ph sical examinati n; patients ma be classified acc rdin t their v lume status (wet r dr ) and perfusi n status (warm r c ld) usin examinati n findin s and ther clinical t ls (JVP, ju ular ven us pressure; PCWP, pulm nar capillar wed e pressure). The patient presented in this chapter falls int the c ld and wet cate r ( ).

248

CASE FILES: CARDIo Lo g y

Table 17-2 • MEDICAL THERAPY IN HEART FAILURE–LANDMARK CLINICAL TRIALS* Beta-bl cker

US Carvedil l Heart Failure Stud , Co PERNICUS, CIBIS II, MERIT-HF

ACE inhibit r/ an i tensin recept r bl cker

Co NSENSUS, So LV-D, Val-HeFT, CHARM

Ald ster ne anta

RALES, EMPHASIS-HF

nist

Examples f s me ri inal landmark clinical trials supp rtin the use f beta bl ckade, ACE inhibiti n, an i tensin recept r bl ckers, and ald ster ne anta nists in heart failure are rep rted in the f ll win literature s urces; this list is n t intended t be c mprehensive:

Packer M, Coats AJ, Fowler MB, et al. Effect of carvedilol on survival in severe chronic heart failure. N Engl J Med. 2001;344(22):1651–1658. Packer M, Bristow MR, Cohn JN, et al. The effect of carvedilol on morbidity and mortality in patients with chronic heart failure. U.S. Carvedilol Heart Failure Study Group. N Engl J Med. 1996;334(21): 1349–1355. The Cardiac Insufficiency Bisoprolol Study II (CIBIS-II): a randomised trial. Lancet. 1999;353(9146): 9–13. Effect of metoprolol CR/XL in chronic heart failure: Metoprolol CR/XL Randomised Intervention Trial in Congestive Heart Failure (MERIT-HF). Lancet. 1999;353(9169):2001–2007. The CONSENSUS Trial Study Group. Effects of enalapril on mortality in severe congestive heart failure. Results of the Cooperative North Scandinavian enalapril survival study (CONSENSUS). N Engl J Med. 1987;316:1429–1435. The SOLVD Investigators. Effect of enalapril on survival in patients with reduced left ventricular ejection fractions and congestive heart failure. N Engl J Med. 1991;325(5):293–302. Pfeffer MA, McMurray JJ, Velazquez EJ, et al. Valsartan in Acute Myocardial Infarction Trial Investigators. Valsartan, captopril, or both in myocardial infarction complicated by heart failure, left ventricular dysfunction, or both. N Engl J Med. 2003;349(20):1893–1906. Cohn JN, Tognoni G for the Valsartan Heart Failure Trial Investigators. A randomized trial of the angiotensin-receptor blocker valsartan in chronic heart failure. N Engl J Med. 2001;345:1667–1675. Young JB, Dunlap ME, Pfeffer MA, et al. for the Candesartan in Heart failure Assessment of Reduction in Mortality and morbidity (CHARM) Investigators and Committees. Mortality and morbidity reduction with candesartan in patients with chronic heart failure and left ventricular systolic dysfunction. Results of the CHARM low-left ventricular ejection fraction trials. Circulation. 2004;110:2618–2626. Pitt B, Zannad F, Remme WJ, Cody R, et al. The effect of spironolactone on morbidity and mortality in patients with severe heart failure. Randomized Aldactone Evaluation Study Investigators. N Engl J Med. 1999;341(10):709–717. Zannad F, McMurray JJV, Krum H, for the EMPHASIS-HF Study Group. Eplerenone in patients with systolic heart failure and mild symptoms N Engl J Med. 2011;364:11–21.

trials supporting the use of beta blockade, ACE/ARB inhibition, and aldosterone antagonists in the treatment of heart failure (Table 17-2). The indications for ICD and CRT continue to evolve. Appropriate implantation of an ICD or CRT device requires an integration of data including ejection fraction, functional class, and time elapsed since diagnosis and the presence of guideline-based medical therapy. She should be referred for an advanced therapy workup, including a transplant and/or left ventricular assist device (LVAD). This workup involves a multidisciplinary team who will assess the patient’s clinical and psychosocial background to

SECTIo N II: CLINICAL CASES

249

assess suitability. In addition, patients and their families require education about the ethical and legal implications of these therapies.

APPRo ACH To : Advanced Heart Failure and Transplantation DEFINITIONS N ON ISCHEMIC CARDIOMYOPATHY: Cardiomyopathy in the presence of normal or minimally narrowed coronary arteries. This term is generally used when a diagnostic coronary angiogram has ruled out the presence of obstructive coronary artery disease. EJECTION FRACTION : A quantitative measurement of the volume of blood ejected from the left ventricle with each contraction compared to the resting volume of the left ventricle at end diastole. This value can be determined using a number of imaging modalities such as echocardiography, ventriculography, and nuclear imaging. CARDIOGEN IC SHOCK: Inadequate end-organ perfusion due to a failing ventricle (right, left, or both) in the setting of adequate filling pressures. CARDIAC RESYN CHRON IZATION THERAPY (CRT): A small pacemakerlike device that sends impulses to both ventricles to coordinate biventricular contraction in the presence of asynchronous contraction due to bundle branch block. MECHAN ICAL CIRCU LATORY SU PPORT: The use of a mechanical/electrical heart pump to provide additional flow to a failing ventricle. Commonly used as a synonym for a long-term VAD (ventricular assist device), but the term is also used in reference to short-term extracorporeal support such as ECMO (extracorporeal membrane oxygenation).

CLINICAL APPROACH When a patient with chronic heart failure presents to your office or emergency room, the clinical approach involves multiple steps: confirm the diagnosis, look for the etiology, characterize baseline status, identify severity, and determine prognosis and possible need for advanced therapies (Table 17-3).

COMMON ETIOLOGIES Chronic heart failure occurs when cardiac output is inadequate to meet the body’s end-organ needs over a sustained period of time. A number of structural abnormalities involving the coronary arteries, valves, pericardium, and myocardium can lead to this persistent abnormal clinical state. Congenital and genetic abnormalities affecting any of these structures can occur and should be sought as a potential etiology in the correct clinical context.

250

CASE FILES: CARDIo Lo g y

Table 17-3 • STEPWISE APPROACH TO EVALUATION OF PATIENTS WITH CHRONIC HEART FAILURE Step 1

Confirm the diagnosis. The clinical hist r must be inte rated with the ph sical exam and ther tests (ie, ECg , ECHo , BNP). N ne f these fact rs al ne are sensitive r specific en u h t make the dia n sis f heart failure, and err rs can ccur at this step. o ther mimics f c n estive heart failure include pulm nar vascular r parench mal disease.

Step 2

Answer the “WHY?” (ie, determine the eti l

Step 3

Characterize the patient’s baseline status. The patient’s clinical s ndr me can be punctuated b numer us acute heart failure exacerbati ns. Durin these acute exacerbati ns, his/ her functi nal status can be markedl different fr m baseline. In rder t pr perl characterize functi nal status at baseline, the discernin clinician must tease this ut durin the hist r . Better clarificati n f a patient’s baseline status can help with steps 4 and 5.

Step 4

Determine the severity of the chronic heart failure syndrome. Patients sh uld be n ptimal medical mana ement pri r t this determinati n.

Step 5

Determine prognosis and need for advanced therapies. This step can be the m st challen in and ften requires rec niti n f clinical “red fla s” and inte rati n with an understandin f the patient’s heart failure traject r ver a specific timeframe.

f r the underl in cardi m

path ).

Acquired abnormalities due to longstanding hypertension, coronary artery disease, valvular regurgitation or stenosis, infection, inflammation, infiltration, druginduced toxicity (cocaine, chemotherapy, alcohol), and arrhythmias are more common. The syndrome of heart failure can occur in the setting of systolic dysfunction (heart failure with low ejection fraction) or diastolic dysfunction [heart failure with preserved ejection fraction (HFpEF)].

Clinical Presentation Most patients with chronic heart failure present with a number of symptoms that can be elicited from a detailed history and physical exam. Exertional dyspnea and/or exercise intolerance is most common. The functional limitation associated with the dyspnea must be determined to better classify the patient’s symptoms. Keeping the ACC/NYHA classification scheme in mind, focused questions such as “Can you climb a flight of stairs?” or “Do you need to stop to catch your breath while bathing yourself?” can further elucidate this limitation. Advanced heart failure patients often complain of volume retention, which can take the form of peripheral edema or ascites despite high-dose diuretics. Orthopnea and paroxysmal nocturnal dyspnea are symptoms that patients rarely volunteer on their own and should be specifically elicited by the physician. Presyncope, anorexia, unexplained weight loss, persistent nausea, and right upper quadrant tenderness are symptoms of congestion and low cardiac output. These symptoms tend to occur more frequently in advanced heart failure and are markers of poor prognosis. Additional poor prognostic markers include frequent hospitalizations despite aggressive fluid management and close medical follow-up. Although patients’ symptoms can change over time (particularly if they are experiencing an exacerbation), a patient with advanced disease spends most days in the functional class III/IV range. To get a sense of chronology and better characterize

SECTIo N II: CLINICAL CASES

251

baseline status (Table 17-3, step 3), questions such as “When was the last time you felt you were at your best?”, “How long have you been experiencing symptom X?”, or “How many days in the week do you experience symptom X?” should be asked. Questions such as these can help the clinician differentiate deterioration due to an acute exacerbation from a more chronic deterioration due to progression of the underlying disease. The physical examination should focus on the vital signs, volume status, and perfusion status. Many patients with advanced heart failure will have atrial fibrillation. Symptomatic hypotension and tachycardia are worrisome signs, reflecting the body’s attempt to maintain cardiac output in the setting of low stroke volume. The best physical examination surrogate for evaluating intravascular volume status is the jugular venous pressure (JVP). When the JVP is elevated, one can fairly accurately conclude that the patient is congested. Peripheral edema, in addition to an elevated JVP, points to heart failure as an etiology for the edema. Edema due to other causes such as venous insufficiency or lymphedema would have a normal JVP. Unlike the lungs in acute pulmonary edema, the lungs in chronic advanced heart may be clear on auscultation, due to a pulmonary vascular system that has adapted to chronically elevated left-sided filling pressures over time. An S3 may be expected, particularly in a severely dilated ventricle and/or in a decompensated state. An S4 may point to hypertension as the etiology of a stiff ventricle. Accompanied murmurs of mitral regurgitation are common. A pulsatile or tender liver and elevated transaminases can occur as a result of congestion. Laboratory evidence of poor cardiac output includes end-organ dysfunction such as elevated creatinine and lactic acidosis. Chronic hyponatremia is commonly seen in advanced heart failure due to elevation in antidiuretic hormone (ADH) in response to low cardiac output.

Medical and Device Treatment Beta blockade and ACE inhibition are the mainstay of medical therapy in chronic heart failure. Both classes of medication have been proven in large randomized control trials to increase survival in heart failure. The use of aldosterone antagonists was originally reserved for patients with advanced heart failure only, but is increasingly recommended in mild to moderate heart failure patients. The guidelines regarding ICD and CRT implantation continue to evolve over time and are quite complex. ICD therapy is recommended for primary prevention of sudden cardiac death in patients who are symptomatic with an EF < 30–35% despite good medical therapy. This recommendation is independent of the etiology of left ventricular systolic dysfunction. CRT is also indicated in these patients if they have a wide LBBB (QRS duration >150 ms) on ECG.

Patient Selection for Advanced Therapies If a patient continues to experience refractory heart failure symptoms despite optimal medical and CRT therapy (if applicable), advanced therapies such as cardiac transplantation and mechanical circulatory support should be considered. The need for alternative therapies to transplant is increasingly necessary, as the demand for donor hearts has outstripped supply. LVAD technology has become a welcome alternative to bridge patients safely to transplant. LVADs can also offer permanent

252

CASE FILES: CARDIo Lo g y

support for patients who are not eligible for cardiac transplantation because of age, comorbidities, or other contraindications to transplant. Although a definitive age cutoff does not exist, regarding cardiac transplantation age >70 years represents the upper limit of candidacy among most transplant programs worldwide. The population typically considered for advanced therapies includes individuals with ACC stage D, NYHA class III/IV symptoms. These are patients whose heart failure is refractory to best medical therapy with recurrent hospitalizations and/or in need of inotrope therapy. Objective measurements to guide candidacy for advanced therapies can also include a VO 2,max from a cardiopulmonary stress test or direct measurement of cardiac output from a right-heart catheterization. Some absolute contraindications to cardiac transplant include malignancy within the last 5 years, severe pulmonary hypertension, active infection, extreme BMI, and psychosocial contraindications. Patient selection for LVAD continues to be a moving target as the field evolves. The reader is referred to the most recent guidelines published by the International Society of Heart and Lung Transplant (ISHLT) in this regard. LVADs were initially implanted in the sickest patient population such as those in acute refractory cardiogenic shock. However, there has been increasing recognition that implanting the LVAD before that critical period is more favorable. The decision to pursue advanced therapies is complex and requires an integration of a number of variables. Any transplant or LVAD program is made up of a multidisciplinary committee, including heart failure cardiologists, transplant/VAD surgeons, psychiatrists, psychologists, social workers, and specialized transplant and VAD nurses. The decision to pursue advanced therapies is often reached by consensus after careful evaluation of all of the available data and therapeutic options.

Com plications Related to Advanced Therapies Survival after transplant beyond the perioperative period is dependent on medication compliance to prevent acute and chronic rejection. Cardiac biopsies are completed routinely in the first year to fine-tune immunosuppression to a level that prevents rejection but also does not allow for significant infection. This fine balance between rejection and infection is a consistent challenge in the posttransplant course. Long-term complications such as transplant vascular disease and malignancy can threaten the long-term integrity of the new heart. Patients with a ventricular assist device require long-term anticoagulation to prevent pump thrombosis and formation of emboli. The need for chronic anticoagulation brings with it a host of complications, including GI bleeding, hemorrhagic stroke, and anemia. Ischemic stroke, chronic hemolysis, risks of device infection, and pump thrombosis remain the drawback of chronic VAD management.

CASE CORRELATION • See also Case 16 (acute heart failure).

SECTIo N II: CLINICAL CASES

253

COMPREHENSION QUESTIONS 17.1 Match the following hemodynamic profiles with the appropriate heart failure profile: 1. COLD AND DRY

A. Congested and not perfusing adequately

2. WARM AND WET

B. Not congested but not perfusing adequately

3. WARM AND DRY

C. Congested but adequately perfused

4. COLD AND WET

D. Not congested and perfusing adequately

17.2 A 34-year-old patient was diagnosed with a nonischemic cardiomyopathy at age 30 years. His previous cardiologist started him on a low-dose beta-blocker, but the patient never saw him again. He presents to your office with NYHA class II symptoms. Echocardiography confirms an EF of 20%. Your next step in management is to A. Refer him for ICD implantation and a CRT device if his QRS > 150 B. Perform a right-heart catheterization C. Increase his beta-blocker dose and plan to initiate an ACE inhibitor D. Refer him for advanced therapies workup 17.3 A patient presents to your office with a 2-year history of ankle swelling that is worse at night than in the morning. He complains of shortness of breath while walking up a hill. On examination you note that he is morbidly obese and has varicose veins. His JVP is 2 cm above the sternal angle. The most likely etiology of his ankle edema is A. Alcoholic cirrhosis B. Lymphedema C. Venous insufficiency D. Congestive heart failure

ANSWERS 17.1 1B, 2C, 3D, 4A. 17.2 C. This 34-year-old patient has a nonischemic cardiomyopathy. Although it is concerning that his EF remains 20%, he has not been on optimal medical therapy. Before he is referred for more invasive therapies, his medical regimen should be uptitrated. It is possible that on good medical therapy he may experience some left ventricular recovery and/or improvement in his symptoms. This is your first time meeting him, and it is too early to consider advanced therapies. This type of decision should be made over time once you get to know this patient and see how he responds first to medical therapy, and then to device therapy if needed.

254

CASE FILES: CARDIo Lo g y

17.3 C. This patient’s shortness of breath is likely accounted for by his morbid obesity. Because his swelling (edema) is less in the morning than at night, and as he has varicose veins, venous insufficiency is more likely. The clincher here is the presence of a normal JVP, further solidifying the likelihood that his edema is related to venous insufficiency and not heart failure.

CLINICAL PEARLS C

Left ventricular ejecti n fracti n is nl a min r piece f the chr nic heart failure s ndr me. Man patients with chr nic heart failure have a preserved ejecti n fracti n.

C

The traject r f heart failure is difficult t predict. Advanced r endsta e heart failure can be better rec nized with l n -term patient f ll w-up and rec niti n f p r pr n stic s mpt ms and si ns such as frequent h spitalizati ns and refract r s mpt ms despite cl se medical mana ement.

C

Timin and patient selecti n are a ke part f the advanced therapies evaluati n. Earl referral t an advanced heart failure specialist ensures that this timin is n t missed.

C

Determinin a patient’s candidac f r advanced therapies (transplant r LVAD) is c mplex and requires the inte rati n f a number f clinical and hist rical variables as evaluated b a multidisciplinar advanced therapeutics c mmittee.

REFERENCES Feldman D, Pamboukian S, Teuteberg J. The 2013 International Society for Heart and Lung Transplantation Guidelines for mechanical circulatory support: executive summary. J Heart Lung Transplant. 2013;32(2):156–186. Mehra MR, Kobashigawa J, Starling R et al: Listing criteria for heart transplantation: International Society for Heart and Lung Transplantation Guidelines for the Care of Cardiac Transplant Candidates. J Heart Lung Transplant. 2006;25(9);1024–1042. Nohria A, Tsang S, Fang JC, et al. Clinical assessment identifies hemodynamic profiles that predict outcomes in patients admitted with heart failure. J A m Coll Cardiol. 2003;41(10):1797–1804. Tracy CM, Darabr D, Dunbar B, et al: 2012 ACCF/AHA/HRS Focused Update of the 2008 Guidelines for Device-Based Therapy of Cardiac Rhythm Abnormalities. A report of the American College of Cardiology Foundation/American Heart Association Task Force on Practice Guidelines. 2012 ( available online at http://www.google.ca/#fp=d8d10c607a15f311&q=updated+CRT+guidelines+2012). Yancy CW, Jessup M, Bozkurt B, et al. 2013 ACCF/AHA Guidelines for the Management of Heart Failure: A report of the American College of Cardiology Foundation/American Heart Association Task Force on Practice Guidelines. J A m Coll Cardiol. 2013;62(16):e147–e239.

CASE 18 A 63- ear-o man comes to visit ou because of mu tip e recent hospita a missions for shortness of breath an recurrent p eura effusions necessitating thoracenteses. The patient escribes 3 months of worsening spnea on exertion an frequent pa pitations. He cannot c imb one f ight of stairs because of spnea an s eeps on three pi ows because he becomes spneic whi e ing f at. On review of s stems (RoS) he reports frequent ighthea e ness, an unintentiona 15- b weight oss, an oss of sensation in his feet with a “pins an nee es” fee ing. He enies chest pain or s ncope. He has no known car iac isease an previous consi ere himse f hea th . He is a ife ong nonsmoker who rinks two g asses of wine each week with inner. He has no fami histor of coronar heart isease, but his father ha atria fibri ation an eve ope heart fai ure in his ate 50s. On examination the patient is non istresse an thin. Vita signs are as fo ows: b oo pressure 108/ 64 mmHg (right arm, sitting), 106/ 64 mmHg ( eft arm, sitting), an 92/ 58 mmHg (right arm, stan ing); caroti pu se was 63 bpm an regu ar; bo mass in ex (BMI) 21 kg/ m 2; an jugu ar venous pressure (JVP) is triphasic an 14 cm H 2O. He has a arge tongue an sma bruises near his e e i s. There is no th romega or pa pab e mpha enopath . Car iac examination revea s a atera isp ace an iffuse point of maxima impu se (PMI). There is an S4 ga op without murmurs on auscu tation. He has u ness to percussion an iminishe breath soun s on auscu tation at the ung bases. The ab omina examination was norma . A remaining pu ses were brisk an s mmetric. His extremities are coo an nonc anotic with pitting e ema to his mi shin. He has iminishe sensation to pinprick on his toes an fingertips. He brings an e ectrocar iogram (ECG) from his previous hospita a mission (Figure 18-1). C C C

What is the most ike iagnosis? What is the best next iagnostic step? What is the best treatment for this patient?

I

a VR

V1

V4

II

a VL

V2

V5

III

a VF

V3

V6

V1

II

V5 Figure 18-1. ECG for main subject of this case.

SECTIO N II: Cl INICAl CASES

257

ANSWER TO CASE 18: Cardiomyopathies Summary: A 63-year-old man is currently being seen in your office for evaluation a history of recurrent pleural effusions, weight loss, palpitations, and neuropathy. He notes that his father had a history of atrial fibrillation and heart failure in his 50s. He explains to you that he is having exertional dyspnea and orthopnea. He denies any angina or syncope but does admit to some positional lightheadedness. He does not smoke and rarely drinks alcohol. He appears thin and has some bruising around his eyelids and a large tongue. His jugular venous pressure is elevated, his PMI is laterally displaced and diffuse, and he has an S4 gallop. He has dullness to percussion and diminished breath sounds at his lung bases with peripheral edema and diminished sensation on neurological examination. His ECG is notable for low limb voltages, atrial enlargement, and left ventricular hypertrophy. • Most likely diagnosis: Amyloidosis with cardiac involvement, possible familial. • N ext diagnostic step: Transthoracic echocardiography. • Best treatment: Start the patient on loop diuretic.

ANALYSIS Objectives 1. Know the breadth and diversity of cardiomyopathy pathology. 2. Comprehend the management of and treatment decisions for the various types of cardiomyopathy. 3. Recognize the importance of family history on cardiomyopathy.

Considerations This is a 63-year-old man with 3 months of recurrent pleural effusions, palpitations, dyspnea on exertion, orthopnea, weight loss, and neuropathy. He has a family history of atrial fibrillation and “heart failure” in his father and physical findings suggestive of decreased weight, pleural effusions, and volume excess. His ECG is consistent with amyloidosis infiltrative cardiomyopathy. The differential diagnosis includes light-chain amyloidosis, hereditary (transthyretin mutant) amyloidosis, senile amyloidosis, or other infiltrative cardiomyopathy. However, given his father’s history of heart failure, hereditary amyloidosis warrants strong diagnostic consideration. In addition to a transthoracic echocardiogram, his workup must include assessment for renal function, complete blood count, and urine and protein electrophoreses to look for any plasma cell dyscrasias. In addition, a tissue diagnosis is essential. Endomyocardial biopsy often reveals amyloid amorphous light pink material interspersed within darker myocytes on hematoxylin and eosin staining. These infiltrates often stain positive with thioflavin S. Additional immunohistochemical staining for transthyretin and kappa and lambda chains will narrow the diagnosis.

258

CASE FIl ES: CARd IO l OGy

He is currently in decompensated heart failure as evidenced by his elevated jugular venous pressures and peripheral edema and warrants diuresis with a loop diuretic. Avoid calcium channel blockers, beta-blockers, digoxin, or other atrioventricular nodal blocking agents as they are poorly tolerated in patients with amyloid heart disease and may worsen hypotension. Pending the above-mentioned workup, consider pathologic diagnosis via endomyocardial biopsy. Furthermore, he is at risk for developing atrial arrhythmias such as atrial flutter and atrial fibrillation and warrants ambulatory monitoring. Should the results of his biopsy suggest hereditary amyloidosis, he should be counseled to undergo genetic testing.

APPROACH TO: Cardiomyopathies DEFINITIONS CARDIOMYOPATHY: Any pathology of the heart muscle that results in a decline of function of the myocardium that typically leads to a heart failure syndrome. Patients with cardiomyopathies may experience edema, ascites, dyspnea, arrhythmias, or even sudden cardiac death. Cardiomyopathies can be coarsely divided into two classes: those attributable to coronary heart disease (ischemic cardiomyopathy, ICM) and those that are not related to ischemic heart disease (nonischemic cardiomyopathy, NICM).

CLINICAL APPROACH Initial Evaluation of the Patient With Cardiom yopathy Given the considerable diversity of cardiomyopathy pathology, taking a thorough and accurate history and family history in addition to physical examination is essential. Certain physical findings, family histories, or exposures can provide strong clues pointing to the specific etiology. However, despite a thorough evaluation, most cardiomyopathy etiologies are idiopathic. When considering the diagnosis of cardiomyopathy, it is first essential to consider the two main categories of cardiomyopathy: primary cardiomyopathies, which are caused by an intrinsic myocardial disorder, or secondary cardiomyopathies, which are caused by extrinsic conditions (Figures 18-2 and 18-3). By far, ischemic cardiomyopathy is both the most prevalent and incident form of secondary cardiomyopathy. Primary cardiomyopathies, however, may have specific external causes. In order to make the diagnosis of a primary cardiomyopathy, obstructive coronary heart disease must be ruled out. These cardiomyopathies include diverse disease states, each with its own etiology.

Prim ary Cardiom yopathy Hypertrophic cardiomyopathy (HCM) is histologically characterized by disordered myocyte growth and fibrosis. Wall thickening typically involves the septum and less commonly, the apex of the heart. HCM is the most common genetically

SECTIO N II: Cl INICAl CASES

Ge ne tic

Fibroe la s tos is, HCM, a rrhythmoge nic right ve ntricula r ca rdiomyopa thy, le ft ve ntricle noncompa ction, mitochondria l myopa thy, dila te d ca rdiomyopa thy

Primary c ardio myo pathy

Mixe d

Ac quire d

259

Dila te d ca rdiomyopa thy, re s trictive ca rdiomyopa thy

Pe ripa rtum ca rdiomyopa thy, Ta kots ubo ca rdiomyopa thy, Loe ffle r’s

Figure 18-2. Primar car iom opathies (HCM, h pertrophic car iom opath ).

mediated cardiovascular disease and is the most prevalent cause of death in young athletes. Symptoms include dyspnea on exertion, fatigue, and lightheadedness, and are usually related to conditions that affect the dynamic left ventricular outflow tract (LVOT) obstruction and increase diastolic pressures. Activities that produce elevated heart rates and decreased preload will worsen LVOT obstruction and cause symptoms. [When there is significant LVOT obstruction, this condition may be referred to as hypertrophic obstructive cardiomyopathy (HOCM).] HCM is a significant risk factor for sudden cardiac death. The murmur of HCM is classically described as a harsh, crescendo-decrescendo murmur heard best at the left

Ne uro mus c ular

Mus cula r dys trophie s, Frie dre ich’s a ta xia , Noona n's syndrome

Mis c e llane o us

Is che mic, va lvula r, hype rte ns ive, ta chyca rdia -induce d, pa ce ma ke r-induce d

In ammato ry

S e c o ndary c ardio myo pathy

Endo c rine

Toxic ity Infe c tio us Me tabo lic / S to rag e

Sa rcoidos is, lymphocytic, e os inophilic, gia nt ce ll, conne ctive tis s ue dis orde rs Thyroid dis orde rs, Addis on’s dis e a s e, phe ochromocytoma , a crome ga ly, dia be te s me llitus

Che mothe ra py, coca ine, a mphe ta mine, a rs e nic, a lcohol, ra dia tion

Cha ga s, Lyme, HIV, e nte rovirus, a de novirus, CMV

Amyloidos is, he ma chroma tos is, glycoge n s tora ge dis e a s e s, Fa bry’s, obe s ity, a ne mia , be ribe ri, ca rnitine de ficie ncy, s e le nium de ficie ncy

Figure 18-3. Secon ar car iom opathies (CMV, c tomega ovirus; HIV, human immuno eficienc virus).

260

CASE FIl ES: CARd IO l OGy

intercostal space that augments with the Valsalva maneuver and standing–conditions that decrease preload. Treatment consists of atrioventricular nodal blocking agents that prolong LV filling time. Most commonly, beta-blockers are the drugs of choice. Patients with HCM (hypertrophic cardiomyopathy) may need ICDs, and definitive treatment involves either surgical or percutaneous septal reduction therapy. For more information regarding HCM, please refer to Case 8. Arrhythmogenic right ventricular cardiomyopathy (ARVC) is another form of genetically mediated cardiomyopathy whereby mutations in desmosomes in the right ventricular myocardium cause an excess of fibrous and fatty tissue in place of normal muscle. It typically begins in the right ventricle, leading to dilatation and RV failure, and may extend well into the left ventricle. Patients with this rare disorder typically present with heart failure, arrhythmia, and sudden cardiac death. An early manifestation of ARVC may be ventricular tachycardia (VT) originating from the right ventricle. Increased levels of myocardial involvement typically correlate with an increased risk of VT and ICDs are often the only available therapy to curtail the risk of sudden cardiac death. Exercise and vigorous physical activity are typically contraindicated as they may provoke the arrhythmias. Dilated cardiomyopathies (DCMs) are the most common form of nonischemic cardiomyopathy. DCM is typically characterized by a dilated hypertrophied ventricle not due to any identifiable cause. Some developments of DCM are often assumed to be related to remote episodes of viral myocarditis due to Coxsackie B or other enteroviruses. However, 25–35% of DCMs are familial, with mutations affecting the cardiomyocyte cytoskeleton or the cardiomyocyte contraction mechanism. However, only 15–25% of familial DCMs have a detectable genetic mutation. DCM is more common in males than females and typically carries a more favorable prognosis than ischemic cardiomyopathy. Peripartum cardiomyopathy is a rare form of DCM that typically develops in a timeframe that lasts from the final month of pregnancy until 5 months into the postpartum period. Although most patients with peripartum cardiomyopathy improve with medical therapy, nearly a third of all patients develop worsening heart failure. The cause of peripartum cardiomyopathy is unknown and likely involves a triggered inflammatory process. If, after months to a year, the left ventricular ejection fraction does not recover, the risk of heart failure during a subsequent pregnancy increases to nearly 21%. Takotsubo cardiomyopathy, or stress-induced cardiomyopathy or apical ballooning syndrome, is a disease entity affecting primarily postmenopausal women. This weakening of the myocardium can be caused by emotional stress such as a sudden relationship breakup, death of a loved one, or severe anxiety. It is thus commonly also called “broken heart syndrome.” Takotsubo cardiomyopathy commonly presents in a manner identical to that of an anterior wall myocardial infarction with chest pain, dyspnea, anterior ST segment elevation, and elevated cardiac biomarkers. Coronary angiography is often normal or reveals mild angiographic disease. The classic finding in Takotsubo cardiomyopathy is ballooning or marked dyskinesis of the left ventricular apex with hyperkinesis of the basal segments (Figure 18-4). The apical ballooning can be seen on ventriculography or echocardiography and it is transient; often there is dramatic improvement in systolic function within 48 hours

SECTIO N II: Cl INICAl CASES

261

Aorta Ba s a l Hype rkine s is

LV

Apica l Ba llooning

Figure 18-4. l eft ventricu ogram in Takotsubo’s car iom opath . S sto ic image emonstrating the c assic appearance of apica ba ooning with basa h perkinesis.

of presentation. A supraphysiological catecholamine surge is believed to cause this syndrome. Although patients may be dramatically ill on presentation, most patients with Takotsubo’s cardiomyopathy improve within 6 months, and the overall prognosis is quite good.

Secondary Cardiomyopathy By far, the most prevalent and incident form of either secondary cardiomyopathy is ischemic cardiomyopathy (ICM). Because it is essential to rule out significant coronary heart disease, diagnostic coronary angiography is an essential component of the evaluation of a patient with a newly diagnosed cardiomyopathy. ICM represents nearly 60–75% of all cases of systolic heart failure in industrialized countries and is typically defined as cardiomyopathy in the presence of extensive postmyocardial infarction scar, ischemia, or severely obstructive coronary heart disease. Once the diagnosis is highly suspected, it is essential to provide a thorough assessment regarding the risks and the benefits of either percutaneous or surgical revascularization. Cardiotoxic agents can typically cause cardiomyopathy. Anthracyclines, cyclophosphamide, and trastuzumab are cancer chemotherapeutic agents associated with cardiomyopathy. Anthracycline chemotherapeutics such as doxorubicin commonly cause cardiomyocyte destruction. Although the highest lifetime risk of developing cardiomyopathy from this drug is near 20% for those receiving a cumulative dose of > 700 mg/m2, most patients who receive 110 bpm may develop tachycardia-induced cardiomyopathy. Atrial fibrillation, atrial flutter, atrial tachycardia, and even ventricular arrhythmia may cause this entity. Identifying this diagnosis is essential, as the resolution of arrhythmia may reverse ventricular dysfunction. Thyroid disorders can commonly lead to the development of cardiomyopathy. Severe hypothyroidism, or myxedema, may cause cardiomyocyte hypertrophy and dilation and lead to the development of decreased cardiac output and heart failure. Hyperthyroidism may lead to the development of cardiomyopathy by causing atrial fibrillation or high-output heart failure. Because reversal of this endocrinopathy can lead to the reversal of cardiomyopathy, measuring thyroid-stimulating hormone levels and–if suspicion is high–thyroxine levels, is critical in anyone presenting with a cardiomyopathy.

SECTIO N II: Cl INICAl CASES

263

N utritional and vitamin deficiencies, although very rare in developed parts of the globe, can lead to the development of ventricular dysfunction. Thiamine deficiency (beriberi) can lead to a syndrome of high-output heart failure characterized by marked edema, pulmonary congestion, and peripheral vasodilation. Thiamine deficiency characterized by cardiomyopathy is typically referred to as “wet” beriberi. Diagnosis is established by decreased erythrocyte transketolase and low 24-urine thiamine levels. Treatment with intravenous thiamine followed by oral thiamine supplementation can lead to resolution of this cardiomyopathy. Rarely, in patients on chronic parenteral nutrition, carnitine and selenium deficiencies may cause cardiomyopathy. Both primary and secondary hemachromatoses may lead to a restrictive cardiomyopathy via chronic stiffening of the ventricle from iron deposition. The myocardium may stretch, and the ventricle can dilate over time, leading to systolic dysfunction as well. Although treatment with iron-chelating agents or phlebotomy may improve myocardial function, the deposition may become so severe that these therapies may have little effect. Genetic skeletal myopathies such as duch*enne’s, Becker’s, and limb girdle muscular dystrophy can involve the myocardium and lead to the development of DCM. Systemic sarcoidosis can involve the heart. Inflammation from sarcoidosis can lead to the development of varying degrees of heart block and to ventricular arrhythmia and LV dysfunction. Although the presence of noncaseating granulomas on endomyocardial biopsy is the gold standard for establishing this diagnosis, these granulomas are rarely seen. As such, this diagnosis is typically made through either cardiac MRI or positron emission tomography (PET). If there is radiographic evidence of active inflammation, immunosuppressives are commonly used. In 20–50% of subjects infected by the protozoan Trypanosoma cruzi, chronic Chagas disease may involve the myocardium. In the chronic phase this causes a DCM in the absence of significant obstructive coronary artery disease. Patients with this disorder are commonly from Latin America. High T. cruzi titers can facilitate diagnosis.

CASE CORRELATION • See also Case 16 (acute heart failure) and Case 17 (advanced heart failure).

COMPREHENSION QUESTIONS 18.1 Which of the following cardiomyopathies can be definitively managed surgically? A. Sarcoid cardiomyopathy B. Chagas cardiomyopathy C. Valvular cardiomyopathy D. Amyloid cardiomyopathy E. Dilated cardiomyopathy

264

CASE FIl ES: CARd IO l OGy

18.2 A 42-year-old African American man presents with dyspnea on exertion and episodes of lightheadedness. He has no family history of cardiomyopathy. On examination he is normotensive, is bradycardic, and has a diffuse and laterally displaced PMI. His jugular venous pulse is at 7 cm of water, and he has dry rales throughout his lung fields without any peripheral edema. On ECG he is in first-degree atrioventricular block. What is the most likely diagnosis? A. Ischemic cardiomyopathy B. C. D. E.

Amyloid cardiomyopathy Sarcoid cardiomyopathy Hypertensive cardiomyopathy Carnitine cardiomyopathy

18.3 A 60-year-old woman presents to the emergency department complaining of dyspnea on exertion and orthopnea. She tells you that she took medication but stopped nearly 2 months ago. She complains of low energy, weight gain, cold intolerance, and thinning hair. On examination her blood pressure is 140/106 mmHg, and her pulse is regular at 58 bpm with peripheral edema and cool extremities. She has a laterally displaced and diffuse PMI, and her JVP is 10 cm of water. Her LV ejection fraction is 35%. Which of the following lab studies would reveal the most likely etiology of her cardiomyopathy? A. Thyroid-stimulating hormone (TSH) B. Ferritin C. Troponin T D. Urinary thiamine E. Hemoglobin A 1C

ANSWERS 18.1 C. The definitive management for most valvular cardiomyopathies is surgical repair or replacement of either the regurgitant or the stenotic valve. 18.2 C. This patient has physical examination findings consistent with both cardiomyopathy and noncardiac lung disease. He has ECG evidence of abnormal AV nodal conduction. Given this constellation of findings in addition to his race, the most likely diagnosis is sarcoid cardiomyopathy. 18.3 A. This woman’s physical findings are consistent with clinical hypothyroidism. Additionally, the medication she is no longer taking was likely thyroid replacement hormone. Now she presents with a cardiomyopathy and checking a TSH level would likely confirm the diagnosis.

SECTIO N II: Cl INICAl CASES

265

CLINICAL PEARLS • The most common cause of cardiomyopathy is ischemic; therefore, diagnostic coronary angiography should be considered in every newly diagnosed cardiomyopathy. • A thorough past medical history, family history, physical examination, and appropriate laboratory evaluation should be undertaken in order to help diagnose that etiology of cardiomyopathy and to look for treatable causes. • Most cases of nonischemic cardiomyopathy are idiopathic. • HCM is often hereditary.

REFERENCES Grodin JL, Tang WH. Treatment strategies for the prevention of heart failure. Curr Heart Fail Rep. 2013;10(4):331–340. Hershberger RE, Siegfried JD. Update 2011: clinical and genetic issues in familial dilated cardiomyopathy. J A m Coll Cardiol. 2011;57(16):1641–1649. Hunt SA. ACC/AHA 2005 guideline update for the diagnosis and management of chronic heart failure in the adult: a report of the American College of Cardiology/American Heart Association Task Force on Practice Guidelines (Writing Committee to Update the 2001 Guidelines for the Evaluation and Management of Heart Failure). J A m Coll Cardiol. 2005;46(6):e1–e82. Jessup M, Abraham WT, Casey DE, et al. 2009 focused update: ACCF/AHA Guidelines for the Diagnosis and Management of Heart Failure in Adults: a report of the American College of Cardiology Foundation/American Heart Association Task Force on Practice Guidelines: developed in collaboration with the International Society for Heart and Lung Transplantation. Circulation. 2009;119(14):1977–2016. Nishimura RA, Holmes DR Jr. Clinical practice. Hypertrophic obstructive cardiomyopathy. N Engl J Med. 2004;350(13):1320–1327.

This page intentionally left blank

CASE 19 A 72- ear-ol man is seen in our clinic for spnea on exertion. Over the past week, he has notice worsening shortness of breath with mo erate amounts of housework an when walking to his car. He has also notice ifficult falling asleep, so he has been sleeping in his recliner chair. Two weeks ago, his chil ren were in town an the ate out frequentl at fast-foo restaurants. The patient has a histor of h pertension, h perlipi emia, an a m ocar ial infarction 3 ears prior. His me ications are aspirin, lisinopril, an atorvastatin. On examination, he appears tach pneic but not in respirator istress. His temperature is 98.9°F, heart rate 94 bpm, bloo pressure 134/ 74 mmHg, respirations 24 per minute, an O 2 saturation 93% on room air. Car iac auscultation reveals a regular rate an rh thm without murmurs or rubs. A low-pitche gallop is heart after the secon heart soun . There are wet crackles at the bases of both lungs bilaterall . The ab omen is soft without organomegal . The extremities are well perfuse , an there is 1+ bilateral pe al an presacral e ema. Jugular venous istension is 12 cm. ECG shows sinus rh thm with Q waves in V1–V4. Results of a complete bloo count an chemistr panel are unremarkable. C C C

What is the most likel iagnosis? What is the best next iagnostic step? What is the best next step in therap ?

268

CASE FILES: CARd IO LOGy

ANSWERS TO CASE 19: Chronic Heart Failure Summary: A 72-year-old man with a history of hypertension, hyperlipidemia, and an old myocardial infarction presented with worsening dyspnea on exertion. His exercise tolerance has worsened after a week of eating high-sodium foods. He is mildly tachypneic on exam with borderline low oxygen saturation. He has an S3 gallop, crackles, lower extremity edema, and elevated neck veins on exam. An ECG reveals Q waves in the anteroseptal leads signifying an old myocardial infarction. • Most likely diagnosis: Acute heart failure exacerbation. • N ext diagnostic step: Echocardiography. • N ext step in therapy: Loop diuretics.

ANALYSIS Objectives 1. Recognize the signs and symptoms of heart failure. 2. Know the diagnostic approach and common etiologies of systolic heart failure. 3. Understand the medical and device management for chronic systolic heart failure.

Considerations This 72-year-old man presented to clinic with worsening dyspnea on exertion and physical exam findings of volume overload (wet profile) and heart failure. The first priority in the office should be to assess his vital signs and tissue perfusion. Signs of low tissue perfusion (cold profile) are hypotension, cold extremities, altered mental status, elevated creatinine, abnormal liver transaminases, and an elevated lactate. Patients with abnormal tissue perfusion should be triaged to a higher level of care. Our patient did not show signs of low tissue perfusion and falls into the “warm and wet” category. Further categorization of heart failure into systolic or diastolic is an important step and can be accomplished by echocardiography. Equally important is determining the etiology of the heart failure. For our patient, systolic heart failure and ischemic cardiomyopathy are the most likely scenarios given his old anterior myocardial infarction. Treatment centers around relieving acute congestion, starting evidence-based medical therapy, and educating patients on diet and lifestyle changes needed to prevent further exacerbations and improve mortality.

SECTIO N II: CLINICAL CASES

269

APPROACH TO: Chronic Heart Failure DEFINITIONS HEART FAILU RE: A clinical syndrome characterized by shortness of breath or fatigue as a result of underlying structural or functional heart disease. CARDIOMYOPATHY: Disease or dysfunction of the myocardium. Cardiomyopathies are commonly categorized as ischemic or nonischemic. Further classifications are based on ventricular structure or etiology (dilated, hypertrophic, restrictive, Takotsubo, alcoholic, etc). ORTHOPN EA: Shortness of breath in a recombinant position. PAROXYSMAL N OCTU RN AL DYSPN EA: Respiratory distress that awakens the patient from sleep.

CLINICAL APPROACH Etiologies Heart failure defines a persistent and growing segment of the US healthcare system, with an estimated prevalence of over 6 million people. It is associated with high morbidity, mortality, and cost to the healthcare system. In addition, heart failure is the most common reason for hospital readmission in both the medical and surgical populations. Heart failure can be broken down into systolic, or heart failure with reduced ejection fraction (HFrEF); and diastolic, or heart failure with preserved ejection fraction (HFpEF). The former is defined as a clinical presentation consistent with heart failure along with an ejection fraction of 120 ms.

SECTIO N II: CLINICAL CASES

273

CASE CORRELATION • See also Case 16 (acute heart failure), Case 17 (advanced heart failure), and Case 18 (cardiomyopathies).

COMPREHENSION QUESTIONS 19.1 A 65-year-old man with heart failure has pillow orthopnea and shortness of breath while walking around his house. He is comfortable only at rest. What NYHA heart failure class symptoms does he have? A. Class I B. Class II C. Class III D. Class IV 19.2 A 45-year-old man presents with dyspnea on exertion and four-pillow orthopnea. JVP is elevated, and you hear an S3 on exam. ECG is normal sinus rhythm without Q waves, ST, or T wave changes. Echocardiography shows global left ventricular hypokinesis and an EF of 30% with no valvular abnormalities. What is the next best step in diagnosis or management? A. Order a chest CT for a coronary calcium score B. Assess for coronary artery disease with a stress test or cardiac catheterization C. Implant a biventricular implantable cardioverter defibrillator (for CRT) D. Order a multigated acquisition (MUGA) scan to better assess cardiac function 19.3 A 56-year-old white man with a history of a large anterior wall myocardial infarction and an EF of 20% presents to your clinic. He notes shortness of breath after climbing two flights of stairs and after walking four or five blocks but denies PND or orthopnea. He takes aspirin, atorvastatin, metoprolol, and lisinopril. Which of the following is the best step in managing his heart failure? A. Begin hydralazine and isosorbide dinitrate B. Begin digoxin C. Begin eplerenone D. Hospitalization for intravenous dobutamine infusion E. Referral for consideration of a left ventricular assist device

274

CASE FILES: CARd IO LOGy

19.4 A 75-year-old man with chronic systolic heart failure presents to your clinic with class III symptoms. His most recent echo showed an EF of 25%. He takes losartan, metoprolol, aspirin, and spironolactone. Which of the following interventions will decrease his mortality? A. Digoxin B. Low-salt diet and fluid restriction to 1.5 L/day C. Implantable cardioverter defibrillator D. Cardiac rehabilitation

ANSWERS 19.1 C. The patient is having class III heart failure symptoms with minimal amounts of exertion. 19.2 B. An ischemic evaluation is essential in a patient presenting with recentonset heart failure since the most common cause of cardiomyopathy in the United States is ischemic heart disease. 19.3 C. Eplerenone is indicated in patients with heart failure and class II symptoms, and is the most reasonable choice provided. 19.4 C. While digoxin, dietary recommendations, and cardiac rehab can improve symptoms and decrease readmission in heart failure, only an ICD has been shown to decrease mortality.

CLINICAL PEARLS C

Heart failure is a clinical s n rome that affects over 6 million Americans an is the lea ing cause of hospital rea mission after both me ical an surgical ischarges.

C

The most important part of the ph sical exam is an assessment of volume status an tissue perfusion.

C

ACE inhibitors or ARBs an beta-blockers are the cornerstones of heart failure management. Al osterone receptor blockers an h ralazine-nitrate combinations can be a e in select patients.

C

I entif ing the etiolog of heart failure is essential in provi ing targete therap .

C

Remember to check the ECG for a left bun le branch block an a QRS uration of >120 ms to assess can i ac for implantation of a BiV-ICd .

SECTIO N II: CLINICAL CASES

275

REFERENCES Grant A, Hanna M. Medical treatment of heart failure. In: Griffin BP, Kapadia SR, Rimmerman CM, eds. T he Cleveland Clinic Cardiology Board Review. 2nd ed. Philadelphia, PA: Lippincott Williams & Wilkins; 2013. Jacob M, Tang WHW. Pathophysiology of congestive heart failure. In: Griffin BP, Kapadia SR, Rimmerman CM, eds. T he Cleveland Clinic Cardiology Board Review. 2nd ed. Philadelphia, PA: Lippincott Williams & Wilkins; 2013. Jencks SF, Williams MV, Coleman EA. Rehospitalizations among patients in the Medicare fee-for-service program. N Engl J Med. 2009;360(14):1418–1428. Nohria A, Lewis E, Stevenson LW. Medical management of advanced heart failure. JA MA . 2002;287(5):628-640.

This page intentionally left blank

CASE 20 A 57- ear-ol man is seen in the emergenc room for chest heaviness. Last week, while sitting on his couch, he note an episo e of chest iscomfort that spontaneousl resolve after 10 minutes. He assume that it was in igestion. However, this morning he ha an episo e of severe recurrent chest iscomfort that i not resolve an was associate with iaphoresis an nausea. The patient has a histor of t pe 2 iabetes mellitus, h pertension, h perlipi emia, an obstructive sleep apnea. His me ications are aspirin, metformin, can esartan, an rosuvastatin. Vital signs were normal in the ambulance, so he was treate with chewable aspirin 325 mg an one sublingual nitrogl cerin. This partiall relieve his pain but cause him izziness an lighthea e ness. On examination in the emergenc room he appears iaphoretic. Temperature is 98.4°C, heart rate 105 bpm, bloo pressure 85/ 57 mmHg, respirations 18 breaths/ min, an O 2 saturation 94% on room air. Car iac auscultation reveals a regular tach car ia without murmurs, rubs, or gallops. The lungs are clear to auscultation bilaterall . The ab omen is soft without istension or ten erness. The extremities are cool with threa pulses an no e ema. Neurological exam is normal. Jugular venous istension is not seen. An ECG is shown in Figure 20-1. C C C

What is the most likel iagnosis? What is the best next iagnostic step? What is the best next step in acute management?

I

a VR

V1

V4

II

a VL

V2

V5

III

a VF

V3

V6

V1

II

V5 Figure 20-1. ECG of the main subject of this case.

SECTIO N II: CLINICAL CASES

279

ANSWER TO CASE 20: Acute Right Ventricular Failure Complicating Myocardial Infarction Summary: A 57-year-old man with type 2 diabetes, hypertension, hyperlipidemia, and obstructive sleep apnea presented with severe chest pain. He has several risk factors for coronary artery disease, and his symptoms are typical of crescendo angina. He was treated with aspirin and nitroglycerin in the ambulance, which caused him to become hypotensive. His ECG reveals ST elevations in the inferior leads. • Most likely diagnosis: Right ventricular infarction. • N ext step in acute management: Intravenous fluids to raise preload. • N ext diagnostic step: Cardiac catheterization.

ANALYSIS Objectives 1. Recognize the signs and symptoms of right-heart failure. 2. Know the most common etiologies of right-heart failure. 3. Understand the management of right-heart failure on the basis of the given etiology.

Considerations This 57-year-old man with coronary artery disease risk factors presented to the emergency room with chest pain. The first priority should be assessment of his vital signs and electrocardiogram. Vital signs were normal in the ambulance; however, sublingual nitroglycerin led to hypotension. This should immediately suggest right ventricular involvement of a myocardial infarction to the care provider. An acutely failing right ventricle depends on preload to maintain cardiac output. Dropping the preload via nitrates will lead to hypotension. Electrocardiography was appropriately done next and showed inferior ST elevations. A right-sided ECG can also be performed to look for RV involvement, which shows up as ST elevations in lead V4R.

APPROACH TO: Right Ventricular Faiure DEFINITIONS RIGHT VEN TRICU LAR (RV) FAILU RE: The inability of the right ventricle to provide adequate blood flow through the pulmonary circulation at normal central venous filling pressures.

280

CASE FILES: CARd IO LOGy

AN ASARCA: Extreme generalized edema. COR PU LMON ALE: Right ventricular enlargement and dysfunction secondary to increased pulmonary artery resistance or pressure. EBSTEIN AN OMALY: A congenital malformation in which the septal and posterior leaflets of the tricuspid valve are displaced toward the apex, leading to atrialization of the right ventricle and tricuspid regurgitation

CLINICAL APPROACH Etiologies The right heart is often forgotten in typical discussions of heart failure. The RV is thinner-walled, more difficult to visualize on echocardiography, and functions to pump blood to only one organ. However, dysfunction of the right ventricle can lead to severe morbidity and mortality. The right ventricle can fail from a variety of primary and secondary causes (Table 20-1). Primary right ventricular dysfunction can result from RV myocardial infarction, myocarditis, or cardiomyopathy. However, the most common causes are secondary ones, where the RV is a bystander to left ventricular, valvular, or pulmonary vascular dysfunction (cor pulmonale). This secondary involvement of the right ventricle portends a worse prognosis. Specifially noteworthy here is tricuspid regurgitation, which is most commonly a functional problem. In other words, the right ventricle is secondarily volumeoverloaded, leading to dilatation of the ventricle, and tricuspid annulus, which causes regurgitation. The severity of this regurgitation often varies with the patient’s volume status. Primary tricuspid regurgitation is typically caused by endocarditis, valvulopathy (carcinoid heart disease), or genetic abnormality such as Ebstein anomaly.

Pathophysiology The right ventricle has the same cardiac output as the left ventricle at about 25% the stroke work given the low resistance in the pulmonary circulation. Thus, the RV myocardium is more thin-walled than the LV. The blood supply to the right ventricular free wall arises from RV branches off the right coronary artery, while the septum is supplied by the septal perforators off the left anterior descending artery Table 20-1 • CAUSES OF RIGHT VENTRICULAR FAILURE Primary RV failure RV m ocar ial infarction M ocar itis Car iom opath Secondary RV failure Left-heart failure Pulmonar h pertension Valvular heart isease Pulmonar embolism Eisenmenger’s s n rome

SECTIO N II: CLINICAL CASES

281

and the posterior descending artery. Given the subsystemic pressures of the RV, its coronary supply is filled during both systole and diastole. It is also important to recognize that left ventricular preload is dependent on the function of the right ventricle. A failed right ventricle requires an adequate blood volume to maintain blood pressure and cardiac output. This is why the treatment for hypotension in the setting of a right ventricular infarction is to first administer intravenous fluids to maintain preload.

Clinical Presentation A failed right ventricle leads to fluid backup into the periphery and venous congestion of the abdominal organs. Patients complain of peripheral edema, abdominal distension, and fullness. Hepatic congestion can lead to liver failure, ascites, and coagulopathy. Renal vascular congestion can lead to acute kidney injury. This condition can be misdiagnosed as other conditions that cause anasarca, such as primary liver failure or nephrotic syndrome. The lack of lung symptoms and pulmonary edema on chest x-ray often confuse clinicians into prematurely ruling out heart failure. However, as most right-heart failure is caused by left-sided heart failure or pulmonary hypertension, patients will often have some pulmonary complaints. On physical exam, vital signs may be abnormal in states of low cardiac output. There may be a low blood pressure or a compensatory increase in heart rate. Weight gain over time should be evaluated. Head and neck exam will show elevated jugular venous pressure (JVP). This is calculated by positioning the patient at a 45° angle and measuring the jugular venous pulsation. JVP above 8 cm of water above the right atrium is considered elevated. Kussmaul’s sign appears when neck veins rise paradoxically during inspiration. This is due to right-heart failure or pericardial tamponade. Another way of approximating central venous pressure is to view the superficial veins in the patient’s arm. The arm is raised until the veins disappear, and this distance in centimeters above the heart will estimate the central venous pressure. One can also elicit the hepatojugular reflux and look for a sustained elevation in JVP, which signifies elevated right-sided filling pressures. On cardiac exam it is important to listen for murmurs, especially right-sided murmurs, which increase with inspiration. A loud P2 signifying pulmonary hypertension and a right-sided S3 of RV failure may also be heard. Abdominal exam may reveal hepatomegaly, ascites, or distended abdominal wall veins. A pulsatile liver is a sign of severe tricuspid regurgitation. The peripheral exam often reveals edema. It is important to remember to look for edema in the sacral region, especially in patients who are bedbound. There may also be bruising from coagulopathy. Electrocardiography may show an acute inferior infarct, or Q waves in the inferior leads signifying an old inferior myocardial infarction. Right-axis deviation, a right bundle branch block, or tall R waves (larger than the corresponding S wave) in lead V1 are all possible signs of right ventricular disease. Chest radiography will show an enlarged right atrial or ventricle (Figure 20-2), while pulmonary findings will vary with the etiology. Laboratory abnormalities may be found, including hyponatremia, elevated BUN, and creatinine and transaminitis due to hepatic congestion. Elevated cardiac biomarkers or brain natriuretic peptide may be found in chronic heart failure or may be a sign of an acute ischemic event.

282

CASE FILES: CARd IO LOGy

Figure 20-2. Enlarge right atrium, right ventricle, an bilateral pulmonar arteries in a patient with primar pulmonar h pertension.

Echocardiography should be used to assess for left and right ventricular function and valvular disease. It can also be used to estimate pulmonary and right atrial pressures. The presence of right ventricular failure portends a poorer prognosis for most patients with cardiovascular disease. Right ventricular systolic dysfunction detected by echocardiography is associated with a nearly twofold increase in mortality in patients with chronic left ventricular systolic heart failure. Significant right ventricular failure also affects the management of patients with advanced heart failure. Mechanical therapies such as left ventricular assist devices (LVADs) may not provide adequate hemodynamic support in patients with failing right ventricles; such patients may require implantation of a biventricular support device or a total artificial heart instead. Evidence of acute right ventricular strain detected by echocardiography or ECG is also associated with increased mortality in patients presenting with acute pulmonary thromboembolism (PE). The presence of right ventricular strain in the setting of PE, even in the face of hemodynamic stability, is a criterion for the use of intravenous fibrinolytic therapy.

Treatm ent Treatment of right ventricular failure centers around treating the underlying etiology of the dysfunction. Symptoms of RV failure due to volume overload states of left ventricular failure are remedied by proper diuresis and treatment of the LV failure. If the cause is pulmonary arterial hypertension, targeted therapies such as epoprostenol infusions appear to improve right ventricular function. Other pressure overload states such as pulmonary embolism should be treated with anticoagulation with the consideration of thrombolytic therapy in cases of hemodynamic compromise. Severe valvular heart disease typically needs to be corrected surgically.

SECTIO N II: CLINICAL CASES

283

Functional tricuspid regurgitation should be treated with diuresis, while symptomatic primary tricuspid regurgitation should be surgically corrected. Undergoing operative treatment for tricuspid regurgitation prior to the onset of advanced heart failure symptoms is associated with a decreased postoperative mortality. Right ventricular failure from myocardial infarction typically also involves the left ventricular myocardium, and coronary artery disease management and neurohormonal blockade with renin-angiotensin-aldosterone system inhibition and beta-blockers is the treatment of choice. Unfortunately, it is unknown whether this therapy is beneficial in isolated RV failure due to other causes. In acute cases of cardiogenic shock due to RV failure, inotropic agents such as dobutamine can be used. Hypoxemia (which causes pulmonary arterial vasoconstriction) should be corrected, and pure alpha agonists should be avoided because of the increase in pulmonary vascular resistance.

CASE CORRELATION • See also Case 16 (acute heart failure), Case 17 (advanced heart failure), Case 18 (cardiomyopathies), and Case 19 (chronic heart failure).

COMPREHENSION QUESTIONS 20.1 A 66-year-old man presented to the emergency room with acute-onset severe 10/10 chest pain. He was found to have ST elevations in leads II, III, and aVF. His blood pressure is 101/64 mmHg. He was given 325 mg of chewable aspirin. What is your next step in management? A. Sublingual nitroglycerin B. Intravenous morphine C. Right-sided ECG D. Echocardiogram E. Cardiac enzymes 20.2 A 42-year-old woman with a history of human immunodeficiency virus (HIV) and active drug use presents to the clinic with 3 weeks of low-grade fevers. She also notes leg swelling and abdominal fullness. On exam, her jugular venous pressure is elevated and she has 2+ edema to her midthighs bilaterally. She has clear lungs and a heart murmur. What do you expect her murmur to sound like? A. Holosystolic murmur at the left lower sternal border that increases with inspiration B. Crescendo-decrescendo murmur heard best at the right upper sternal border C. Blowing diastolic mumur heard best at the right upper sternal border while sitting forward D. Holosystolic murmur at the apex that radiates to the axilla and increases with expiration

284

CASE FILES: CARd IO LOGy

20.3 A 72-year-old man with a history of mitral valve prolapse, moderate mitral regurgitation, and a remote history of smoking presents with worsening lower extremity edema. Echocardiography shows the aforementioned findings as well as a dilated left atrium and moderate right ventricular dysfunction. His ejection fraction is 60%. What is the cause of his right ventricular dysfunction? A. Right ventricular myocardial infarction B. Primary pulmonary hypertension C. Mitral regurgitation D. COPD E. Left ventricular systolic heart failure

ANSWERS 20.1 C. A patient with ST elevations in the inferior leads should have a right-sided ECG done to look for RV infarction (diagnosed with ST elevations in lead V4R). Acute right ventricular failure, as seen in an RV infarction, requires preload to maintain blood pressure. Giving nitroglycerin prior to this may drop his blood pressure even more. After an ECG, a heart catheterization should be facilitated before obtaining an echocardiogram or cardiac enzymes. 20.2 A. The patient is a drug user with fever, signs of right-sided heart failure, clear lungs, and a new heart murmur. All of these factors point to right-sided valvular infective endocarditis. This most commonly affects the tricuspid valve and causes tricuspid regurgitation, which results in a holosystolic murmur best heard at the left lower or sternal border. This murmur classically increases with inspiration. 20.3 C. The most likely cause of this patient’s right ventricular failure is due to volume overload from mitral regurgitation. A normal left ventricular ejection fraction coupled with a dilated left atrium signals left ventricular diastolic dysfunction or significant mitral valve disease.

CLINICAL PEARLS C

The most common cause of right ventricular failure is left-si e heart failure.

C

Right-si e heart failure shoul be consi ere in patients presenting with anasarca or cr ptogenic cirrhosis.

C

The t pical heart failure regimen of ACE inhibitors, ARBs, al osterone receptor blockers, beta-blockers, an h ralazine-nitrate combinations has not been shown to work in isolate right ventricular heart failure.

C

I entif ing the etiolog of right-heart failure is essential in provi ing targete therap .

C

Functional tricuspi regurgitation is often a sign of right ventricular volume overloa an is treate with iuresis, while severe primar tricuspi regurgitation requires surgical management.

SECTIO N II: CLINICAL CASES

285

REFERENCES Aksoy O, Tuzcu EM. Complications of myocardial infarction. In: Griffin BP, Kapadia SR, Rimmerman CM, eds. T he Cleveland Clinic Cardiology Board Review. 2nd ed. Philadelphia, PA: Lippincott Williams & Wilkins; 2013. Jacob M, Tang WHW. Pathophysiology of congestive heart failure. In: Griffin BP, Kapadia SR, Rimmerman CM, eds. T he Cleveland Clinic Cardiology Board Review. 2nd ed. Philadelphia, PA: Lippincott Williams & Wilkins; 2013. Rogers JH, Bolling SF. The tricuspid valve. Perspective and evolving management of tricuspid regurgitation. Circulation. 2009;119:2718–2725. Voelkel NF, Quaife RA, Leinwand LA, et al. Right ventricular function and failure: report of a National Heart, Lung, and Blood Institute Working Group on Cellular and Molecular Mechanisms of Right Heart Failure. Circulation. 2006;114(17):1883–1891.

This page intentionally left blank

CASE 21 A 68- ea -ol obese man is b ou ht in b ambulance to the eme enc oom complainin o ab upt onset o chest iscom o t o the past hou . He esc ibes “seve e achin ” un e the istal aspect o his ste num with a iation into the in e io le t si e o his chest. His s mptoms sta te at est, have been constant, an wo sen when he takes a eep b eath. He has a histo o aci e lux isease, alcoholism, h pe lipi emia, h pe tension, p ostate cance , an a st on amil histo o ea l m oca ial in a ction. On examination, the patient appea s estless an in mo est ist ess. Vital si ns a e tempe atu e 98.4°F, hea t ate 112 bpm, bloo p essu e 82/ 54 mmH in the le t a m an 84/ 57 mmH in the i ht a m, espi ations 26 b eaths/ min, an ox en satu ation 90% on oom ai . The patient’s b eathin is labo e , with no mal b eath soun s. He has a tach ca ic, e ula h thm without mu mu s, ubs, o allops. The epi ast ium is mil l ten e to palpation, an his stool is uiac-ne ative. C C

What a e the p io it ia nosis to evaluate? What a e ou next ia nostic steps?

288

CASE FILES: CAr d IO LOg y

ANSWEr TO CASE 21: Chest Pain, Undifferentiated Summary: This 68-year-old man presents with vague substernal and left-sided chest pain for 1 hour. His pain is associated with dyspnea, tachypnea, and unstable vital signs, including hypotension, tachycardia, and relative hypoxia. He is currently in respiratory distress, and triage should focus on differentiating between possible lifethreatening etiologies of his symptoms that require urgent attention. • Priority differential diagnosis: “Can’t miss” diagnoses include pulmonary embolism (PE), acute coronary syndrome (ACS), aortic dissection, and tension pneumothorax, since these are potentially fatal conditions. • N ext diagnostic steps: Stat ECG, CXR, labs (including cardiac biomarkers and ABG), and consider contrast enhanced CT of the chest.

ANALYSIS Objectives 1. Understand the differential diagnosis and triage of chest pain, and be able to rule out life-threatening emergent causes (acute myocardial infarction, pulmonary embolism, pneumothorax, and aortic dissection). 2. Be able to distinguish angina from other types of chest pain. 3. Identify the treatment options available for common causes of chest pain.

Considerations Undifferentiated chest pain in the emergency medical setting is an extremely common presentation. In all patients with undifferentiated chest pain, the initial priority should be ruling out life-threatening causes, as prompt intervention may prove life-saving. Subtle changes in vital signs (such as tachycardia and mild hypoxia) should be noted; although nonspecific, this may be the first clue of a serious underlying etiology. This patient has risk factors for thromboembolic disease (obesity and malignancy), cardiovascular disease (obesity, age, gender, hypertension, hyperlipidemia, and a strong family history), as well as peptic ulcer disease (acid reflux and alcoholism). As such, the differential should initially be kept broad, and narrowed once life- threatening causes are ruled out.

APPr OACH TO: Chest Pain DEFINITIONS ACU TE CORON ARY SYN DROME (ACS): Consisting of unstable angina, non-ST elevation MI (NSTEMI), and ST elevation MI.

SECTIO N II: CLINICAL CASES

289

AORTIC DISSECTION : Intimal tear of the aorta with blood tracking within the media of the ascending (type A) or descending (type B) aorta, creating a false lumen. PERCU TAN EOU S CORON ARY IN TERVEN TION (PCI): Catheter-based therapy by blood flow is returned to an occluded coronary artery by balloon angioplasty or stenting. PN EU MOTHORAX: Occupation of the pleural space by air, often due to trauma, rupture of a pleural bleb, or iatrogenesis. May be simple (uncomplicated), or tension, with shifting of the mediastinum, respiratory distress, and hemodynamic compromise. THROMBOLYSIS: Restoration of blood flow to the coronary or pulmonary vasculature by administration of thrombolytic agents, which activate plasmin, promoting fibrinolysis.

Differential Diagnosis The differential diagnosis of chest pain is extensive, and although it is usually due to benign causes, some causes of chest pain may be life-threatening. As such, for each patient presenting with chest pain, serious causes should be ruled out before less dangerous conditions are considered. Common etiologies of chest pain grouped by system are summarized in Table 21-1. The differential diagnosis should be tailored to individual patients according to their presentation, specific risk factor profile, and setting in which they are evaluated. N onemergent chest pain evaluated in the primary care office is most often due to musculoskeletal pain followed by gastrointestinal issues and is less commonly due to cardiac causes (most of which are stable angina). However, even in the primary care office, patients with risk factors for coronary artery disease are much more likely to have cardiac pain, and in patients older than 40, up to 50% of cases of acute chest pain may be due to a cardiac cause. Further, in the emergency department (ED), the likelihood of a life-threatening cause of chest pain is much greater, and as such the index of suspicion for such causes should be higher in patients seen in the ED.

History Several important clues to be gained from the patient history can help rule in or out potentially fatal causes of chest pain. The duration of the chest pain is of specific

Table 21-1 • COMMON CAUSES OF CHEST PAIN BY ORGAN SYSTEM Cardiovascular

Respiratory

Gastrointestinal

Other

ACS (UA/ NSTEMI o STEMI) Ao tic issection Stable an ina Pe ica itis M oca itis

Pulmona embolism Pneumotho ax Pneumonia Pleu ac

g Er d d ES PUd Cholec stitis Panc eatitis

MSK Costoch on itis Anxiet

Abbreviations: ACS, acute co ona s n ome; d ES, i use esopha eal spasm; g Er d , ast oesopha eal ef ux isease; MSK, musculoskeletal; NSTEMI, non-ST elevation MI; PUd , peptic ulce isease; STEMI, ST elevation MI. Indicates potentially life-threatening diagnosis.

290

CASE FILES: CAr d IO LOg y

importance, as MI, pneumothorax, PE, and aortic dissection typically have an abrupt onset and may last for hours, but are rarely present for days on end. Further, it is crucial to determine whether the pain is exertional, occurs at rest, is pleuritic, is positional, or is similar to pain in any prior episodes. Additionally, the quality of the pain, location, and radiation are important. Although myocardial ischemia classically localizes to the midchest or left chest and often radiates to the left jaw and down the left arm, presentations may be atypical in the elderly, women, and diabetics. U p to one-third of these patients may not experience classic ischemic chest pain with myocardial infarction (MI). Further, the pain of an aortic dissection is typically severe and “tearing,” radiating from the chest through to the back. Pulmonary embolus classically causes pleuritic pain (which worsens on deep inhalation), and is more common in patients with malignancy, on oral contraceptive pills, in hypercoagulable states (Factor V Leiden, oral contraceptive pills, etc), and following recent travel (eg, a long plane flight or car ride). Spontaneous pneumothorax classically occurs in tall patients, those with cystic fibrosis, α 1-antitrypsin deficiency, following trauma to the chest, or iatrogenically. Certain characteristics increase the likelihood of chest pain that is attributable to myocardial ischemia, including pain that radiates to either arm or shoulder, exertional pain, pain associated with diaphoresis, and pain similar to or worse than pain during a prior MI. Findings that decrease the likelihood that the chest pain is due to a myocardial ischemia include pain that is pleuritic, positional, sharp, focal, or reproducible with palpation. Less urgent causes of chest pain that may mimic MI include the following (see also Table 21-1): pericarditis (pain is typically better when leaning forward, and may be pleuritic), myocarditis (may be preceded by a recent flulike illness), pneumonia (may be associated with fevers, chills, cough, and leukocytosis), peptic ulcer (pain is more epigastric, is reproducible, and may be associated with peritoneal signs if perforated), pancreatitis, cholecystitis, and musculoskeletal pain (always a diagnosis of exclusion).

Physical Exam Assessment of the vital signs is essential in the early evaluation of chest pain. Tachycardia and tachypnea may be early signs of a pulmonary embolism, even if the patient is not hypoxic. Additionally, while not very sensitive, a blood pressure differential of >20 mmHg between the arms is suggestive of an aortic dissection. The physical examination may otherwise be completely normal in a patient with life-threatening chest pain. As such, a normal exam may be falsely assuring, and diagnostic testing should be pursued. However, patients with an MI may develop a 4th heart sound, and may have an audible murmur of ischemic mitral regurgitation or signs of heart failure on examination. Patients with a pneumothorax may exhibit decreased breath sounds or hemodynamic instability, an important clue to this diagnosis.

Diagnostic Testing An ECG should be obtained within 10 minutes of arrival to the ED to rule out acute MI. Evidence of right-heart strain may be apparent on ECG in patients presenting

SECTIO N II: CLINICAL CASES

291

with PE, but this is not a uniform finding. A portable CXR should be obtained as soon as able if pneumothorax is suspected; this will also help rule out respiratory causes of chest pain, such as pneumonia. Labs, including cardiac biomarkers [troponin, creatine kinase myocardial band (CK-MB)], creatinine, platelet count, coagulation factors (prothrombin and activated thromboplastin time), and blood type should be obtained in case anticoagulation is needed. Further, a d -dimer may be ordered for patients at low to intermediate risk of PE, and a CT angiogram should be considered in patients for whom PE or aortic dissection is suspected. d -dimer has outstanding negative predictive value in the assessment of patients with suspected PE or aortic dissection; a completely normal d -dimer virtually excludes these diagnoses from the differential. Additional diagnostic testing should be individualized to each patient’s needs (liver function tests, amylase/lipase, etc). Findings on ECG suggestive of an acute MI include ST segment depression and T wave inversion in the setting of an NSTEMI, and ST segment elevation or new left bundle branch block due to a STEMI. If ECG is equivocal or the diagnosis is in question and the patient is stable, a bedside transthoracic echocardiogram should be performed to rule out regional wall motion abnormalities due to cardiac ischemia.

Em ergent Therapy Emergent therapy for patients for chest pain may be lifesaving and should be directed at the underlying cause. Patients with an acute MI should be given a full-dose aspirin (325 mg), and started on IV unfractionated heparin, and administration of a potent platelet inhibitor (such as clopidogrel, prasugrel, or ticagrelor) should be considered in patients without contraindications. Patients with STEMI should immediately proceed to PCI, and patients with STEMI who cannot receive PCI within 120 minutes should be considered for thrombolysis (with an agent such as alteplase, reteplace, or tenekteplase), whereas lytic agents are contraindicated in NSTEMI. For NSTEMI, if not high-risk, PCI can be delayed for up to 72 hours, and patients with high-risk NSTEMI (persistent chest pain, heart failure, or electrical instability) should proceed immediately to PCI. Efforts should also be made to control pain (with IV nitroglycerin or morphine if refractory), and myocardial oxygen demand (with IV nitroglycerine, beta-blocker, ACE inhibitor, or other therapy). See Cases 1 and 2 for additional information regarding the management of patients with acute coronary syndromes. It is essential to determine whether the chest pain may be due to aortic dissection or MI, as antiplatelet and anticoagulant agents are contraindicated in patients with aortic dissection but standard of care with MI. Patients with aortic dissection are typically emergently treated with IV beta-blockers (which decrease heart rate, blood pressure, and shear force of blood along the arterial wall) and afterload reduction with nitroprusside. Type A dissections (involving the ascending aorta to the left subclavian artery) are typically managed with immediate surgery, whereas type B dissections (involving the descending aorta distal to the left subclavian artery) may be initially managed medically with surgery reserved for patients with refractory pain or evidence of end-organ hypoperfusion. In the case of simple, uncomplicated pneumothorax, patients are typically monitored closely with serial CXR, and 100% oxygen may be empirically administered

292

CASE FILES: CAr d IO LOg y

to increase the rate of absorption. Patients with tension pneumothorax are often unstable on presentation, and require a needle thoracotomy to the 2nd intercostal space, midclavicular line. This immediately relieves the pressure, and a chest tube may be placed surgically immediately thereafter.

CASE CORRELATION • See also Case 1 (acute coronary syndrome/STEMI), Case 2 (acute coronary syndrome/NSTEMI), Case 4 (chronic coronary artery disease), Case 8 (hypertrophic obstructive cardiomyopathy), and Case 10 (valvular stenosis).

COMPREHENSION QUESTIONS 21.1 A 68-year-old man with no medical history presents to a rural emergency department with chest pain for the past 30 minutes. The ECG shows ST elevation in V3–V6 and I, and aVL. The hospital is not equipped for PCI, and the closest hospital that performs PCI is 3 hours away. Vital signs are HR 110 bpm, BP 150/84 mmHg, RR 18 per minute, and O 2 saturation 98% on room air (RA). In addition to aspirin and IV heparin, what is the most appropriate next step? A. Administration of full-dose thrombolysis, and transfer to the nearest PCI capable hospital for angiography B. Administration of full-dose thrombolysis, and subsequent transfer only if patient is unstable C. Administration of half-dose thrombolysis, and transfer to the nearest PCI capable hospital for immediate PCI D. Medical management with the addition of clopidogrel 21.2 A 70-year-old woman with a history of hypertension, coronary artery disease, and smoking presents with tearing chest pain across the chest that radiates to the back for the past 1 hour. Vitals are HR 100 bpm , BP 190/110 mmHg, RR 18 per minute, and O 2 saturation 97% on RA. A chest CT with contrast shows an aortic dissection extending 1 cm distal to the left subclavian artery to 2 cm superior to the renal arteries. What is the most appropriate management strategy? A. Immediate surgery B. Administration of IV labetalol, nitroglycerine, and surgery when stable C. Administration of IV heparin, IV metoprolol, and continued monitoring D. Administration of IV heparin, IV nitroprusside, IV metoprolol, and continued monitoring E. Administration of IV metoprolol, IV nitroprusside, and continued monitoring

SECTIO N II: CLINICAL CASES

293

21.3 An 18-year-old man presents with chest pain and dyspnea with deep breathing for the past 1 hour. Vitals are stable. CXR shows a small pneumothorax involving 10% of area of the left lung. What is the most appropriate management strategy? A. Needle thoracotomy of the L 2nd intercostal space, midclavicular line B. Placement of a chest tube C. 100% oxygen and serial CXR over the next 24 hours D. Albuterol inhaler, 100% oxygen, and chest physical therapy 21.4 A 45-year-old man with a history of hypertension and lung cancer presents with pleuritic chest pain, and left calf swelling after a 4-hour plane flight. He is tachycardic, hypoxic, but otherwise stable. What is the most appropriate next step in management? A. Obtain a left lower extremity venous ultrasound B. Obtain a chest CT scan with contrast C. Obtain a bedside transthoracic echocardiogram D. Check a d -dimer E. Empiric administration of IV unfractionated heparin

ANSWERS 21.1 A. Patients who present to a hospital not equipped for PCI who are more than 120 minutes from the nearest PCI-capable hospital should be given thrombolysis unless contraindicated. Angiography can then be performed, and PCI carried out if reperfusion is not complete. Trials of half-dose lytic and immediate PCI (called “facilitated PCI”) have not shown favorable results, and this strategy is not advocated. 21.2 E. This patient has a type B aortic dissection, which may be managed medically with IV metoprolol and IV nitroprusside. Intravenous labetalol does not reduce shear force of blood along the arterial wall as well as metoprolol, and nitroglycerine is generally considered inferior to nitroprusside for afterload reduction. Surgery is not required unless the aneurysm continues to extend or there are complications, and IV heparin is contraindicated. 21.3 C. This young man has a simple, uncomplicated pneumothorax, which may be monitored with serial CXR for stability. No urgent intervention is required, and 100% oxygen may help it resorb. Needle thoracotomy and chest tube are therapies reserved for tension pneumothorax.

294

CASE FILES: CAr d IO LOg y

21.4 B. This patient likely has a pulmonary embolism, caused by a left lower extremity deep-vein thrombosis (DVT). The next best step is to obtain a chest CT with contrast to confirm the diagnosis. In patients with renal insufficiency, a venous ultrasound to confirm a DVT may be sufficient to infer a diagnosis, but is less ideal. A bedside echocardiogram is typically unnecessary unless the right heart needs to be assessed in a patient with signs of hemodynamic instability. A d -dimer is reasonable to rule out a PE in a patient with low to intermediate probability for PE; however, this may be falsely elevated in this patient with lung cancer. IV heparin should not be administered without a diagnosis if this can be avoided.

CLINCIAL PEARLS C

The evaluation o chest pain shoul be tailo e to in ivi ual patients, consi e in thei histo an the settin in which the a e evaluate .

C

“Can’t miss” potentiall li e-th eatenin ia noses inclu e acute m oca ial in a ction, ao tic issection, tension pneumotho ax, an pulmona embolism.

C

An ECg shoul be one within 10 minutes o a ival to the Ed to ule out an acute MI.

C

A bloo p essu e i e ential o >20 mmH in both a ms is su estive o an ao tic issection, an a CT an io am shoul be u entl o e e to exclu e this ia nosis.

C

The t eatment o STEMI is imme iate evascula ization with PCI o th ombol sis. NSTEMI ma be mana e with a ela e invasive st ate o PCI within 72 hou s an shoul not be iven th ombol tics.

REFERENCES Hagan PG, Nienaber CA, Isselbacher EM, et al. The International Registry of Acute Aortic Dissection (IRAD): new insights into an old disease. JA MA . 2000;283:897–903. Klinkman MS, Stevens D, Gorenflo DW. Episodes of care for chest pain: a preliminary report from MIRNET. Michigan Research Network. J Fam Pract. 1994;38:345. Panju AA, Hemmelgarn BR, Guyatt GH, et al. The rational clinical examination. Is this patient having a myocardial infarction? JA MA . 1998;280:1256–1263. Sahn SA, Heffner JE. Spontaneous pneumothorax. N Engl J Med. 2000:342:868–874. Swap CJ, Nagurney JT. Value and limitations of chest pain history in the evaluation of patients with suspected acute coronary syndromes. JA MA . 2005;294:2623–2629.

CASE 22 A 21- ear-ol female college stu ent is seeing ou in the outpatient clinic for evaluation of two episo es of “fainting” in the past month. Both episo es were witnesse . The first occurre after she stoo up from eating a large inner while out with frien s. She was caught b one frien as she was falling an suffere no significant trauma other than a scrape elbow. She was unconscious for 5–10 secon s, i not have an tonic-clonic muscle movements, i not lose bowel or bla er control, an regaine consciousness with some confusion about what happene but otherwise with no significant cognitive impairment. She reporte lighthea e ness prior to loss of consciousness. The other episo e was similar to the first an occurre uring a hot a while she was out at a picnic with her famil . She was stan ing for a perio of time when she recalle feeling lighthea e an then ha a loss of consciousness with loss of postural tone. She has no me ical histor , an her famil histor is negative for su en or unexplaine eath an is otherwise noncontributor . She enies an fever, chills, sweats, hea aches, iarrhea, constipation, palpitations, chest iscomfort, shortness of breath, weakness or numbness, change in cognition or memor , or change in motor function. She takes no me ications or herbal supplements. She is not on a special iet. She has never smoke an oes not use an rugs. She rinks one or two alcoholic beverages per weeken . She is not sexuall active. She oes not regularl exercise an she rinks four to five cups of coffee ail . On examination she is a health , normal-appearing woman in no acute istress. Her temperature is 98°F, heart rate 70 bpm, an bloo pressure 110/ 60 mmHg. A comprehensive ph sical examination is entirel within normal limits. A complete bloo count an a basic metabolic panel are within normal limits. ECG shows sinus rh thm 70 bpm, normal axis, normal PR an QT intervals, normal QRS uration, an is generall unremarkable without evi ence of preexcitation or ST abnormalities. C C C

What is the most likel iagnosis? What is the best next iagnostic step? What is the best next step in therap ?

296

CASE FILES: CARd IO LOGy

ANSWER TO CASE 22: Syncope Summary: A 21-year-old healthy woman presents to the outpatient clinic with two episodes of loss of consciousness. The first episode occurred when she stood up after eating a large meal. The second episode occurred during a particularly hot day while standing for a period of time. Review of systems was negative, without palpitations, chest pain, or shortness of breath. She has no past medical history, and family history is noncontributory. Her exam is unremarkable. Her CBC and BMP were within normal limits. Her ECG is normal. • Most likely diagnosis: Reflex (neurally mediated) syncope. • N ext diagnostic step: Orthostatic vital signs and tilt table test. • N ext step in therapy: Adequate fluid hydration and avoidance of triggers.

ANALYSIS Objectives 1. Know the diagnostic approach to syncope. 2. Understand that syncope can be caused by varying etiologies and mechanisms that have a common result of loss of consciousness. 3. Understand that treatment is dependent on the cause. 4. Be familiar with the different modalities of treatment for reflex syncope, including first-line lifestyle modification and supportive measures.

Considerations This is a 21-year-old woman who is otherwise healthy. Her history is consistent with reflex (neurally mediated) syncope. Her episodes of loss of consciousness were preceded by lightheadedness. The first episode occurred after a change in posture, and may have been exacerbated by the large meal and resultant shunting of blood to the digestive system. The second episode occurred after standing for a period of time and may have been exacerbated by the heat. If the patient had a sudden loss of consciousness without preceding symptoms or without precipitating factors, then this would be more suggestive of an arrhythmic cause such as a tachy- or bradyarrhythmia. She denies palpitations or chest discomfort, which would also be worrisome for an arrhythmic cause. Her ECG is normal without findings indicating arrhythmia development (such as a long QT interval or ST elevations consistent with a Brugada pattern). She denies shortness of breath, and her cardiac exam is normal, suggesting that a significant valvular abnormality, such as severe aortic stenosis, or other structural abnormality, such as hypertrophic obstructive cardiomyopathy, is unlikely. Her carotid exam did not reveal any bruit, and the scenario is unlikely to be due to bilateral carotid artery stenosis. Her witnessed episodes were devoid of tonic-clonic movements or loss of bowel or

SECTIO N II: CLINICAL CASES

297

bladder control, which would be suspicious for seizure activity. Her episodes resulted in minor skin abrasions, suggesting that pseudoseizure or factitious behavior is less likely. A normal CBC and BMP point away from metabolic derangements affecting neurocognitive function. Likewise, she denies significant alcohol or drug use. Overall, her history and evaluation in the office thus far does not raise any red flags or overtly dangerous etiologies and are more likely consistent with reflex syncope. The diagnosis can be strengthened by bedside orthostatic vital signs and a formal tilt table test. If a vasovagal response is seen, then supportive measures and lifestyle modification are first-line therapy.

APPROACH TO: Syncope DEFINITIONS SYN COPE: Abrupt loss of consciousness, due to global cerebral hypoperfusion, with a concomitant loss of postural tone. VASOVAGAL RESPON SE: Vasovagal response on tilt table test is an abrupt decrease in heart rate and blood pressure with loss of consciousness and loss of postural tone. ORTHOSTATIC HYPOTEN SION : Orthostatic hypotension on tilt test is defined as a decrease in blood pressure of 20 mmHg systolic and/or 10 mmHg diastolic. TABLE TILT TEST: An examination where the patient’s BP and HR are assessed while lying on a table that is slowly tilted upwards.

CLINICAL APPROACH Overview Syncope is a transient loss of consciousness due to global cerebral hypoperfusion, with a concomitant loss of postural tone. It is abrupt in nature, with a short duration and a spontaneous complete recovery. Other causes of transient loss of consciousness not due to cerebral hypoperfusion, such as seizure or trauma, are therefore, by definition, distinct from syncope. Presyncope, also called “near-syncope,” is the prodrome of syncope without total loss of consciousness. Prodromal symptoms include lightheadedness, nausea, and sweating. Syncope can have cardiac or noncardiac causes. The overall prognosis depends on the cause of the syncope.

Burden Syncope It is estimated that up to 3% of men and 3.5% of women will experience syncope at some point during their lifetime. The prevalence of syncope increases in older age groups (0.7% in patients between 35 and 44 years old vs 4–6% in patients >75 years old). Episodes of transient loss of consciousness or syncope account for up to 3% of ER visits and up to 6% of hospital admissions. The majority of cases

298

CASE FILES: CARd IO LOGy

of transient loss of consciousness are attributed to syncope or similar mimicking conditions. The financial burden of syncope is difficult to estimate because of the significant indirect costs of syncope such as loss of earnings of patients or family members. The direct costs associated with syncope are due mostly to the high rate of hospitalization after emergency department evaluation.

Classification The European Society of Cardiology has published a classification scheme for syncope with three main categories: reflex syncope, syncope due to orthostatic hypotension, and cardiac syncope. Reflex (neurally mediated) syncope: This includes mechanisms such as vasovagal syncope, situational syncope, carotid sinus syncope, and atypical forms. Vasovagal syncope can be mediated by emotional distress such as pain or the sight of blood, or by orthostatic stress (postural challenge). Examples of situational syncope are syncope occurring after coughing (cough syncope), after micturition (postmicturition syncope), or after eating (postprandial syncope). Carotid sinus syncope is due to hypersensitivity of carotid sinus baroreceptors and can be triggered by mechanical stress (such as a tight shirt collar, or physical palpation such as with a carotid massage). Syncope due to orthostatic hypotension: This includes syncope due to autonomic failure. Autonomic failure can be primary (such as multiple system atrophy or pure autonomic failure) or secondary (such as due to diabetes or spinal cord injuries). Orthostatic hypotension can also result from alcohol use, or from medications such as diuretics and vasodilators. Finally, intravascular volume depletion (such as from vomiting, diarrhea, or hemorrhage) can result in orthostatic hypotension. Cardiac syncope: This includes syncope due to arrhythmia or structural heart disease. Significant bradycardia (such as from sinus node dysfunction or AV nodal conduction disease) can result in syncope. Conversely, significant tachycardia (from either a supraventricular or ventricular origin) can result in syncope. Structural heart disease can also cause syncope; hypertrophic obstructive cardiomyopathy, cardiac valvular abnormalities, and cardiac tamponade are examples of cardiac syncope.

DIAGNOSIS History A thorough history should be taken for each patient presenting with loss of consciousness. The diagnosis of syncope as the cause can sometimes be difficult. Complete loss of consciousness, rapid onset, short duration, complete and spontaneous recovery, and loss of postural tone all point to syncope as the cause of loss of consciousness. The prodromal symptoms of syncope can further help elucidate the etiology. Symptoms of postural intolerance are related to cerebral hypoxia resulting from decreased cerebral perfusion. Symptoms related to cerebral hypoxia include lightheadedness, dizziness, imbalance, tunnel vision, blurriness, spotted visual field, and headache. Of note, patients may find that it is possible to abort these symptoms by assuming a sitting or supine posture. The occurrence and severity of symptoms are influenced not only by the quantitative drop of blood pressure but also by the rapidity of blood pressure decline. However, it is noteworthy that elderly patients

SECTIO N II: CLINICAL CASES

299

with chronic orthostatic hypotension may have adaptive mechanisms of cerebral autoregulation such that symptoms may not occur even with extensive fall in blood pressure. Conversely, symptoms such as palpitations or chest pain may point toward a cardiac cause of syncope. Alarming features indicating a high-risk scenario include syncope during exertion, syncope while lying down, family history of sudden cardiac death, or slow recovery from syncope. Appropriate and timely consultation should be obtained if clinical suspicion of high-risk scenarios is present.

Physical Exam ination A careful, comprehensive physical examination is essential. Blood pressure should be checked in both arms, and in the supine and standing positions (orthostatic vital signs). Helpful exam findings include signs of dehydration, and presence of carotid bruits, cardiac murmurs, or varicose veins.

Additional Testing Depending on the history and physical exam findings, further testing can be done. These include 12-lead ECG and cardiac telemetry monitoring (inpatient or ambulatory) when there is concern for arrhythmia or ischemia, carotid sinus massage for a concern of hypersensitivity, echocardiogram to look for structural cardiac abnormalities, or tilt table testing for further evaluation of possible reflex syncope or orthostatic hypotension. Advanced, specialized testing such as circulatory hemodynamics or blood volume measurement can also be useful in some cases. The tilt table test is a provocative test that moves a patient from a supine position to an upright position (70° tilt) while strapped to a table, for up to 45 minutes. Blood pressure and heart rate are intermittently or continuously measured, and continuous or intermittent ECG monitoring is done. The tilt table test is used to examine autonomic neural regulation of cardiovascular orthostatic responses. Blood pressure and heart rate responses can be helpful in diagnosing the mechanism of syncope such as reflex syncope or orthostatic hypotension. Examples of tilt table results are shown in Figures 22-1 and 22-2. Evaluation of the tilt table test performance showed a sensitivity of 32–85%, specificity of 75–93%, reproducibility of 62–85%, and a false-negative rate of 14%. Risk stratification scores have been published in order to identify high-risk patients needing urgent evaluation and treatment. For example, the use of the San Francisco Syncope Rule showed that patients with one or more risk factors were at higher risk of serious events at 7 days, with a sensitivity of 98% and specificity of 56%. Risk factors included were abnormal ECG, congestive heart failure, shortness of breath, hematocrit 100 packs per year in total. He denies chest pain, orthopnea, PND, or productive cough, but he does report moderate weight loss over the past year. His examination reveals a thin man in no distress who smells of smoke. He is afebrile and normotensive with a room air saturation of 92%. He has diffuse end expiratory wheezing and distant heart sounds without murmurs. His legs are warm and without edema. His ECG is remarkable for sinus rhythm with low QRS voltage and slight right-axis deviation. His labs are all within normal limits, including a B-type natriuretic peptide level. From what you know about this patient right now, in addition to smoking cessation counseling, what would you recommend next? A. B. C. D. E.

Furosemide Inhaled ipratropium bromide and albuterol Azithromycin Metoprolol Heparin

SECTIO N II: CLINICAL CASES

311

23.3 A 58-year-old man presents to your office for evaluation of his 6-month complaint of exertional dyspnea. He currently becomes winded after climbing one flight of stairs, but he denies any other symptoms. He does not see a doctor regularly and takes no medications on a regular basis. He is a truck driver who was a one-pack/day smoker for 25 years until he quit 5 years ago after his brother was diagnosed with smoking-related lung cancer. On physical examination his vitals are as follows: BP 165/105 mmHg, pulse 80 bpm, respirations 16/minute, room air saturation 98%, and BMI 28 kg/m2. His cardiopulmonary examination is normal aside from trace ankle edema and an estimated jugular venous pressure of 9 cm H 2O. His ECG reveals sinus rhythm with voltage criteria for left ventricular hypertrophy. His basic laboratory data are normal with the exception of a BNP level of 305 pg/mL. His chest x-ray reveals a borderline enlarged cardiac shadow and normal lungs. You have him return after spirometry and an echocardiogram; the spirometry demonstrated mildly reduced FEV1 and forced vital capacity (FVC) with a normal ratio, and his echocardiogram was normal aside from stage 2 diastolic dysfunction. What is the most appropriate next step? A. Arrange for a CT scan of the chest to exclude interstitial lung disease B. Begin treatment with an ACE inhibitor and a thiazide-type diuretic for hypertensive heart disease C. Begin inhaled albuterol and ipratropium bromide in combination for COPD D. Arrange for a cardiopulmonary exercise test to further assess his dyspnea E. Arrange for a pharmacologic myocardial perfusion imaging study to rule out coronary artery disease

ANSWERS 23.1 C. Echocardiography. This patient has a history and clinical examination that is classic for severe aortic stenosis; an echocardiogram would confirm this diagnosis, and this would be the appropriate next step. Spirometry would most likely be normal or reveal a mildly restrictive pattern, which is common in patients with left atrial hypertension caused by left-sided heart disease. Cardiopulmonary stress testing would almost certainly demonstrate reduced VO 2 secondary to her valvular heart disease and would not add any additional information. A BNP in her case would likely fall into the nondiagnostic range and would not facilitate the diagnosis. Her carotid artery findings are due to her aortic stenosis; isolated carotid stenosis would not be expected to cause dyspnea. 23.2 B. This patient almost certainly has emphysema caused by cigarette smoking, a diagnosis that is supported by his history and physical examination. Smoking cessation is of utmost importance for this patient, but inhaled bronchodilator therapy would also be appropriate. There is little in his history or physical exam to suggest the presence of heart failure, ischemic heart disease, pulmonary embolism, or pneumonia.

312

CASE FILES: CAr d IO LOg y

23.3 B. Treat for hypertensive heart disease. This patient presents with stage 2 hypertension with evidence of end-organ involvement (left ventricular hypertrophy) and increased left atrial pressure (stage 2 diastolic dysfunction). Although he does not have overt heart failure, a dual-agent antihypertensive treatment regimen that includes a diuretic would be the most appropriate next step. With a normal chest film and lung examination, there would be no clear role for a CT scan of the chest. For similar reasons and with no obstruction on spirometry, there would be no indication for COPD treatment with inhaled bronchodilators. A cardiopulmonary exercise test seems unnecessary as the patient’s hypertensive heart disease appears to explain his dyspnea. Although this patient does have risk factors for coronary disease, and myocardial ischemia can cause dyspnea, he does not report chest pain, and his overall clinic picture is more consistent with hypertensive heart disease.

CLINICAL PEARLS C

The cause of ch onic spnea can be ete mine f om the histo ph sical examination in app oximatel two-thi s of cases.

an

C

Fo the ia nosis of hea t failu e in nonobese patients, BNP values 400 p / mL have excellent ne ative an positive p eictive value, espectivel .

C

A no mal t anstho acic echoca io am can confi entl exclu e a numbe of ca iovascula con itions associate with spnea.

C

Ca iopulmona exe cise testin can be ext emel helpful in fin in a ia nosis in patients with spnea who o not have an obvious cause afte p ima investi ation.

REFERENCES Maisel AS. B-Type natriuretic peptide levels: diagnostic and prognostic in congestive heart failure. What’s next? Circulation. 2002;105:2328–2331. Milani RV, Lavie CJ, Mehra MR, Ventura HO. Understanding the basics of cardiopulmonary exercise testing. Mayo Clin Proc. 2006;81(12):1603–1611. Nishino T. Dyspnoea: underlying mechanisms and treatment. Br J A naesth. 2011;106(4):463–474. Pratter MR, Curley FJ, Dubois J, Irwin RS. Cause and evaluation of chronic dyspnea in a pulmonary disease clinic. A rch Intern Med. 1989;149(10):2277.

CASE 24 A 70- ear-old woman presents to our o ice or preoperative cardiovascular risk assessment prior to elective hip replacement. Her medical histor is remarkable or h pertension, diabetes mellitus, chronic kidne disease, obesit , and osteoarthritis. She has no histor o heart disease. She denies chest pain, orthopnea, or PND, but she does report mild d spnea with exertion that has graduall progressed over the past 6 months. This corresponds with reduced ph sical activit secondar to hip pain. She can walk unassisted at a slow pace or one cit block be ore stopping because o hip pain. The patient is a retired school teacher who has never smoked cigarettes. She drinks alcohol rarel and has no known drug allergies. Her medications include aspirin, lisinopril, metoprolol, glargine insulin, rosuvastatin, naproxen as needed, and a multivitamin. Her examination is remarkable or the ollowing: blood pressure 152/ 82 mmHg, pulse 78 bpm, respirations 16 breaths/ min, and BMI 40 kg/ m 2. She is an obese Caucasian woman in no distress. There are no carotid bruits or obvious jugular venous distention. Cardiac auscultation is normal, and her chest is clear. She has mild bilateral ankle edema. Her o ice ECG is shown (Figure 24-1). Laborator data include the ollowing: sodium 135 mEq/ L, potassium 4.1 mEq/ L, BUN 32 mg/ dL, creatinine 2.1 mg/ dL, and hemoglobin 10.5 g/ dL. C C C

How would ou consult the patient on the cardiovascular risk in surger ? What additional testing would be help ul? What can be done to reduce her perioperative cardiac risk?

Re fe rre d by:

Unconfirme d

V1

V4

a VL

V2

V5

a VF

V3

V6

I

a VR

II

III

V1

II

V5

Figure 24-1. ECG or the main subject o this case.

SECTIO N II: CLINICAL CASES

315

ANSWER TO CASE 24: Cardiac Risk Assessment Prior to Noncardiac Surgery Summary: The patient is a 70-year-old obese woman with diabetes and limited functional capacity who is planning to have elective hip replacement. She has no history of heart disease, and she denies symptoms that strongly suggest an active cardiovascular condition; however, she does report NYHA class II dyspnea, which may simply be due to deconditioning and obesity but can also be an “angina equivalent” symptom in women and patients with diabetes. Her physical examination is limited by her obesity, but it does not suggest the presence of structural heart disease or decompensated heart failure. Her resting ECG reveals nonspecific ST and T wave abnormalities possibly related to left ventricular hypertrophy. Recent laboratory assessment confirms the presence of stage 4 chronic kidney disease (glomerular filtration rate 23 mL/min). In this case unit we will review the process of risk stratification from a cardiologist’s perspective, highlighting an approach to risk stratification that is jointly endorsed by the American Heart Association (AHA) and the American College of Cardiology (ACC). Management guidelines regarding preoperative risk assessment are also published by the American College of Physicians (ACP) and a number of subspecialty-oriented academic societies. Although these guidelines vary on certain points, there is generally agreement between them regarding major themes such as the importance of clinical risk prediction models to guide the use of noninvasive imaging and the judicious use of preoperative coronary revascularization. • Perioperative cardiac event risk: At risk for major perioperative cardiac event. • Additional testing: Noninvasive assessment for myocardial ischemia. • Reducing perioperative cardiac risk: Optimization of her blood pressure control.

ANALYSIS Objectives 1. Understand the purpose of risk stratification prior to noncardiac surgery. 2. Know the clinical elements associated with increased perioperative cardiovascular risk. 3. Review the process of risk stratification, including when to order additional imaging to assess for myocardial ischemia. 4. Appreciate the limitations of the evidence related to perioperative risk reduction, including beta-blocker therapy and revascularization.

Considerations This patient’s risk for major adverse cardiac events related to hip replacement is at least moderate. She has several robust risk factors for ischemic heart disease, and

316

CASE FILES: CARDIO LOGy

her lack of suggestive symptoms may be due to her limited functional capacity and inability to provoke ischemia in her daily life. The concern is that the stress of surgery combined with the pain, anemia, and immobility that follows it might create the “perfect storm” for cardiovascular complications to occur. Additional information obtained from a noninvasive test to assess for myocardial ischemia will allow for better risk stratification and aid the patient and the surgeon with respect to decision making. If high-risk features are discovered, then the surgery should be postponed to allow for additional study and possibly revascularization if high-risk coronary disease is found. Alternatively, she may be found to have no evidence of occult ischemic heart disease, in which case she may proceed with surgery with little concern for major adverse cardiac events. The purpose of risk stratification is to quantify the risk for adverse cardiac events as accurately as possible in order to allow the treating surgeon, anesthesiologist, and patient to compare the cardiac risk related to surgery to the patient’s risk for harm without surgery. The risk stratification process also provides the cardiologist with an opportunity to make recommendations regarding risk attenuation and medical optimization. This patient has suboptimally controlled blood pressure, and she may benefit from optimization of her antihypertensive medications prior to surgery.

APPROACH TO: Preoperative Risk Assessment DEFINITIONS REVISED CARDIAC RISK IN DEX (RCRI): Prospectively validated scoring system consisting of six clinical risk variables that were found to be associated with increased cardiac risk by multivariable analysis. FU N CTION AL CAPACITY: Ability to perform physical activities without symptoms or assistance; poor functional capacity is associated with an approximate twofold increase in the risk for major adverse cardiac events (MACEs). Poor functional capacity is typically defined as an inability to achieve 4 metabolic equivalents (METs), which is roughly the effort required to walk four blocks or climb two flights of stairs without stopping. HIGH-RISK MYOCARDIAL ISCHEMIA: Findings on a noninvasive imaging study that imply the presence of severe left main, proximal left anterior descending, or multivessel coronary artery disease. Examples include exercise-induced hypotension, marked ST segment deviation with exercise, reduced left ventricular systolic function, left ventricular dilatation with exercise, or five or more ischemic myocardial segments during imaging. PERCU TAN EOU S CORON ARY IN TERVEN TION (PCI): Method of coronary artery revascularization that includes balloon angioplasty and stenting. Although PCI provides immediate coronary revascularization with relatively little risk, intracoronary stenting necessitates the use of potent antiplatelet therapy for

SECTIO N II: CLINICAL CASES

317

at least 4 weeks and ideally 1 year to prevent stent thrombosis. The issue of recent stenting and the need for antiplatelet therapy in patients who require noncardiac surgery has become a major challenge in the field of perioperative medicine. DU AL-AN TIPLATELET THERAPY (DAPT): The combination of aspirin and a second, more potent antiplatelet agent prescribed to prevent acute thrombosis of intracoronary stents. Current agents include clopidogrel, prasugrel, and ticagrelor. ACU TE STEN T THROMBOSIS: Sudden thrombosis of an intracoronary stent, typically triggered by premature withdrawal of dual antiplatelet therapy or poor apposition of the stent and the coronary arterial wall. Patients with intracoronary stents are vulnerable to stent thrombosis in the perioperative period following preoperative cessation of antiplatelet therapy and the transient increase in platelet activation and hypercoagulability that accompanies the stress of surgery. Stent thrombosis almost always results in myocardial infarction, and the mortality associated with acute postoperative stent thrombosis can approach 50%.

CLINICAL APPROACH Overview Cardiovascular risk assessment and management prior to noncardiac surgery can be challenging. Part of the challenge relates to the relative paucity of prospective data regarding benefit from therapies intended to specifically reduce risk such as coronary revascularization and beta-blocker therapy. Thus, the focus of risk assessment is to define risk so that the patient and surgeon may weigh the risks of surgery against the risk of not having surgery. The preoperative visit also affords the cardiologist an opportunity to identify and treat previously undiagnosed heart disease in at-risk patients and to optimize established cardiovascular diseases such as heart failure, ischemic heart disease, and hypertension prior to surgery in order to reduce risk. The following paragraphs will review a stepwise approach to preoperative risk assessment that is recommended by the ACC/AHA guidelines (Figure 24-2).

Clinical Risk Assessm ent The risk for cardiovascular events related to noncardiac surgeries can be attributed to patient-specific factors and surgery-specific factors. Patient-specific factors include major active cardiovascular signs and symptoms, medical diagnoses such as cardiovascular disease, or its risk factors and limited functional capacity. Surgeryspecific factors include the length of the surgery, potential for blood loss, and risk for major fluctuations in intravascular volume or hemodynamic measures such as heart rate or blood pressure. Patients with symptoms suggestive of active severe cardiovascular disease are at high risk for perioperative cardiac events related to noncardiac surgery. These major risk predictors include unstable coronary syndromes, decompensated heart failure, severe valvular heart disease, and dangerous arrhythmias (Table 24-1). In these cases it is usually appropriate to delay surgery to allow the active cardiac issue to be properly treated. The obvious exception would be the high-risk patient who requires emergency surgery to prevent death or major disability; in those cases

318

CASE FILES: CARDIO LOGy

1

Eme rge ncy s urge ry?

Ye s

To OR with clos e pe riope ra tive monitoring

Ye s

Pos tpone OR a nd tre a t a ctive ca rdia c proble ms

No

2

Ma jor-ris k pre dictiors ? No

Ye s

Low-ris k s urge ry?

3

To OR a s pla nne d

No

4

Ye s

Functiona l ca pa city ≥ 4 METs with no symptoms ?

To OR a s pla nne d

No

5

RCRI 0

RCRI 1-2

Inte rme dia te ris k s urge ry

To OR a s pla nne d

High-ris k s urge ry

RCRI ≥ 3

Inte rme dia te ris k s urge ry

To OR or cons ide r noninva s ive te s ting if re s ults will a lte r ma na ge me nt

High-ris k s urge ry

Cons ide r te s ting if re s ults will a lte r ma na ge me nt

Figure 24-2. Stepwise approach to preoperative cardiac risk assessment based on current guidelines (Fleisher et al. 2009). (OR, operating room; METs, metabolic equivalents; RCRI, revised cardiac risk index.)

Table 24-1 • MAJOR CLINICAL PREDICTORS OF ADVERSE CARDIAC EVENTS RELATED TO NONCARDIAC SURGERY Clinical Risk Predictor

Definition

Recent m ocardial in arction

M ocardial in arction in past 4–6 weeks

Severe angina

Unstable angina or CCS class 3–4 angina

Decompensated heart ailure

NyHA class IV s mptoms Exam evidence o volume overload and/ or congestion Newl diagnosed heart ailure

Severe valvular heart disease Severe aortic stenosis Severe mitral stenosis Severe aortic or mitral regurgitation

S mptoms, AVA < 1.0 cm 2, mean gradient ≥ 40 mmHg S mptoms, MVA < 1.0 cm 2, mean gradient ≥ 10 mmHg Severe s mptoms or heart ailure

Severe arrh thmia

High-grade AV block (3rd-degree, 2nd-degree Mobitz 2) Ventricular tach cardia Atrial ibrillation or lutter with poorl controlled rate

Abbreviations: AV, atrioventricular; AVA, aortic valve area; CCS, Canadian Cardiovascular Societ ; MVA, mitral valve area; NyHA, New york Heart Association.

SECTIO N II: CLINICAL CASES

319

Table 24-2 • PREDICTORS OF INCREASED CARDIAC RISK RELATED TO NONCARDIAC SURGERIES Risk Predictor

Definition

High-risk surger

Intraperitoneal, intrathoracic, suprainguinal vascular

Ischemic heart disease

Prior m ocardial in arction or positive stress test CCS class 1–2 angina or nitrate use Pathologic Q waves on ECG

Heart ailure

Histor o prior clinical heart ailure Reduced LVEF

Diabetes mellitus

Insulin treatment

Kidne disease

Serum creatinine > 2.0 mg/ dL

Cerebrovascular disease

Histor o stroke or TIA

Abbreviations: CCS, Canadian Cardiovascular Societ ; LVEF, le t ventricular ejection raction; RCRI, revised cardiac risk index; TIA, transient ischemic attack. [Data from Lee TH, Marcantonio ER, Mangione CM, et al. Derivation and prospective validation of a simple index for prediction of cardiac risk of major noncardiac surgery. Circulation. 1999;100(10):1043.]

surgery is often performed with close inpatient monitoring and comanagement of the patient by the surgical and cardiac teams in an intensive care setting. Patients without major cardiac risk predictors may still be at relatively high risk for adverse cardiac events related to noncardiac surgery. Identifying these patients prior to surgery is difficult because they may not have overt symptoms or a previously diagnosed cardiovascular problem. Multivariate analyses performed on cohorts of patients undergoing noncardiac surgery in the 1970s identified several clinical factors that were individually associated with an increased risk for postoperative cardiovascular events. In the decades that followed variables were added and others refined, resulting in the creation of several risk stratification models such as the revised cardiac risk index (RCRI). The RCRI employs six equally weighted clinical features that are predictive of postoperative cardiac events (Table 24-2). Each feature is assigned one point, and the points are added to produce the RCRI. The validity of the RCRI as a risk predictor has been prospectively confirmed in cohorts of patients undergoing a wide range of noncardiac surgeries (Table 24-3). Table 24-3 • PERFORMANCE OF THE REVISED CARDIAC RISK INDEX (RCRI). RCRI Score

Major Complications (MI, Pulmonary Edema, VF, CHB)

Mortality and Morbidity (Death, MI, Nonfatal Cardiac Arrest)

Event Rate (%)

Event Rate (%)

95% CI

95% CI

0.4

0.05–1.5

0.4

0.1–0.8

1

0.9

0.3–2.1

1.0

0.5–1.4

2

6.6

3.9–10.3

2.4

1.3–3.5

≥3

11.0

5.8–18.4

5.4

2.8–7.9

Abbreviations: CHB, complete heart block; MI, m ocardial in arction; VF, ventricular f brillation. Note: “Major Complications” data were obtained rom Lee et al. (1999); “Mortalit and Morbidit ” data were obtained rom Devereau et al. (2005).

320

CASE FILES: CARDIO LOGy

The RCRI performs well for most patients undergoing noncardiac surgery, particularly thoracic and abdominal surgeries. It may slightly overestimate risk in patients undergoing orthopedic surgery and tends to underestimate the risk associated with major vascular surgery. Nonetheless, its generally accurate performance and simplicity have made the RCRI a widely used tool in the assessment of preoperative cardiac risk. Functional capacity is also another important factor in the assessment of risk. Patients with reduced functional capacity have a nearly twofold increase in the risk for adverse postoperative cardiac events compared to those who have better physical ability. Poor functional capacity is generally defined as the inability to walk four city blocks or climb two flights of stairs without rest or assistance. This level of exercise is equivalent to four metabolic equivalents (METs). Surgery-specific risk is driven by the nature of the surgery and the urgency with which the surgery needs to be performed. Low-risk surgeries include most ambulatory outpatient surgeries, superficial surgeries (dermatologic and breast procedures), endoscopic procedures, and cataract surgery. These procedures are associated with a risk for death and nonfatal myocardial infarction of 10%. For this reason moderate- or high-risk patients, particularly those with poor functional capacity, may benefit from noninvasive assessment for occult myocardial ischemia. Noninvasive assessment for myocardial ischemia includes exercise testing and imaging studies that employ pharmacologic agents to induce coronary vasodilatation or increase heart rate and contractility. Exercise testing is always preferable to pharmacologic stimulation in patients who can safely exercise, because functional capacity itself aids in the prediction of cardiac risk. These studies are most helpful when they are normal or have findings suggestive of high-risk coronary artery disease (reduced left ventricular systolic function, findings consistent with left main, proximal left anterior descending, or three-vessel coronary disease). Normal noninvasive ischemic studies have excellent negative predictive value, and studies with high-risk findings generally warrant surgical delay to allow for coronary angiography and revascularization if appropriate.

SECTIO N II: CLINICAL CASES

321

Abnormal studies without high-risk features have limited positive predictive value for postoperative cardiac events; these studies may or may not be associated with a level of risk greater than that provided by the RCRI. This variability is due to the fact that test abnormalities vary with respect to their implied level of risk, and all imaging studies have a defined false-positive rate. For example, an intermediaterisk patient with an adenosine myocardial perfusion study suggestive of two apical segments of myocardial ischemia may not be at any greater risk for adverse effects because this finding might be a false positive (breast attenuation), or it may be a true positive but indicative of limited ischemia that, if properly medically managed, may not be associated with a greater risk for perioperative events. Noninvasive assessment for myocardial ischemia is usually performed with an imaging modality such as echocardiography or nuclear imaging with single-photon emission computed tomography (SPECT) or positron emission tomography (PET). There is no clear consensus regarding which modality is better, and both provide comparable information. Echocardiography has the advantage of providing information regarding valvular function, structural cardiac abnormalities, and pulmonary artery pressure in patients who raise clinical concern for these disorders. In patients without such concerns (eg, no murmurs, unexplained heart failure symptoms, or findings consistent with pulmonary hypertension), there is no role for routine resting echocardiography. Coronary angiography is seldom required for risk stratification in patients prior to noncardiac surgery. The major reason for this is the fact that preoperative coronary artery revascularization (percutaneous and surgical) has not been shown to reduce the risk for major adverse cardiac events in the peri- and postoperative period (see following section). The general rule applied to the use of angiography in the setting of preoperative risk assessment is to ask whether the patient has an indication for coronary angiography that is independent from his or her surgical evaluation. If the answer is “yes,” then an angiogram should be performed.

Risk Attenuation Once the magnitude of a patient’s risk for major adverse cardiac events has been determined, the cardiologist should make appropriate recommendations regarding ways to attenuate risk. This can be challenging as there are little data to support the routine use of therapies such as revascularization and beta-blockers for the sole purpose of reducing the risk for postoperative cardiac events. Rather, risk attenuation can be achieved by a careful assessment and proper medical optimization of the patient’s established cardiac issues and risk factors.

Coronary Artery Revascularization Preoperative coronary revascularization is appropriate in patients with symptoms consistent with acute coronary syndromes or severe chronic angina despite proper medical therapy. Similarly, patients with high-risk findings on a noninvasive study may derive a survival benefit from the revascularization of underlying left main or multivessel coronary disease, and thus revascularization would also be appropriate. However, these patients are the exception to the norm in the arena of preoperative cardiac risk, as a far greater number will be asymptomatic or have lower risk findings on noninvasive imaging.

322

CASE FILES: CARDIO LOGy

Although it would seem sensible to perform coronary angiography and revascularization prior to surgery in any patient with an abnormal stress test or chronic stable angina, well-designed prospective randomized trials have consistently shown no significant benefit with respect to postoperative event rates in patients randomized to revascularization. As mentioned previously, the presence of ischemia on a noninvasive imaging study has a relatively poor positive predictive value for postoperative cardiac events. Many patients with chronic coronary artery disease have well-developed collateral blood supply to affected arteries that protects them from myocardial injury during times of stress. Also, with modern anesthetic techniques and aggressive management of postoperative issues such as pain and elevated blood pressure, patients may not experience a large increase in myocardial oxygen demand. Revascularization itself may also introduce complexity and harm. Coronary artery stenting usually delays surgery and obligates the patient to a period of dual-antiplatelet therapy (DAPT). Premature interruption of DAPT to reduce the risk for operative bleeding increases the risk for stent thrombosis. Continuation of DAPT to prevent stent thrombosis may increase the risk for perioperative bleeding. Surgical coronary revascularization carries its own significant risk for morbidity and also typically delays noncardiac surgery.

Beta-Blocker Therapy Beta-blockers reduce blood pressure, heart rate, and myocardial oxygen demand. Beta-blocker therapy has clear benefit for patients with multiple cardiovascular disorders such as ischemic heart disease, stable angina, heart failure, and tachyarrhythmias, and their use in such patients prior to noncardiac surgery is clearly indicated. For this reason beta-blockers are continued throughout the perioperative period in patients who are already taking them and heart-rate–guided optimization of beta-blocker dosing in patients who have a clear indication for their use is also appropriate. Given the previously mentioned findings, there has been interest in the use of beta-blockers in asymptomatic, at-risk patients in the preoperative period to reduce operative risk. However, current clinical evidence does not support the routine use of beta-blockers in patients who do not otherwise have an indication for beta-blocker therapy. A series of contemporary Dutch trials reported a significant benefit from the preoperative use of bisoprolol in a population of relatively high-risk patients undergoing vascular surgery; however, the legitimacy of the authors’ findings was questioned and the results of these studies were ultimately discredited. Another large randomized trial of metoprolol succinate given just prior to surgery to reduce the risk of postoperative cardiac events resulted in an increased risk for hypotension and stroke in the metoprolol-treated patients. However, the adverse events in this trial may have been related to the use of relatively high doses of long-acting betablockers in drug-naïve patients treated just prior to surgery. A well-designed and honestly run trial of cautiously titrated oral beta-blocker therapy prior to surgery may ultimately resolve the beta-blocker issue, but as of now there is no evidencebased indication for the use of beta-blockers in patients to reduce the risk for postoperative cardiac events.

SECTIO N II: CLINICAL CASES

323

Surgery Following Intracoronary Stenting Occasionally patients who have recently undergone PCI with coronary artery stenting require noncardiac surgery. Although these patients have been revascularized, their recent PCI places them at risk for bleeding and perioperative myocardial infarction due to the use of dual-antiplatelet therapy (DAPT) and stent thrombosis, respectively. Current management guidelines recommend at least 12 months of uninterrupted DAPT following stenting to reduce the risk of stent thrombosis. Premature cessation of DAPT is associated with an increased risk for stent thrombosis, and this risk is magnified in the perioperative setting. Thus, withholding DAPT to prevent postoperative bleeding comes with a higher risk for stent thrombosis. The magnitude of this risk is time-dependent and highest in the first 6 weeks after PCI. The type of stent also matters; drug-eluting stents are felt to require a longer period of DAPT because they require more time to endothelialize than bare-metal stents. The precise risk for stent thrombosis is difficult to quantify, but what is well known is that stent thrombosis in the postoperative setting is devastating and associated with a mortality of approximately 40%. Prevention of postoperative stent thrombosis should begin with a dialog between the patient, cardiologist, and surgeon so that all parties involved clearly appreciate the risk involved. From a cardiac perspective, the safest option would be to proceed with surgery on DAPT and accept the potential for bleeding provided that the risk for fatal or uncontrollable bleeding is low. If continued DAPT is deemed too unsafe and the patient is far enough out from the scheduled PCI (>4 weeks after bare-metal stenting or >6 months after drug-eluting stenting), then the patient should remain on low-dose aspirin and stop taking the other antiplatelet agent prior to surgery. Surgery that needs to be performed soon after PCI poses a management challenge; such patients are often hospitalized for treatment with an intravenous short-acting antiplatelet agent or anticoagulant as the DAPT wears off. These “bridging” maneuvers are commonly done, but there use is empiric and not evidence-based.

CASE CORRELATION • See also Case 1 (acute coronary syndrome/STEMI), Case 2 (acute coronary syndrome/NSTEMI), Case 3 (cardiogenic shock), Case 6 (acute valvular regurgitation), Case 8 (hypertrophic obstructive cardiomyopathy), Case 10 (valvular stenosis), Case 16 (acute heart failure), and Case 17 (advanced heart failure).

COMPREHENSION QUESTIONS 24.1 All of the following factors are associated with an increased risk for perioperative cardiac events in patients undergoing non-cardiac surgery except A. History of myocardial infarction B. History of heart failure C. History of insulin-requiring diabetes D. History of atrial fibrillation E. History of moderate chronic kidney disease

324

CASE FILES: CARDIO LOGy

24.2 A 60-year-old man presents to the intensive care unit with massive hematemesis and hypovolemic shock. His medical history includes an ischemic cardiomyopathy with prior myocardial infarction and a left ventricular ejection fraction of 30%. He describes baseline CCS class 3 angina symptoms despite appropriate medical therapy, and his physical examination is remarkable for pallor, tachycardia, an S3 gallop, and faint rales in the lung bases. His ECG reveals sinus tachycardia with inferior Q waves and 2-mm horizontal ST segment depression in the precordial leads. Following transfusion and resuscitation with intravenous fluids, he has an endoscopy that reveals a large duodenal ulcer with a visibly bleeding vessel that cannot be controlled by endoscopic therapy. General surgery is consulted and operative management of his bleeding ulcer is scheduled. You are asked to see him for preoperative cardiac risk assessment. Which of the following recommendation is most appropriate? A. High risk for major cardiovascular complications; proceed with surgery immediately. B. High risk for major cardiovascular complications; consider coronary angiogram prior to surgery to define his coronary anatomy. C. High risk for major cardiovascular complications; consider noninvasive myocardial perfusion imaging to assess his burden of ischemia prior to surgery. D. High risk for major cardiovascular complications; double his home dose of carvedilol to lower his cardiac risk and proceed with surgery. E. High risk for major cardiovascular complications; recommend medical management only. 24.3 Which of the following statements regarding stress testing prior to noncardiac surgery is most accurate? A. Nuclear imaging is superior to echocardiography for the prediction of postoperative cardiac events. B. Vasodilator perfusion imaging is superior to treadmill exercise for the provocation of myocardial ischemia in preoperative patients. C. A normal noninvasive test for myocardial ischemia has excellent negative predictive value for postoperative cardiac events. D. Noninvasive testing for myocardial ischemia is indicated for better risk stratification in all moderate-risk patients with poor functional capacity. E. An abnormal noninvasive test for myocardial ischemia has excellent positive predictive value for postoperative cardiac events.

SECTIO N II: CLINICAL CASES

325

24.4 Which of the following clinical findings is not considered a major predictor of cardiac events prior to noncardiac surgery? A. First-degree AV block with a PR interval of 230 ms B. Aortic stenosis with a valve area of 0.8 cm2 C. Heart failure with an S3 gallop and pulmonary rales D. Coronary artery disease with a non-ST segment elevation myocardial infarction 3 weeks ago E. Atrial flutter with a ventricular rate of 150 bpm

ANSWERS 24.1 D. History of atrial fibrillation. Although atrial fibrillation with an uncontrolled ventricular rate would be considered a major clinical risk predictor, a history of atrial fibrillation alone is not associated with an increased risk for perioperative cardiac events. Ischemic heart disease, heart failure, insulinrequiring diabetes, and moderate chronic kidney disease are all associated with increased operative risk and are features of the revised cardiac risk index. 24.2 A. High risk for major cardiovascular complications; proceed with surgery immediately. This patient is experiencing a major gastrointestinal bleed that has failed endoscopic therapy. He carries a high risk of death due to exsanguination without surgery that certainly exceeds his cardiac risk related to surgery, even in the face of major risk predictors such as severe chronic angina and features of decompensated heart failure. Coronary angiography is not indicated because this patient is not an appropriate candidate for percutaneous or surgical coronary revascularization. Noninvasive imaging is not indicated because his cardiac risk is already obviously high and the results of the study will not change management. Although beta-blockers are indicated for the management of heart failure and angina, it would be contraindicated to escalate beta-blocker therapy in a hypotensive patient in shock. Avoiding surgery would likely carry a mortality risk related to bleeding that is greater than the mortality related to cardiac causes. 24.3 C. A normal noninvasive test for myocardial ischemia has excellent negative predictive value for postoperative cardiac events. Studies comparing nuclear to echocardiographic assessment of ischemia have demonstrated no clear superiority of either modality, but exercise stress is generally considered superior to pharmacologic stimulation. Although many patients with moderate-risk predictors and low functional class might benefit from additional risk stratification, noninvasive studies should be performed only if the results are likely to change management, and this is clearly not the case for every patient. Abnormal imaging studies have relatively poor positive predictive value for postoperative cardiac events.

326

CASE FILES: CARDIO LOGy

24.4 A. First-degree AV block with a PR interval of 230 ms. High-grade AV block is considered a major clinical risk marker and includes Mobitz type 2 second-degree AV block and third-degree AV block. Severe aortic stenosis, decompensated heart failure, recent myocardial infarction, and uncontrolled tachyarrhythmias are all major clinical risk indicators.

CLINICAL PEARLS C

Clinical risk assessment tools such as the revised cardiac risk index can accuratel estimate the risk or adverse cardiac events in most patients.

C

Noninvasive assessment or m ocardial ischemia is appropriate or moderate-risk patients with poor unctional status or high-risk patients undergoing moderate- or high-risk surgeries i the results will in luence management.

C

Coronar angiograph and revascularization is indicated in preoperative patients who have a separate and clear indication or revascularization that is independent rom their need or risk strati ication.

C

Preoperative beta-blocker therap is recommended onl in patients who have an independent indication or beta-blocker treatment. Doses should be titrated to a resting heart rate between 60 and 70 bpm, and h potension should be avoided.

C

Chronic cardiovascular issues such as stable angina, heart ailure, and tach arrh thmias should be medicall optimized prior to noncardiac surger .

C

Postoperative stent thrombosis is an uncommon but potentiall devastating complication o coronar arter disease treated with recent PCI.

REFERENCES Devereaux PJ, Goldman L, Cook DJ, et al. Perioperative cardiac events in patients undergoing noncardiac surgery: a review of the magnitude of the problem, the pathophysiology of the events and methods to estimate and communicate risk. Can Med A ssoc J. 2005;173(6):627–634. Fleisher LA, Beckman JA, Brown KA, et al. 2009 ACCF/AHA focused update on perioperative beta blockade incorporated into the ACC/AHA 2007 guidelines on perioperative cardiovascular evaluation and care for noncardiac surgery: a report of the American college of cardiology foundation/ American heart association task force on practice guidelines. Circulation. 2009;120(21):e169. Hawn MT, Graham LA, Richman JS, et al. Risk of major adverse cardiac events following noncardiac surgery in patients with coronary stents. JA MA . 2013;310(14):1462. Lee TH, Marcantonio ER, Mangione CM, et al. Derivation and prospective validation of a simple index for prediction of cardiac risk of major noncardiac surgery. Circulation. 1999;100(10):1043. McFalls EO, Ward HB, Moritz TE, et al. Coronary-artery revascularization before elective major vascular surgery. N Engl J Med. 2004;351(27):2795. POISE Study Group; Devereaux PJ, Yang H, Yusuf S, et al. Effects of extended-release metoprolol succinate in patients undergoing non-cardiac surgery (POISE trial): a randomised controlled trial. Lancet. 2008;371(9627):1839.

CASE 25 A 60- ea - ld-w man is bein seen b he p ima ca e p vide . This is he secnd visit, and she has neve established ca e with a ph sician p evi usl . She did n t ep t an c mplaints s mpt ms du in the p evi us visit, is n t takin an medicati ns, and exe cises n a e ula basis. He famil hist is si nificant f a b the with c na a te disease equi in su ical evascula izati n at a e 50 ea s, and he fathe died f c mplicati ns elated t c n estive hea t failu e in his mid-70s. The patient d es n t d ink alc h l use illicit d u s, but she has sm ked tw th ee ci a ettes pe da since the a e f 21 ea s. He vital si ns t da include a e ula hea t ate f 70 bpm and a seated bl d p essu e f 148/ 90 mmH . The emainde f he ph sical examinati n is un ema kable, and a c mp ehensive metab lic panel, includin a fastin lipid p file, was btained p i t the cu ent app intment. The ph sician eviews these esults with the patient, and n tes that while he se um elect l tes, bl d luc se, live t ansaminases, and c eatinine a e all within n mal limits, he fastin lipid p file is abn mal. She has a t tal ch leste l (TC) f 260 m / dL, a hi h-densit lip p tein (HDL) value f 42 m / dL, a t i l ce ide value f 135 m / dL, and a calculated l w-densit lip p tein (LDL) value f 176 m / dL. C C C

What is the m st likel dia n sis? What is the best next dia n stic step? What is the best next step in the ap ?

328

CASE FILES: CAr DIo Lo g y

ANSWEr To CASE 25: Preventive Cardiology Summary: This asymptomatic 60-year-old woman has an abnormal lipid profile and is a current smoker, and her risk for experiencing adverse outcomes related to coronary heart disease (CHD) needs to be determined. There is a history of coronary artery disease in her family, and this may also contribute to her risk profile. She is otherwise asymptomatic and exercises regularly, although her in-office blood pressure measurement (148/90 mmHg) is consistent with stage 1 hypertension. • Most likely diagnosis: Increased risk for development of CHD due to untreated hyperlipidemia, hypertension, and active tobacco use. • N ext diagnostic step: Determine risk profile for development of CHD and major cardiovascular events. • N ext step in therapy: Lifestyle modification to include healthy eating, regular aerobic exercise, smoking cessation, and moderate- or high-intensity statin therapy.

ANALYSIS Objectives 1. Recognize modifiable and nonmodifiable risk factors for development of coronary heart disease. 2. Be able to determine risk profiles for major adverse cardiovascular events due to CHD. 3. Know when to institute pharmacologic therapies for primary prevention of myocardial infarction and death from cardiovascular causes based on risk profile.

Considerations This asymptomatic 60-year-old woman has hyperlipidemia and hypertension, and actively smokes, and therefore has three major risk factors for CHD. Given that she is a woman and has pertinent, positive family history for CAD, her CHD risk could be estimated using the Reynolds risk score, but since this model requires the measurement of high-sensitivity C-reactive protein (hs-CRP), her risk can instead be calculated using the Framingham model (see below). According to the Framingham risk prediction tool, her 10-year risk of CHD is 11%, and a combination of therapeutic lifestyle changes and drug therapy with moderate- or high-intensity statin treatment would be indicated. In addition, lifestyle changes and/or drug therapy should be prescribed to reduce her systolic blood pressure and assist with smoking cessation.

SECTIo N II: CLINICAL CASES

329

APPr o ACH To : The Patient with Increased Risk for CHD DEFINITIONS HYPERLIPIDEMIA: An abnormally elevated level of any or all lipids/lipoproteins in the blood. DYSLIPIDEMIA: An abnormal quantity, either increased or decreased, of lipids in the blood PRIMARY PREVEN TION : Interventions targeted at preventing a disease process from occurring, thereby reducing both the prevalence and the incidence of the disease. SECON DARY PREVEN TION : Interventions targeted at preventing recurrent events from a disease process that has already been diagnosed. CORON ARY HEART DISEASE (CHD): Atherosclerotic disease of the epicardial coronary arteries and their subdivisions, manifested by angina pectoris, myocardial infarction, sudden cardiac death, and ischemic heart failure. CHD RISK EQU IVALEN T: Includes diabetes mellitus, chronic kidney disease, and noncoronary atherosclerotic disease, such as prior stroke, symptomatic carotid artery disease, peripheral vascular disease, and thoracic or abdominal aortic aneurysms. STATIN THERAPY: Drugs in the HMG-CoA reductase inhibitor family. Statins lower LDL cholesterol and have been shown in numerous large randomized clinical trials to reduce the risk of death and nonfatal cardiovascular events in patients who have or are at risk for developing cardiovascular disease. Their benefit is superior to other lipid-lowering agents, and some of this benefit has been attributed to their “pleotropic” effects, particularly the attenuation of systemic inflammation.

CLINICAL APPROACH Etiologies Several modifiable and nonmodifiable risk factors contribute to the development of CHD. Nonmodifiable factors include age, gender, and family history. Hypertension, dyslipidemia, diabetes, smoking, obesity, and kidney disease constitute major modifiable risk factors. It is important to note here that a significant proportion of cases of myocardial infarction (MI) and stroke occur in patients without hyperlipidemia, highlighting the fact that development of atherosclerotic arterial disease is a complex and multifactorial process. More research has, therefore, focused on risk markers for increased inflammation and abnormalities related to thrombosis and hemostasis, such as high-sensitivity C-reactive protein (hs-CRP), blood hom*ocysteine levels, and lipoprotein(a). Risk prediction models that incorporate these more novel risk markers have evolved and are being used more frequently as evidence of their utility grows.

330

CASE FILES: CAr DIo Lo g y

Clinical Presentation History The clinical history should stratify patients into those with overt or known cardiovascular disease and those with increased risk for the development of atherosclerotic disease. In the former group preventive strategies are focused on secondary prevention and slowing progression of established disease. In addition to risk factor modification, various pharmacologic strategies for secondary prevention are based on the nature and extent of disease and previous cardiovascular events, and include the use of antiplatelet agents, drugs that block the β-adrenergic system, and agents that attenuate the renin-angiotensin-aldosterone axis. Family and social history are of critical importance in establishing risk profiles, and assessment of the desire and motivation of the patient to attempt tobacco cessation should be undertaken at every visit. Physical Examination The physical examination may be completely normal, or may reveal evidence of significant coronary and peripheral vascular disease. Auscultation should be carried out to identify bruits over the femoral, renal, and carotid arteries. Signs of left ventricular dysfunction, such as elevated jugular venous pulsations, pulmonary rales, lateral displacement of the point of maximal impulse, and the presence of murmurs and extra heart sounds should be identified.

Treatm ent Risk Assessment The first step in the treatment decision process is to establish the individual patient’s risk profile. According to guidelines established by the third panel of the United States National Cholesterol Education Program Expert Panel on Detection, Evaluation, and Treatment of High Blood Cholesterol in Adults (Adult Treatment Panel III or ATP III), the risk of developing CHD should be estimated in those individuals with two or more risk factors for atherosclerotic cardiovascular disease. There are several established risk assessment tools available, and both the Framingham and Reynolds risk calculators are widely available in online formats: http://cvdrisk.nhlbi.nih.gov/calculator.asp; http://www.reynoldsriskscore.org. In the Framingham model, risk estimation is based on age, smoking status, blood pressure, diabetes, total cholesterol, and HDL cholesterol level. The final score estimates the 10-year CHD risk, and stratifies individuals into low (20%) risk. The Reynolds score incorporates all of the risk factors employed by the Framingham model, with the addition of premature family history of CHD (MI in a parent aged 20%, in which case the LDL-C goal should again be ≤100 mg/dL . Quite recently the American College of Cardiology (ACC) and the American Heart Association (AHA) released their 2013 guideline on the treatment of cholesterol to reduce cardiac event rates. Their recommendations are based on reanalysis of previously published data from >24 large primary and secondary prevention trials with the goal of optimizing statin usage in patients at highest risk. Their recommendations represent a bit of a paradigm shift away from targeting specific blood levels of HDL and LDL cholesterol in favor of targeting greater intensity of statin therapy in select populations of patients on the basis of their underlying risk for cardiac events (Table 25-1). Lifestyle Changes Therapeutic lifestyle changes (dietary modifications, weight loss, regular aerobic exercise) should be a universal recommendation for all patients with risk factors for cardiovascular disease. Patients who opt to begin regular exercise of their own volition have been shown to experience fewer major adverse cardiac events and have lower mortality rates than comparable subjects who do not regularly exercise. Patients should be advised to pursue at least 150 minutes of moderate intensity aerobic exercise or 75 minutes of intense aerobic exercise per week. Dietary modification should include emphasizing both portion control to reduce excessive caloric intake and dietary composition to avoid excessive intake of saturated fats and simple sugars. Protein intake should consist of nuts and lean meats, and, whenever possible, whole grains should be substituted for simpler carbohydrates. There are numerous heart-healthy diets for patients to choose from that emphasize these fundamental features, such as the dietary approach to stop hypertension (DASH) diet advocated by the National Heart, Lung and Blood Institute (NHLBI) (Table 25-2).

332

CASE FILES: CAr DIo Lo g y

Table 25-2 • EXAMPLE OF THE DIETARY APPROACH TO STOP HYPERTENSION (DASH) DIET Food Group

Servings per Day

Examples of Servings

Wh le

6–8

1 slice f b ead ½ cup f d ce eal ½ cup c ked ice

ains

pasta

Drug Therapy The cornerstone agents for the treatment of elevated LDL-C for the primary prevention of CHD are the HMG-CoA reductase inhibitors, also known as statins. Numerous large randomized clinical trials and meta-analyses have demonstrated that statin therapy reduces overall mortality and the rate of cardiovascular events by their LDL-C-lowering effect, and they should be employed in all high-risk individuals, barring intolerance or adverse reactions. The decision to use statins in those patients at low or intermediate risk needs to be individualized, and should involve a discussion with the patient about the anticipated absolute risk reductions with therapy. Interestingly, there is evidence to suggest that statins may reduce the rate of cardiovascular events in relatively healthy individuals with elevated hs-CRP levels despite having LDL-C levels below goal before initiation of therapy, although this concept remains controversial.

CASE CORRELATION • See also Case 1 (acute coronary syndrome/STEMI), Case 2 (acute coronary syndrome/NSTEMI), Case 4 (chronic coronary artery disease), and Case 5 (peripheral artery disease).

SECTIo N II: CLINICAL CASES

333

COMPREHENSION QUESTIONS Match the following intervention (A–E) to the clinical situation (questions 25.1 through 25.4): A. Initiate statin therapy B. Recommend a trial of lifestyle modification C. Use the Reynolds risk calculator to estimate long-term risk of CHD D. Increase statin dose 25.1 A 45-year-old woman is found to have an elevated total cholesterol, a normal hs-CRP, and an LDL-C of 140 mg/dL. She does not smoke, has stage 1 hypertension, and would like to know whether she will need to initiate statin therapy. 25.2 A 60-year-old woman with insulin-dependent diabetes has recently established care with a new physician. Her most recent LDL-C level was 110 mg/dL. What is the most appropriate management step? 25.3 A 50-year-old woman has an LDL-C of 165 mg/dL and one major risk factor for CHD (stage 1 hypertension). What is the initial management strategy for this individual? 25.4 A 72-year-old man is being seen by his primary care provider. The patient was recently found to have an abdominal aortic aneurysm. His LDL-C is 110 mg/dL on 10 mg simvastatin daily, which he is tolerating without any adverse effects. What is the next best step in management?

ANSWERS 25.1 C. The Reynolds risk score takes gender and hs-CRP level into account, and is a good option in this case for estimating this individual’s risk. 25.2 A. This patient has a CHD equivalent since she is a diabetic. She should be initiated on statin therapy with a goal LDL-C of 60 mmHg or a mean gradient > 40 mmHg. In symptomatic patients intervention is recommended for peak and mean gradients of 50 and 30 mmHg, respectively.

Congenital Shunts a p c are the c n m c mm n c n n c c c in adults after bicuspid aortic valve. They arise from abnormal formation of the interatrial septum or subsequent failure of normal septation in adulthood. The fetal atrial septum consists of two separate membranes with offset foramina that direct oxygenated placental blood from the inferior vena cava to the left atrium. These membranes fuse at birth, directing venous flow through the right heart. In some cases the septum is incompletely formed with residual “holes” that persist after birth. These are referred to as p c (ASDs). In other cases the normal membranes of the septum separate after birth, resulting in a patent foramen ovale (PFO). Defects of the atrial septum are often discovered incidentally when cardiac imaging is performed for other indication. Atrial septal defects occur in 1 of every 1,000 live births. Midseptum (ostium secundum) defects are the most common, but defects adjacent to the base of the septum (ostium primum ASD), vena cavae (sinus venosus ASD), and coronary sinus also occur. Ostium secundum defects are usually isolated abnormalities, while mp m m c c w n c c n c n n c mp c m 21. Sinus venous ASDs are associated with anomalous pulmonary venous return to the right atrium or vena cavae. The major problem with ASDs is excessive left-to-right shunting of blood across the septum. Shunting can result in right-sided pressure and volume overload, leading to pulmonary hypertension, right ventricular failure and Eisenmenger syndrome. Atrial septal defects can be diagnosed with transthoracic or transesophageal echocardiography or by cardiac MRI, which can also quantify shunt flow. ASD closure is considered appropriate when the ratio of pulmonary to systemic flow is >2:1 or when patients manifest evidence of right ventricular overload. Closure may be performed with open-heart surgery or catheter-based device closure; however, for technical reasons, device closure is limited to ostium secundum defects. P n m n v (PFO) is a far more common abnormality that can be seen on as many as 25% of all echocardiograms, often after provocation with maneuvers such as Valsalva. Shunting in PFOs is usually minor, and right-sided overload is rare. There is an association between PFO and stroke that is incompletely understood, but one theory is that small thrombi may form between the membranes of the septum and subsequently embolize. PFO is also associated with migraine headaches. At present the indications for surgical or device closure of PFOs are controversial, but it is generally considered appropriate in patients with PFO who have had recurrent despite appropriate medical therapy. V n c p c (VSDs) are the m c mm n c n n c n (after bicuspid aortic valves and ASD); they are much more common in children, where most close spontaneously. VSDs can involve any portion of

SECTIO N II: CLINICAL CASES

341

the interventricular septum, but defects in the membranous portion of the septum near the base of the heart are most common. Approximately 10% of VSDs occur in the muscular septum, and 5% are located near the left and right ventricular outflow tracts. Small (< 0.5-cm diameter) Vs d n p c c c n n m m v c w. Flow volume with small VSDs is minimal, and these are usually asymptomatic. Larger VSDs produce softer murmurs but produce higher flow. The principal concern with VSD is volume overload of the pulmonary circuit and the development of pulmonary hypertension. Symptoms include exertional dyspnea, exercise intolerance, and edema. Surgical or device closure is indicated when the shunt fraction is ≥2:1 and there is evidence of left ventricular volume overload or if the patient has a history of endocarditis. One can consider closure with a shunt fraction ≥1.5:1 if there is evidence of increased but reversible pulmonary vascular resistance. Repair is typically via openheart surgery; however, in select patients with membranous VSDs, catheter-based device closure may be an option. P n c (PDA) is failed closure of the ductus arteriosus, the vascular conduit that communicates between the fetal main pulmonary artery and the aortic arch (Figure 26-3). At birth, the ductus usually closes and becomes the ligamentum arteriosum. PDA is often an isolated defect in adults and tends to affect women more often than men. Symptoms are generally related to the degree of shunting and mimic those of ASD and VSD, namely, exertional dyspnea, orthopnea, and symptoms consistent with pulmonary hypertension. Percutaneous closure is the preferred treatment in adults with symptomatic PDA.

Pa te nt ductus a rte rios us

Ao RA

LA PA

LV RV

Figure 26-3. Patent ductus arteriosus. (Reprinted with permission, Cleveland Clinic Center for Medical Art & Photography © 2014. All Rights Reserved.)

342

CASE FILES: CARDIO LOGY

e

nm n n m , a very serious complication, is defined by the m c- -p m n c nn c n, p m n p n n, n c n . Eisenmenger syndrome is usually an end-stage complication in patients with congenital left-to-right shunts. Left-to-right shunts increase pulmonary blood flow, leading to cellular and structural changes in the pulmonary arteries that result in increased pulmonary vascular resistance and pulmonary hypertension. Untreated, the pulmonary pressures in patients with Eisenmenger syndrome eventually rival or exceed systemic arterial pressures, resulting in a reversal of shunt flow. r - w np n p x m n c n , n , n n nc n, n v n . Ventricular septal defects account for one-third of all cases of Eisenmenger syndrome, followed closely by atrial septal defects. The risk for Eisenmenger syndrome is proportional to shunt size and the volume of increased pulmonary flow. However, much of what we know about Eisenmenger syndrome comes from observation of patients who already have the diagnosis; the events involved in the progression to Eisenmenger syndrome are not well described. Patients with Eisenmenger syndrome experience a broad range of symptoms, including fatigue, syncope, and shortness of breath. Physical examination reveals central cyanosis, digital clubbing, a right ventricular heave, and a prominent or even palpable P2. Prominent jugular V waves and a systolic murmur at the left sternal border may be indicative of related tricuspid regurgitation. Advanced Eisenmenger syndrome can produce right ventricular failure, including peripheral edema, ascites, and hepatomegaly. Management of Eisenmenger syndrome is largely supportive. Pregnancy is considered absolutely contraindicated since mortality approaches 50-70%, and women with Eisenmenger syndrome should be counseled regarding contraception or surgical sterilization. Patients with Eisenmenger syndrome should avoid dehydration, weight-bearing exercise, peripheral vasodilator therapy, and anemia. Erythrocytosis is common in Eisenmenger syndrome, and patients with symptoms of hyperviscosity benefit from phlebotomy with saline replacement of blood volume to prevent dehydration. There are data suggesting that pulmonary vasodilator therapy may improve quality of life in patients with Eisenmenger syndrome. Combined heart and lung transplantation is an option for qualified patients. Life expectancy is reduced in Eisenmenger syndrome by about 20 years when compared to the general population.

Com plex Congenital Abnorm alities t np n (TGA) is characterized by discordance between the ventricles and the great arteries. In D-type transposition (D-TGA; dextrorotary TGA) the atria and ventricles are concordant, but the ventricles and great arteries are discordant (Figure 26-4). This results in separate pulmonary and systemic circulatory beds without communication. D-TGA affects 3–4 per 10,000 live births and accounts for 3% of all cases of congenital heart disease. It was a uniformly fatal disease until a few decades ago, when techniques were developed to allow for mixing of right- and left-sided circulation systems. a m n n p c c n e1 k p c p n, n b n p m c nb p m c n m- v n . Once stable, the infant can then undergo a more definitive surgical repair. In patients with no other major congenital defects, the treatment of choice is an arterial switch procedure (Figure 26-5). Prior to the development of the

SECTIO N II: CLINICAL CASES

343

LA Ao

PA

RA LV RV

Figure 26-4. D-type transposition of the great arteries (D-TGA). The right atrium (RA) and right ventricle are in correct position, but the aorta (AO) fills via the RV. The left atrium (LA) and left ventricle (LV) are correctly positioned, but the pulmonary artery (PA) is filled via the LV. (Reprinted with perm ission, Cleveland Clinic Center for Medical Art & Photography © 2014. All Rights Reserved.)

Ao PA

LA

RA LV RV

Figure 26-5. Arterial switch surgery for D-TGA. Right atrium (RA), right ventricle (RV), left atrium (LA), left ventricle (LV), aorta (AO), and pulmonary artery (PA) are shown. (Reprinted with perm ission, Cleveland Clinic Center for Medical Art & Photography © 2014. All Rights Reserved.)

344

CASE FILES: CARDIO LOGY

Ba ffle

RA Ao

PA

LA RV LV Ba ffle Figure 26-6. Atrial switch surgery (Mustard procedure) in D-TGA. The anatomic right ventricle (located on the right side) acts as the systemic pump. Vena cava flow is routed to the right atrium via a baffle (arrow). Right atrium (RA), right ventricle (RV), left atrium (LA), left ventricle (LV), aorta (AO), and pulmonary artery (PA) are shown. (Reprinted with permission, Cleveland Clinic Center for Medical Art & Photography © 2014. All Rights Reserved.)

arterial switch procedure, patients were treated with an atrial switch surgery (Mustard or Senning procedure). These surgeries involved creating an intra-atrial baffle to direct deoxygenated blood from the venous circulation into the left (venous) ventricle (Figure 26-6). The baffle also directs pulmonary venous blood to the right (systemic) ventricle. Atrial switch surgeries eliminate hypoxemia but do not correct ventriculoarterial discordance, a fact that often leads to problems later in life. For patients with D-TGA and large VSDs and pulmonic stenosis or atresia, a Rastelli procedure is usually performed. In this surgery a baffle is created to shunt oxygenated from the VSD into the aorta while a synthetic conduit is used to connect the right ventricle to the pulmonary artery. l- p np n (L-TGA; levorotary TGA) is also called c n n c c t g a . L-TGA affects fewer than 1 in 10,000 live births and accounts for less than 1% of all cases of congenital heart disease. In this disorder there is discordance between the atria and ventricles and between the ventricles and great arteries (Figure 26-7). Thus there is normal transit of venous blood through the pulmonary circuit and into the systemic circulation, but the left ventricle is the venous ventricle and the right ventricle is the systemic ventricle. Patients who have undergone a successful arterial switch procedure generally do well. Patients who had atrial switch procedures commonly develop failure of the systemic right ventricle, which is at a mechanical disadvantage when pushing against system vascular resistance. This is also a problem for patients with L-TGA. Atrial arrhythmias are also common complications of the atrial switch. Baffle stenosis can

SECTIO N II: CLINICAL CASES

Ao RA

345

LA PA

LV RV

Figure 26-7. L-type transposition of the great arteries (L-TGA). The atria are concordant with their respective great arteries, but there is ventriculoarterial discordance. Normal cardiopulmonary circulation is maintained but the anatomic right ventricle (located on the left side) acts as the systemic pump. Right atrium (RA), right ventricle (RV), left atrium (LA), left ventricle (LV), aorta (AO) and pulmonary artery (PA) are shown. (Reprinted with perm ission, Cleveland Clinic Center for Medical Art & Photography © 2014. All Rights Reserved.)

occur, leading to obstruction of the superior vena cava and/or pulmonary veins. Management of these complications is similar to the management of patients with heart failure and arrhythmias who do not have congenital heart disease. Baffle stenosis is sometimes amenable to balloon dilatation. Patients with medically refractory heart failure can be considered for advanced procedures such as ventricular assist devices and transplantation. t f (ToF) is defined by the simultaneous presence of p m n n ,v n c p c, w p c mn , n c nc n c v n c p p (Figure 26-8). ToF affects approximately 4 of every 10,000 live births and represents approximately 10% of all cases of congenital heart disease. Although ToF is defined by its four cardinal abnormalities, the disorder is frequently associated with other congenital defects such as anomalous origin of the coronary arteries, patent ductus arteriosus, and a right-sided aortic arch. The principal hemodynamic lesion in ToF is right ventricular outflow tract (RVOT) obstruction, the severity of which dictates whether the patient will become cyanotic. VSDs in ToF are usually large and unrestricted, so there is little difference between left and right ventricular pressure. Ventricular outflow will be directed through whichever conduit (aorta or pulmonary artery) has lower resistance. If the aortic vascular resistance is higher, the patient will have a left-to-right shunt and will not become cyanotic. If the pulmonary vascular resistance is higher, the patient

346

CASE FILES: CARDIO LOGY

RAo

PS VS D RVH

Figure 26-8. Tetralogy of Fallot. (Reprinted with perm ission, Cleveland Clinic Center for Medical Art & Photography © 2014. All Rights Reserved.)

will have a right-to-left shunt and cyanosis will occur. The degree of RVOT obstruction in ToF can vary according to factors such as vasodilatation and the degree of RVOT contractility, causing episodic cyanosis. Most patients with ToF undergo surgical repair within the first year of life. Definitive repair involves relieving RVOT obstruction (RVOT enlargement), separation of systemic and pulmonic circulation (VSD closure), and preservation of pulmonic valve function (Figure 26-9). In patients with small pulmonary annular sizes, it may not be possible to enlarge the RVOT without placing a large transannular patch, which can result in severe pulmonic valve regurgitation. Older adults with ToF may have received a palliative shunt rather than a definitive repair. The Blalock-Taussig shunt consists in creating a conduit between the innominate or subclavian artery and the pulmonary artery to maintain flow within the pulmonary circuit. Adults with a history of ToF may develop complications related to their procedures. Pulmonic regurgitation is a common problem and when severe, it can lead to right ventricular dysfunction. In this case surgical or percutaneous pulmonic valve replacement may be required. Tachyarrhythmias are another major concern, particularly ventricular tachycardia originating from the modified RVOT. Patients treated with palliative shunt may develop symptoms consistent with subclavian steal. e b n’ n m is a congenital cardiac condition that involves the tricuspid valve and right ventricle. Ebstein’s anomaly occurs in approximately 1 in 20,000 live births and is linked to m n xp m. In Ebstein’s anomaly the leaflets of the tricuspid valve are elongated and malformed, often with a “sail-like” anterior. The valve is apically displaced in the right ventricle and the proximal portion of the

SECTIO N II: CLINICAL CASES

347

TAP TAP VS D

VS D

B A Figure 26-9. Surgical repair of tetralogy of Fallot with internal (a) and external (b) views of the heart; the right ventricular outflow tract is enlarged using a transannular patch (TAP), and the ventricular septal defect is closed. (Reprinted with permission, Cleveland Clinic Center for Medical Art & Photography © 2014. All Rights Reserved.)

right ventricle is “atrialized,” limiting the functional right ventricle to the apex or RVOT alone. The tricuspid valve in Ebstein’s anomaly is severely regurgitant, and many cases are associated with an atrial septal defect. Other associated anomalies include VSD, PDA, coarctation of the aorta, and pulmonary outflow obstruction. Adults with Ebstein’s anomaly may present with palpitations due to tachyarrhythmias. Atrial tachyarrhythmias are common, but accessory pathways between the right atrium and ventricle are also common and patients may present with WolffParkinson-White syndrome. Severe tricuspid regurgitation with right ventricular dysfunction may also occur. Treatment of adults with Ebstein’s anomaly involves management of tachyarrhythmias and volume overload related to heart failure. In some cases tricuspid valve replacement and ASD repair is required.

CASE CORRELATION • See also Case 10 (valvular stenosis), Case 22 (syncope), and Case 23 (dyspnea).

348

CASE FILES: CARDIO LOGY

COMPREHENSION QUESTIONS 26.1 A 32-year-old woman with an ostium secundum ASD presents for follow-up after a 4-year absence from your clinic. You previously advised her to undergo catheter-based closure of her ASD, but she declined because she was feeling well. She has not been seen by a physician since her last office visit with you. Over the past year she has experienced a significant functional decline with progressive weakness, dizziness, and shortness of breath. She also reports headaches, blurry vision, and epistaxis. Her vital signs are as follows: blood pressure 90/68 mmHg, pulse 110 bpm, respirations 22 per minute, and room air oximetry 78%. She has perioral cyanosis. Examination reveals a loud and palpable second heart sound with III/VI holosystolic murmur at the left sternal border. Her lungs are clear, and her extremities are cool with marked digital clubbing. Laboratory data include a normal basic metabolic panel, hemoglobin 21 g/dL, and hematocrit 65%. Her echocardiogram reveals marked right-sided chamber enlargement with severe right ventricular systolic dysfunction with severe tricuspid regurgitation and an estimated right ventricular systolic pressure of 120 mmHg. There is a large secundum ASD with laminar flow from right to left. She agrees to hospitalization for additional diagnostic testing and management. Which of the following treatment choices is not appropriate for this patient? A. Right-heart catheterization with trial of pulmonary vasodilator therapy B. Percutaneous ASD closure C. Phlebotomy D. Advanced heart failure consultation to discuss eligibility for heart and lung transplantation E. Oral contraceptive therapy 26.2 All of the following untreated congenital heart defects are associated with a risk for Eisenmenger syndrome except… A. ASD B. VSD C. PDA D. Bicuspid aortic valve E. ToF 26.3 Which of the following is not a feature of tetralogy of Fallot (ToF)? A. Aortic dilatation B. Pulmonic stenosis C. Rightward displacement of the aorta D. Right ventricular hypertrophy E. Ventricular septal defect

SECTIO N II: CLINICAL CASES

349

26.4 Which of the following statements regarding adult congenital heart disease is correct? A. D-TGA is most effectively treated with an arterial switch procedure B. L-TGA is most effectively treated with a Rastelli procedure C. ASD is the most common congenital heart disease encountered in adults D. Adult patients with Ebstein’s anomaly seldom present with tachyarrhythmias E. ToF is most effectively treated with a Blalock-Taussig shunt

ANSWERS 26.1 B. ASD closure is not appropriate in Eisenmenger’s syndrome because the patient has already developed irreversible pulmonary arterial hypertension. This patient has cyanotic heart disease and hyperviscosity syndrome. Appropriate therapies include phlebotomy with saline replacement and iron therapy if iron deficiency is present. Pregnancy is absolutely contraindicated in her case, and contraception would be appropriate. Pulmonary vasodilator therapy has symptomatic benefit in some patients with Eisenmenger syndrome and would be reasonable to try. This patient has a poor prognosis, and a discussion of advanced treatment options such as transplantation would be reasonable. 26.2 d . Bicuspid aortic valve is an isolated valvular abnormality that is not associated with left-to-right shunting. All of the remaining diagnoses have shunting as a key hemodynamic feature and may result in Eisenmenger syndrome. 26.3 a . Aortic dilatation is not a feature of tetraolgy of Fallot. 26.4 a . The aortic switch is the best therapy for D-TGA. L-TGA is congenitally corrected. Atrial switch surgeries (Mustard or Senning) were common therapies for D-TGA prior to the development of the arterial switch. Bicuspid aortic valve is the most common congenital heart problem in adults. Almost half of adult patients with Ebstein’s anomaly present with tachyarrhythmias. Definitive ToF surgery includes RVOT enlargement with a transannular patch and VSD closure. The Blalock-Taussig shunt is a palliative surgery designed to support pulmonary flow.

350

CASE FILES: CARDIO LOGY

CLINICAL PEARLS C

The numbers of adults with congenital heart diseases are increasing.

C

Cardiac MRI is the preferred imaging modality for patients with complex congenital heart disease.

C

Adults with surgically managed complex congenital disease in childhood commonly present with symptoms related to their surgeries.

C

Closure of congenital shunt lesions is dictated by patient symptoms, ventricular function, and the size of the shunt fraction.

C

Arrhythmias are the most common complication of Ebstein’s anomaly in adults.

REFERENCES Diller GP, Dimopoulos K, Broberg CS, et al. Presentation, survival prospects, and predictors of death in Eisenmenger syndrome: a combined retrospective and case-control study. Eur Heart J. 2006;27(14):1737–1742. Galiè N, Beghetti M, Gatzoulis MA, et al. Bosentan therapy in patients with Eisenmenger syndrome: a multicenter, double-blind, randomized, placebo-controlled study. Circulation. 2006;114(1):48–54. van der Velde ET, Vriend JW, Mannens MM, et al. CONCOR, an initiative towards a national registry and DNA-bank of patients with congenital heart disease in the Netherlands: rationale, design and first results. Eur J Epidemiol. 2005;20(6): 549–557. Warnes CA, Williams RG, Bashore TM, et al. ACC/AHA2008 Guidelines for the Management of Adults with Congenital Heart Disease: Executive Summary: a report of the American College of Cardiology/American Heart Association Task Force on Practice Guidelines (writing committee to develop guidelines for the management of adults with congenital heart disease). Circulation. 2008;118(23):2395–2451. Windram JD, Siu SC, Wald RM, Silversides CK. New directives in cardiac imaging: imaging the adult with congenital heart disease. Can J Cardiol. 2013;29(7):830–840.

CASE 27 A 28-year-old man with no medical history presents with acute chest pain. The pain started suddenly approximately 2 hours ago, and he describes it as a severe (rated at 10/ 10) tearing sensation radiating to the back between the scapulae. He never experienced this pain before. His family history is unremarkable. He doesn’t smoke, use alcohol, or take illicit drugs. On presentation, vitals were as follows: heart rate 120 bpm, blood pressure in left arm 100/ 40 mmHg and in right arm 105/ 40 mmHg, respiratory rate 25 breaths/ min, temperature 37.2°C (98.96°F), and his oximetry (O 2 saturation) was 96% on 2 L/ min of oxygen. His height is 2 m (6.56 ft) and weight 100 kg (220 lb). Physical examination revealed a tall, young gentleman in moderate distress. Cardiac examination revealed a regular rhythm with tachycardia, jugular venous pressure at 9 cm H 2O, III/ IV diastolic murmur with highest intensity in the left lower sternal border, and no rubs. Chest exam revealed pectus carinatum and mild bibasilar crackles. His neurological exam is intact. Other features included mild scoliosis and arachnodactyly in his fingers. His presenting ECG and chest x-ray are shown in Figures 27-1 and 27-2. Laboratory data were normal except for an elevated CK, CK-MB, and troponin T. C C C

What is the most likely diagnosis? What is the best next diagnostic step? What is the best next step in therapy?

3 5 2 C A S

a VF

V5

V3

V6

F

E

III

V2

I

a VL

V4

L

II

V1

E

a VR

Y

G

O

L

O

I

D

R

A

C

:

S

I

V1

II

V5 Fi ure 27-1. ECG for main subject of this case. Note the ST segment elevation in the inferior leads.

SECTIO N II: CLINICAL CASES

Fi ure 27-2. PA Chest x-ray for main subject of this case.

353

354

CASE FILES: CARDIO LOGY

ANSWER TO CASE 27: Acute Type A Aortic Dissection • M k an : Acute type A aortic dissection complicated by severe aortic regurgitation and inferior myocardial infarction. • N x

an

• N x

p n

p: CT angiogram of the aorta. ap : Emergent surgery.

ANALYSIS Objectives 1. Know the different types of acute aortic dissection, the diagnostic approach, and how to manage them. 2. Understand the etiology, pathophysiology, and risk factors of different types of aortic aneurysms. 3. Be familiar with the different strategies of surveillance and management of aortic aneurysms according to their type and size.

Considerations The patient is young and presents with sudden-onset severe and tearing chest pain radiating to his back. This presentation is classic for an acute aortic dissection. He also has skeletal features suggestive of Marfan’s syndrome. Patients with Marfan’s syndrome are at higher risk for developing thoracic aortic aneurysms and dissection. Marfan’s syndrome is an autosomal dominant disease, but he has no family history of Marfan’s syndrome. Remember that up to one-third of patients with Marfan’s syndrome develop the mutation in the Fibrillin-1 gene spontaneously so absence of family history does not preclude the diagnosis. The patient’s physical exam shows signs of type A aortic dissection with complications. There is no differential blood pressure between both arms because the dissection starts proximally and involves the right brachiocephalic and left subclavian arteries. The patient has a wide pulse pressure and a diastolic murmur heard at the left lower sternal border suggestive of aortic insufficiency, which occurs with proximal type A dissection involving the aortic root. The chest x-ray shows a widened mediastinum. In addition, he has an inferior ST elevation myocardial infarction suggestive of dissection into the ostium of the right coronary artery. He is tachycardic with borderline low blood pressure, jugular venous distention, and bibasilar crackles, which are all signs of acute heart failure due to the severe aortic insufficiency and inferior myocardial infarction. The clinician should be vigilant about recognizing these complications because they determine the next steps in management. Although this patient has abnormal vitals, he is relatively stable and the next step would be to get a cardiac gated CT aortogram to diagnose the aortic dissection, determine its type, and identify the origin of the dissection flap as well as the degree of involvement of other major vessels. Cardiothoracic surgery should be

SECTIO N II: CLINICAL CASES

355

consulted emergently in this case. T he mortality rate for aortic dissection involving the ascending aorta is around 20% and increases by 1% per hour during the first 48 hours until the patient undergoes surgery. While waiting to go to the OR, initial medical management should focus on stabilizing the patient by controlling heart rate and blood pressure using intravenous beta-blockers and intravenous arterial vasodilators if needed. The beta-blockers should be given before the administration of direct-acting vasodilator therapy to reduce aortic wall strain, and the dose should be titrated to a target a heart rate between 55 and 65 bpm. However, beta-blocker therapy in this patient is relatively contraindicated because of the presence of severe aortic regurgitation and physical exam findings consistent with acute heart failure.

APPROACH TO: Aortic Diseases DEFINITIONS As c e N d i N g t h o r Ac i c Ao r t A: The ascending segment of the aorta extending from the aortic root to the aortic arch. Ao r t i c Ar c h : The aortic segment that gives rise to the major vessels of the head and arms (right brachiocephalic, left common carotid, and left subclavian); it represents the transition from the ascending aorta to the descending aorta. d e s c e N d i N g t h o r Ac i c Ao r t A: Aortic segment extending from just distal to the left subclavian artery until the diaphragmatic crura. Ab d o Mi N Al Ao r t A: The part of the aorta distal to the diaphragmatic crura Ao r t i c AN e u r y s M: Pathologic weakening in the aortic wall leading to an increase in the diameter of the aorta by >50% of that expected for age- and sexmatched individuals. Ao r t i c d i s s e c t i o N : A tear in the aortic wall between the tunica intima and media creating a false lumen compromising the integrity of the aorta and its branches.

CLINICAL APPROACH Pathophysiology A dissection usually results from a tear in the vascular intima, causing it to separate from the media with creation of a false lumen. Less commonly, it can result from rupture of the vasa vasorum (arterioles and capillary network supplying the aortic wall) with subsequent hemorrhage into the aortic wall. The high pressure in the aorta causes the false lumen to expand and propagate in an antegrade fashion, thus compromising the true lumen and branching vessels and causing end-organ damage. The tear can result from either pathologic weakness of the aortic wall or extremely high blood pressure.

356

CASE FILES: CARDIO LOGY

Epidem iology Aortic dissection involving the a n n a a is the most common aortic emergency with an estimated prevalence of 5–30 cases per million. It usually affects older adults between 50 and 70 years of age. Presentation in younger individuals should raise suspicion for genetic or congenital disorders. Prevalence is 2–3 times more common in men than women. It is associated with very high mortality. estimated at 20% and then 1% per hour in the first 24–48 hours if surgery is not performed.

Risk Factors h p n n is present in 75% of individuals with dissection and represents the most prevalent risk factor for aortic dissection. Older age, atherosclerosis, and tobacco use are also common risk factors for dissection. Genetic diseases also contribute to the risk of aortic dissection and dissections in younger patients should prompt some investigation for an underlying predisposing disorder. Ma fan’ n m an a ma m nan f n Fibrillin-1 n that regulates extracellular matrix and TGF-B. It is associated with ocular and musculoskeletal manifestations in addition to aortic pathology. Loeys-Dietz syndrome is also an autosomal disorder with a defect in the T GFBR1 or T GFBR2 gene that encodes TGF-B receptors. It is associated with hypertelorism and bifid uvula and tends to involve the aortic root. Ehlers-Danlos type IV is an autosomal dominant defect in the COL3A 1 gene, which encodes procollagen 3. Other clinical manifestations include easy bruising and uterine rupture. Certain congenital disorders are also associated with an increased risk for aortic dissection such as bicuspid aortic valve (BAV), coarctation of the aorta, Turner’s syndrome, and tetralogy of Fallot. Other causes of aortic dissection include cocaine abuse and chest trauma, including recent surgical or procedural instrumentation. Inflammation of aortic tissue can also cause aortic dissection including infections such as tertiary syphilis and inflammatory vasculitides such as Takayasu’s arteritis, giant cell arteritis, and Behçet disease.

Classification Aortic dissections are classified in several ways based on their anatomical features (see Figure 27-3). An acute dissections are by definition is 6 cm have >20% per year risk of rupture. Smaller (5–6-cm) aneurysms have 6% yearly risk of rupture. Rupture most commonly occurs in the left retroperitoneal space and is contained. ep m : AAA is more common than TAA. Of all aneurysm cases reported, 75% involve the abdominal aorta. Incidence of all aortic aneurysms in men is approximately 5% and women 1–2% in those aged more than 65 years. Also, 95% f AAA a nf a na . Rupture of AAA is associated with very high mortality. The “rule of quarters” applies to the outcome of aortic rupture—onequarter die before reaching the hospital, a second quarter die before surgery, and a third quarter die perioperatively while undergoing surgery, making overall survival rate after rupture about 25%. e an k fa : Older age, current or past history of smoking, male gender, hypertension, hyperlipidemia, and atherosclerosis in other vascular beds are all associated with an increased risk for aortic aneurysms. Approximately one-quarter of AAA patients have a first-degree relative with an AAA. Firstdegree male relatives of patients with AAA have a 12 times greater risk of having an AAA. Infections such as salmonella and Staphylococcus aureus can cause mycotic aneurysms. Large-vessel vasculitides such as Takayasu’s arteritis, giant cell arteritis, or Behçet’s syndrome can also lead to aneurysm formation. As with dissections, blunt external or internal iatrogenic trauma can predispose to the development of aneurysms. c n a p n a n: The majority of AAAs are discovered incidentally during routine care or imaging for other reasons and are asymptomatic. Symptoms are usually associated with large aneurysms or those with rapid growth. Patients can present with sudden, constant, sharp, or dull back pain, which may radiate to the legs, flanks, or buttocks. Shock, pallor, abdominal distention, and signs of

358

CASE FILES: CARDIO LOGY

Table 27-1 • CLINICAL PRESENTATION, PHYSICAL ExAm, DIAg NOSTIC mODALITIES, AND mANAg EmENT OF DIFFERENT TYPES OF AORTIC DISSECTION Stanford Classification

Type A or Pro i al

Type B or Distal

DeBakey classification

Types 1 and 2

Type 3

Clinical presentation

Sudden and sharp chest pain radiating to the interscapular region in the back (47%) or abdomen (21%)

Sudden and sharp back pain (64%), chest pain (63%), or abdominal pain (43%)

Complications

Hypotension, shock, acute heart failure, aortic insufficiency, pericardial effusion, cardiac tamponade, ostial coronary dissection and ST elevation myocardial infarction, carotid dissection and stroke, limb ischemia, distal endorgan damage; rupture of the contained dissection can occur

Damage to organs supplied by vessels distal to the left subclavian artery; this can cause gut and limb ischemia, paraplegia due to involvement of the anterior spinal artery, hepatic ischemia, and acute kidney injury; it can also rupture

Physical exam

Detailed physical examination of the cardiovascular system including pulse checks in all extremities, blood pressure measurement in both arms compared to legs, auscultation for aortic insufficiency murmur, and careful examination for signs of cardiac tamponade or acute heart failure; a detailed neurologic exam is also necessary to establish carotid involvement; examine for signs of end-organ ischemic damage; ECG

Auscultation can reveal an abdominal bruit due to turbulent flow caused by the dissection; detailed examination of distal pulses and a detailed abdominal exam are also necessary to detect signs of end-organ decreased perfusion

Diagnosis

Contrast enhanced, cardiac ated CT an io ra (CTA)–it has the best sensitivity and specificity approaching 100% ma netic resonance an io raphy (mRA): Excellent specificity and sensitivity and better at identifying aortic wall pathology such as penetrating atherosclerotic ulcer or intramural hematoma. Better for chronic follow up of dissections and for assessment of ejection fraction, aortic root and valve function Transesopha eal echocardio ra (TEE) has high sensitivity and specificity and is usually reserved for unstable patients Conventional an io raphy has the lowest sensitivity

CTA is the test of choice mRA also has high sensitivity and specificity TEE can miss distal descending thoracic dissections and is operator-dependent Conventional an io raphy has the lowest sensitivity and doesn’t allow for examination of extravascular complications

(continued)

SECTIO N II: CLINICAL CASES

359

Table 27-1 • CLINICAL PRESENTATION, PHYSICAL ExAm, DIAg NOSTIC mODALITIES, AND mANAg EmENT OF DIFFERENT TYPES OF AORTIC DISSECTION (CONTINUED) Stanford Classification

Type A or Pro i al

Type B or Distal

DeBakey classification

Types 1 and 2

Type 3

Management

It is a surgical emergency; immediate surgical repair is indicated, especially with complications such as aortic insufficiency, cardiac tamponade, and end- organ dysfunction such as stroke or myocardial infarction Initial medical management to reduce heart rate and blood pressure should be initiated until the patient goes to surgery; IV β-blockers are the drug of choice with further addition of nitroprusside or other arterial vasodilators if needed; goal is a heart rate < 60 bpm and mean arterial pressure < 70

Medical management with β-blockers and other arterial vasodilators; heart rate and blood pressure must be tightly controlled quickly with IV medications and later transitioned to orals in order to decrease the rate of expansion of the dissection Surgical therapy is indicated only if: • Signs of end-organ perfusion • Retrograde extension into the ascending aorta • Rupture/ impending rupture • Associated with genetic or congential diseases with a high risk of rupture

I

II

III

IV

Fi ure 27-4. Crawford classification system for aortic aneurysms. (Reprinted, with perm ission, from Edm unds LH Jr, ed. Cardiac Surgery in the Adult. New York: McGraw-Hill; 1997).

360

CASE FILES: CARDIO LOGY

end-organ damage could all be signs of uncontained ruptured aneurysms. Complications of AAA include formation of arteriovenous fistula between AAA and the inferior vena cava as well as aortoenteric fistula between AAA and the intestines, causing gastrointestinal bleeding, and these are all indications for surgical intervention. P a xam f n n : A palpable, pulsatile abdominal mass can be felt in many patients with large AAAs. Deep palpation should be avoided in patients with a known large aneurysm to prevent aneurysmal rupture. Auscultation could reveal an abdominal bruit. Bruits in other vessels are also common because these patients usually have atherosclerosis in multiple major arteries. Absent pulses or cold extremities could be signs of distal embolization from a mural thrombus in the wall of the aneurysm. d an n : Abdominal ultrasound is the most commonly used screening tool for AAA, and it is widely available with no radiation exposure or need for IV contrast. It is cost-effective and accurate for aneurysms up to 4.5 cm in diameter. It can be used for surveillance as well. However, it is not a good modality for evaluating the branches of the aorta, and thus further imaging is needed for better delineation of detailed aortic anatomy prior to surgical intervention if indicated. CT aortography provides accurate measurement of aneurysm size, shape, and involvement of branching vessels. It is better than ultrasound for aneurysms >4.5 cm, and it allows for assessment of extravasated blood and the extent of rupture. Magnetic resonance angiography (MRA) is also accurate for assessing aneurysm size and pathology and is well suited for the assessment of suprarenal and ileofemoral involvement. Conventional aortography is invasive and can cause underestimation of aneurysm size if there is mural thrombus. s n n : The US Preventative Services Task Force recommends one-time ultrasound screening for AAA for men 65–75 years of age with a history of smoking. The Society for Vascular Surgery recommends AAA screening also in men and women with a family history of AAAs. Mana m n : Management is dictated by whether the patient is symptomatic, the size of the aneurysm, and associated complications or end-organ involvement. (See Tables 27-2, 27-3, and 27-4 for more information regarding management.) Thoracic Aortic Aneurys s (TAAs) Pa p : TAA is most commonly caused by cystic medial degeneration characterized by loss of the elastic lamina and smooth muscles in the tunica media and replacement with cysts with basophilic ground substance, leading to a cystic appearance. It can also be caused by atherosclerosis and inflammation in the aortic wall as in AAA. Genetic diseases can cause TAA by various mechanisms such as defects in collagen, procollagen, and TGF-B production pathways. e an k fa : Older age, current or past history of smoking, male gender, hypertension, hyperlipidemia, and atherosclerosis in other vascular beds are all risk factors for TAA. Genetic diseases such as Marfan’s, Loeys-Dietz, and Ehlers Danlos type IV syndromes are also risk factors. Infections such as syphilis affecting the aortic root or mycotic bacterial aneurysms can also result in TAA

SECTIO N II: CLINICAL CASES

361

Table 27-2 • mANAg EmENT OF ABDOmINAL AORTIC ANEURYSm BY SIz E AND SYmPTOmS Aneurys Dia eter (c ) Asymptomatic

Symptomatic

Surveillance and mana e ent

3–4

Aggressive risk factor modification, medical therapy, and 1 year surveillance

4–5

Aggressive risk factor modification, medical therapy, and 6 months surveillance; some vascular societies recommend elective surgical repair for women with aneurysms 4.5–5 cm because women have higher risk of rupture at smaller diameters

5–5.5

Aggressive risk factor modification, medical therapy, and 3–6 months surveillance; controversy exists in whether these patients should undergo elective surgical repair; surgery is considered in women and in younger patients with known genetic predisposition or congenital diseases because they have a higher risk of rupture at smaller diameters; surgery is recommended if the rate of aneurysmal expansion is >1 c / year

> 5.5

Refer for endovascular aortic repair or open surgical repair Refer for endovascular aortic repair or open surgical repair

formation. Large-vessel vasculitides, trauma, and remodeling of chronic aortic dissections are also risk factors for TAA. c n a p n a n: Most TAAs are discovered incidentally during routine care or imaging for other reasons and are asymptomatic. Symptoms usually develop when the TAA is large or ruptured, causing sudden-onset, severe, sharp chest or back pain. They rupture into the left pleural space more commonly but can also rupture into the pericardium, causing cardiac tamponade, or into the esophagus, causing hematemesis. Large aneurysms can also compress other structures in the thorax. This, in turn, can cause superior vena cava compression syndrome, dysphagia due to esophageal compression, hoarseness due to laryngeal nerve compression, or dyspnea and cough due to trachea or main bronchi compression. Large aortic root aneurysms can cause severe aortic insufficiency with heart failure. They can also compress the coronary vessels, causing myocardial ischemia; they can also cause aortic dissection. P a xam f n n : The physical exam is not always helpful in patients with TAA. Rarely, a suprasternal pulsatile mass can be felt. Detailed cardiovascular examination is necessary to evaluate for complications of TAA such as aortic regurgitation, cardiac tamponade, or thromboembolic phenomena. The physician must complete a full body exam to detect signs of end-organ compromise due to the TAA. d a n n : Chest x-ray can show a widened mediastinum, trachea or bronchial displacement, or unusual aortic borders. Computed tomography angiography (CTA) and MRA are very sensitive and specific for the assessment

362

CASE FILES: CARDIO LOGY

Table 27-3 • COmPARISON OF DIFFERENT mEDICAL AND SURg ICAL STRATEg IES FOR ABDOmINAL AORTIC ANEURYSm REPAIR medical mana e ent β-blockers decrease the rate of enlargement and risk of rupture in small studies; other antihypertensive agents such as angiotensin receptor blockers might also be beneficial. Aggressive risk factor modification by controlling hypertension, hyperlipidemia, and smoking cessation.

Endovascular Aortic Repair (EVAR) Percutaneous placement of an aortic stent graft across the AAA to seal it off and stop aneurysmal expansion and/ or rupture. FDA-approved for descending thoracic aneurysms or infrarenal AAA. Suitable anatomy such as normal aortic diameter at the proximal and distal ends of the stent and minimal aortic angulation is necessary; 30–60% of patients will have suitable anatomy for EVAR. Three randomized trials have compared EVAR to open surgical repair; DREAM trial showed no survival benefit to EVAR over open repair; EVAR-1 showed lower short-term operative mortality to EVAR but no long-term differences in total or aneurysm-related mortality; OVER trial has shown early operative advantage of EVAR; EVAR-2 trial compared EVAR to medical therapy in patients deemed too high risk for surgery, but the findings showed no benefit for EVAR over medical therapy. Endoleak is a well-recognized complication of EVAR that can cause aneurysmal reexpansion and rupture; EVAR should be followed by routine surveillance to ensure that the stent is intact.

Open Sur ical Repair It is a high-risk surgery, and as most patients with large AAA are elderly, with multiple risk factors as well as established atherosclerosis, it is recommended that they undergo preoperative cardiac risk assessment. Perioperative mortality is usually 4–6% overall and 0.5 cm/ year for TAA Patients with genetic or congenital diseases should undergo elective operation at diameters 4–5 cm (depending on condition) to prevent rupture or dissection; among patients with bicuspid aortic valve or Marfan’s, a small study has shown that those in which the ratio of maximal aortic root cross-sectional area in centimeters squared to the patient’s height (in meters) of >10 should undergo surgical intervention as they are at higher risk of dissection EVAR can be performed in patients with very high operative risk but has not been proven to be more beneficial than medical management in these cases

Table 27-5 • COmPARISON OF DIFFERENT mEDICAL AND SURg ICAL STRATEg IES FOR ABDOmINAL AORTIC ANEURYSm REPAIR medical mana e ent Aggressive risk factor modification by controlling hypertension, hyperlipidemia, and smoking cessation. β-blockers have been shown in some small studies to reduce the rate of aneurysm expansion and risk of rupture. β-blockers and the angiotensin receptor blocker losartan have proved beneficial in patients with Marfan’s syndrome, particularly in reducing the rate of aneurysmal expansion, especially when started at a young age

Endovascular Aortic Repair (EVAR) Use of EVAR has been reported in the aortic arch and descending thoracic aorta. The Talent VALOR and Zenith TX2 trials have suggested that EVAR is a safe and effective alternative to open repair; however, it exposed the limitations of EVAR; EVAR requires careful patient selection with appropriate anatomy, easy vascular access and no need for other concomitant vascular interventions such as subclavian or carotid bypass grafting EVAR is currently reserved for patients with TAA who are at high risk for open repair and have ideal anatomy Endoleak is a well-recognized complication of EVAR that can cause aneurysmal reexpansion and rupture; EVAR should be followed by routine surveillance to ensure that the stent is intact

Open Sur ical Repair It is a high-risk surgery; preoperative cardiac risk assessment is indicated especially in patients with multiple risk factors for coronary artery disease. Perioperative mortality is usually 5–10%; perioperative morbidity includes MI (7.2%), stroke (4.8%), acute renal failure (2.4%), hemorrhage (7.2%), and paralegia (6%), due to compromise of anterior spinal artery Surgery involves resection of the aneurysm with placement of a Dacron graft and connecting the major arteries to the graft

364

CASE FILES: CARDIO LOGY

CASE CORRELATION • See also Case 1 (acute coronary syndrome/STEMI), Case 2 (acute coronary syndrome/NSTEMI), Case 3 (cardiogenic shock), Case 6 (acute valvular regurgitation), Case 21 (chest pain), and Case 22 (syncope).

COmPREHENSION QUESTIONS 27.1 Which of the following is an indication to undergo emergent aortic surgery? A. Acute type B aortic dissection B. Abdominal aortic aneurysm 4.5 cm in diameter C. Chronic type A aortic dissection D. Ruptured aortic aneurysm 5 cm in diameter 27.2 A 65-year-old gentleman with history of uncontrolled hypertension and smoking had a screening abdominal ultrasound last year showing an infrarenal abdominal aortic aneurysm 4 cm in diameter. He says he has been taking his blood pressure medications, but his blood pressure in clinic today is 170/90 mmHg. He continues to smoke. He is here today for a repeat abdominal ultrasound that shows the aneurysm to be 5.2 cm in diameter. In addition to counseling him to quit smoking, what is the best next step in this patient’s management? A. Optimize blood pressure control only B. Optimize blood pressure control and refer to vascular surgery for surgical repair C. Repeat an ultrasound in 3 months D. Repeat an ultrasound in 1 year E. Make no changes or referrals at this time 27.3 A 55-year-old man presents with worsening shortness of breath and exercise tolerance for the past 6 months. He has also been noticing worsening lower extremity edema, orthopnea, and paroxysmal nocturnal dyspnea. His medical history is significant only for being told that his thoracic aorta might be larger than average 5 years ago when he had a chest x-ray. One of his cousins had problems with one of his heart valves when he was young. No other significant family history. Physical examination reveals a blood pressure of 120/40 mmHg and a heart rate of 120 bpm. He has a II/IV diastolic murmur heard over the left lower sternal border and Corrigan’s pulse. He has no abnormal skeletal, occular, or morphologic features. What is the most likely diagnosis? A. Bicuspid aortic valve and aortic root dilation causing aortic insufficiency B. Aortic insufficiency and aortic root dilation due to Marfan’s syndrome C. Patent foramen ovale with right-to-left shunt D. Infective endocarditis causing tricuspid valve stenosis

SECTIO N II: CLINICAL CASES

365

ANSWERS 27.1 d . Ruptured aortic aneurysm is a life-threatening emergency, especially when the rupture is not contained, and is an indication to undergo emergent surgery. It has very high mortality rate. 27.2 b . Optimize blood pressure control and refer to vascular surgery for surgical repair because his abdominal aneurysm has expanded at a rate of >1 cm per year. This patient also needs to stop smoking and needs better blood pressure control. 27.3 A. The patient clearly has aortic insufficiency, evident by symptoms of progressive heart failure, diastolic murmur at the left lower sternal border, wide pulse pressure, and Corrigan’s pulse. He has no morphologic features to suggest Marfan’s. His age of presentation is typical of bicuspid aortic valve disease. Bicuspid aortic valve can itself cause aortic insufficiency, but it also predisposes to ascending aortic aneurysms, causing dilation of the root and aortic insufficiency.

CLINCIAL PEARLS C

Acute aortic dissection presents with sharp chest pain radiating to the back.

C

Type A acute aortic dissection is a surgical emergency. Type B aortic dissections should be managed medically unless surgery is indicated for rupture or vascular compromise of end organs.

C

Aortic aneurysms are generally asymptomatic and discovered incidentally. Asymptomatic aneurysms should be monitored periodically until they reach a size of ≥5.5 cm, and then surgery is indicated. Surgery is indicated also for symptomatic patients.

C

Endovascular aortic repair (EVAR) performs as well as open-heart surgery in randomized controlled trials, where it has lower perioperative mortality, but the long-term mortality is similar. However, EVAR requires certain criteria that are available in only 30–60% of patients.

C

Screening with an abdominal ultrasound is recommended for male patients 65–75 years old with a history of smoking.

REFERENCES Baverman AC, Thompson RW, Sanchez LA. Diseases of the aorta. In: Bonow O, Braunwald E, eds. Braunwald’s Heart Disease: A Textbook of Cardiovascular Medicine. 9th ed. Philadelphia, PA: Elsevier, Saunders; 2012:1309–1337. Blankensteijn J, de Jong SECA, Prinssen M, et al. Two-year outcomes after conventional or endovascular repair of the abdominal aortic aneurysms. N Engl J Med. 2005;352:2398–2405.

366

CASE FILES: CARDIO LOGY

Bunte MC, Thamilarasan M. Aortic aneurysm and aortic dissection. In: Griffin BP, ed. Textbook of Cardiovascular Medicine. 4th ed. Philadelphia, PA: Lippincott Williams & Wilkins; 2013:459–480. EVAR Trial Participants. Comparison of endovascular aneurysm repaire with open repair in patients with abdominal aortic aneurysm (EVAR 1 trial), 30-day operative mortality results: randomised controlled trial. Lancet. 2004;364:843–848. EVAR Trial Participants. Endovascular aneurysm repair and outcome in patients unfit for open repair of abdominal aortic aneurysm (EVAR trial 2): randomised controlled trial. Lancet. 2005;365:2187–2192. Fleming C, Whitlock EP, Beil TL, et al. Screening for abdominal aortic aneurysm: a best-evidence systematic review for the U.S. Preventative Services Task Force. A nn Intern Med. 2005;142:203–211. Greenhalgh RM, Brown LC, Powell JT, Thompson SG, Epstein D, Sculpher MJ; United Kingdom EVAR Trial Investigators. Endovascular versus open repair of abdominal aortic aneurysm. N Engl J Med. 2010;362:1863–1871. Rooke TW, Hirsch AT, Misra S, et al. 2011 ACCF/AHA focused update of the guideline for the management of patients with peripheral arterial disease (updating the 2005 guideline). J A m Coll Cardiol. 2011;58(19):2020–2045. Svensson LG, Khitin L. Aortic cross-sectional area/height ratio timing of aortic surgery in asymptomatic patients with Marfan syndrome. J T horac Cardiovasc Surg. 2002;123:360–361. Svensson LG, Kim KH, Lytle BW, et al. Relationship of aortic cross-sectional area to height ratio and the risk of aortic dissection in patients with bicuspid aortic valves. J T horac Cardiovasc Surg. 2003;126:892–893.

CASE 28 A 27-year-old intravenous drug user presents to the emergency department with 2 weeks of fever, productive cough, and pleuritic chest pain. He has had intermittent chills and drenching night sweats. He reports a decreased appetite and a 5-pound weight loss. He has had no sick contacts. The patient has also noted occasional blood in his urine. He denies any medical history and takes no prescription medications. He injects heroin daily, smokes tobacco and marijuana, but does not drink alcohol. He is unemployed and has had no recent travel. On examination, the patient is febrile to 101°F, is tachycardic to 115 bpm, and has a blood pressure of 92/ 65 mmHg. He is thin and ill-appearing. His mucous membranes are dry, and he has poor dentition. His sclera are anicteric. Neck exam reveals 6 cm of jugular venous distention above the sternal angle. The pulmonary exam reveals bronchial breath sounds over the right upper and left lower lobes. The cardiac exam reveals tachycardic, regular rate, a normal S1 and S2, and a III/ VI blowing holosytolic murmur best heard at the left lower sternal border. His abdomen is soft and nontender with normal bowel sounds. Skin exam reveals scattered erythematous, nontender macules on his right palm and needle marks on both upper extremities. A complete blood count reveals an elevated white blood cell count to 19,000 cells/ mm 3 with 78% neutrophils and 11% bands. Chest x-ray reveals scattered multifocal wedge-shaped opacities in the right upper and left lower lobes. Electrocardiogram shows sinus tachycardia with a normal PR interval. C C C

What is the most likely diagnosis? What is the best next diagnostic step? What is the best next step in therapy?

368

CASE FILES: CARDIO LOGY

ANSWER TO CASE 28: Inf cti endocarditis Summary: A 27-year-old IV drug user is presenting with fevers, chills, night sweats, cough, and chest pain for 2 weeks. He has a murmur on exam and evidence of septic pulmonary emboli on chest x-ray. He is ill-appearing, febrile, and tachycardic. His murmur is consistent with tricuspid regurgitation. He has abnormal breath sounds over multiple lung fields. His skin exam reveals Janeway lesions on his right palm. His white blood cell count is 19,000 cells/mm 3 with 78% neutrophils and 11% bands. His ECG shows sinus tachycardia with no evidence of atrioventricular conduction delay. • M

ik

• N x

i n

• N x

i n i : Infective endocarditis. in

i

: Blood cultures. : Intravenous fluid resuscitation.

ANALYSIS Objectives 1. Identify risk factors for developing infective endocarditis. 2. Recognize the common clinical manifestations of infective endocarditis, including Roth spots, Janeway lesions, Osler nodes, and splinter hemorrhages. 3. Become familiar with the workup for infective endocarditis, which includes blood cultures and echocardiography. 4. Know the indications for surgical management of infective endocarditis.

Considerations A 27-year-old IV drug user is presenting with fevers, cough, and chest pain with a new murmur and septic emboli on chest x-ray suggestive of infective endocarditis. His initial management should include intravenous fluid resuscitation and broadspectrum antibiotics once blood cultures have been obtained. Infective endocarditis can lead to acute heart failure, which may be worsened with excessive volume administration, so you must carefully consider how much volume is given according to that patient’s volume status on exam. An echocardiogram should be performed to evaluate for vegetations on the heart valves, evidence of valvular regurgitation, or evidence of left ventricular dysfunction (decreased ejection fraction). An ECG should be performed to evaluate for atrioventricular conduction delay as the infection can involve the electrical system of the heart leading to varying degrees heart block. Other complications of endocarditis include septic emboli, most frequently to the brain, which can lead to stroke and/or brain abscess formation. Laboratory evaluation will often reveal a leukocytosis, an elevated erythrocyte sedimentation rate, and C-reactive protein. Rheumatoid factor can also be positive.

SECTIO N II: CLINICAL CASES

369

Staphylococci and streptococci account for 80% of infective endocarditis cases. Initial empiric antibiotic therapy should include an antimicrobial with activity against methicillin-resistant Staphylococcus aureus, such as vancomycin, and the addition of either cefepime or a carbapenem if gram-negative bacteria are suspected or if the patient has prosthetic heart valves. The final antibiotic choice should be based on the sensitivities of the causative microorganism if identified through culture data. Patients are typically treated with intravenous antibiotics for 6 weeks. Surgical valve repair or replacement should be pursued if the patient has moderate to severe heart failure, severe aortic or mitral regurgitation, perivalvular abscess or fistula formation, fungal endocarditis, or very large vegetations.

APPROACH TO: Inf cti endocarditis DeFINITIONS IN f e c t Iv e e N d o c a r d It Is : Infection of the endocardium, typically the heart valves. r o t h s po t s : Exudative retinal hemorrhages, often with a pale or white center. o s l e r N o d e s : Painful, palpable, red and purple lesions found on the pulps of the fingers and toes. Ja N e w a y l e s Io N s : Flat, erythematous, nonpainful macules on the palms and soles. s pl IN t e r h e Mo r r h a g e s : Small dark red or brown linear lesions found underneath the fingernail.

CLINICAL APPROACH Etiology Of all infective endocarditis cases, 80% u b s i n s i i , in u in n u . Other causative microorganisms include members of the h a c e K u ni m (Haemophilus, A ggregatibacer, Cardiobacterium, Eikenella, and Kingella), bartonella, brucella, and Coxiella burnetii. Less common causes include other gram-negative bacteria and fungi, including candida and aspergillus. Risk factors include prosthetic heart valves, other intracardiac devices (such as pacemakers), rheumatic heart disease, unrepaired congenital heart disease, underlying structural valvular abnormalities, intravenous drug use, and hemodialysis.

Clinical Presentation M i n (80%) i in i n ii n i . Other presenting symptoms can include malaise, fatigue, weight loss, chills, and night sweats. Patients may present with symptoms of heart failure, including dyspnea on exertion, orthopnea, paroxysmal nocturnal dyspnea, and peripheral edema. Depending on the

370

CASE FILES: CARDIO LOGY

microbiologic cause, some patients may present with weeks to months of symptoms, while others will present much more rapidly, on the order of days. In addition to the above mentioned symptoms, the history should focus on risk factors for developing endocarditis. Any cause of bacteremia, such as central venous lines or hemodialysis catheters, can lead to endocarditis, especially in patients with abnormal or prosthetic heart valves. Comorbid conditions can also increase the likelihood of infective endocarditis, including diabetes mellitus, HIV, and any cause of immunosuppression. Intravenous drug use is a major risk factor for developing endocarditis, particularly in younger patients. Iv u u m mm n i -i n ii ( i u i u m ni ) in comparison to non-IV drug users who typically get left-sided endocarditis (mitral or aortic valve). The physical exam should first focus on the patient’s vital signs and the presence or absence of hypotension or shock. The patient’s volume status must be carefully assessed because the in i n n n i n n in u um i n i imu n u u in u iu u u u u i i n. The patient may display signs of heart failure, including jugular venous distention, peripheral edema, and pulmonary rales. A careful cardiovascular exam might reveal a regurgitant murmur. Tricuspid and mitral regurgitations are systolic murmurs, whereas aortic and pulmonic regurgitations are both diastolic murmurs. The exam should also include a search for i i m in i n ii in u r ,o n ,Jn i n, n in m. Roth spots are exudative retinal hemorrhages often with a pale or white center. Osler nodes are painful, palpable, red and purple lesions found on the pulps of the fingers and toes. Janeway lesions are flat, erythematous, nonpainful lesions of the palms and soles. Splinter hemorrhages, which are not specific for endocarditis, are small dark red or brown linear lesions found underneath the fingernail in the nailbed. Laboratory workup typically reveals a leukocytosis as well as an elevated erythrocyte sedimentation rate and C-reactive protein. Rheumatoid factor is often positive. Electrolyte abnormalities and renal failure may also be seen. Urinalysis may reveal hematuria. Chest x-ray may reveal pulmonary cavitations or consolidations from septic emboli.

Diagnosis The clinical diagnosis of infective endocarditis is often based on the symptoms of the patient, laboratory findings, and imaging results. The i i n i u b b in i b u u b u u m n in in b mini i n n ibi i . Identification of the causative agent will greatly assist in the final choice of antibiotic. Blood cultures should be repeated every 24 hours until there is no evidence of bacteremia. An electrocardiogram should be obtained to evaluate for any conduction abnormalities such as i n i u b k. Echocardiography should be pursued early. In general, a transthoracic echo is performed first to evaluate for any vegetations on the heart valves. However, if the transthoracic echo is negative and the suspicion for infective endocarditis remains high, or if the patient has prosthetic heart valves, then a transesophageal echo should be performed.

SECTIO N II: CLINICAL CASES

371

A ini i n i in i n i i nb m u in d uk i i , which include two major and five minor criteria. The first major criterion requires sustained positive blood cultures from a microorganism known to cause endocarditis or alternatively, a single positive blood culture or serology for Coxiella burnetii. The second major criterion requires evidence of endocardial involvement, which can be fulfilled by either new valvular regurgitation or echocardiographic evidence of valvular vegetations. The five minor criteria are (1) fever, (2) a predisposing condition (including IV drug use), (3) vascular phenomena, (4) immunologic phenomena, and (5) positive blood cultures that do not meet the first major criterion. Vascular phenomena include Janeway lesions, arterial emboli, septic pulmonary emboli, conjunctival hemorrhage, intracranial hemorrhage, and mycotic aneurysm. Immunologic phenomena include Roth spots, Osler nodes, a positive rheumatoid factor, and glomerulonephritis. A i n i in i n i i can be made based on any of the following: mj i i , n mj i i n, n min i i , i min i i (see Table 28-1).

Treatm ent The

372

CASE FILES: CARDIO LOGY

Fi r 28-1. Intraoperative photograph of prosthetic aortic valve endocarditis. (Reproduced, with permission, from Jose Navia, M.D.)

to severe heart failure, severe aortic or mitral regurgitation, perivalvular abscess or fistula formation, prosthetic valve dehiscence or perforation, fungal endocarditis, and very large vegetations (>10 mm) to prevent embolization (Figures 28-1 and 28-2). Recent data have suggested that early surgery in patients with infective endocarditis and large vegetations has led to a decrease in mortality and embolic events.

Com plications In-hospital mortality for patients with infective endocarditis is about 15–20%, and 5-year mortality approaches 40%. The development of systolic heart failure is a very poor prognostic sign, with a mortality of >50% if there is no surgical intervention. Destruction of the valves can occur, which can lead to acute valvular insufficiency,

Fi

r 28-2. Aortic valve vegetations

ex vivo. (Reproduced, with perm ission, from Jose Navia, M.D.)

SECTIO N II: CLINICAL CASES

373

valvular dehiscence, and perforation. Abscesses and fistulas can form, which can lead to further valvular insufficiency or disruption of the conduction system. This can lead to varying degrees of heart block, including complete heart block. Systemic embolization is a major complication of infective endocarditis. The central nervous system is the most common and the most severe location for septic emboli. This can lead to ischemic or hemorrhagic strokes and the development of brain abscesses. The n n ki n mm n i i mb i. Systemic embolization most often occurs in left-sided endocarditis, although it can occur in right-sided endocarditis in the setting of a patent foramen ovale. Pulmonary septic emboli occur more frequently in right-sided endocarditis.

Prophylaxis Antimicrobial prophylaxis is recommended for certain patients who are at high risk for developing endocarditis before certain procedures with high rates of postprocedural infectious complications. The current guidelines recommend that patients with i , i in i n ii, n n ni i , n i n n i n i u u i i n u b i n n ibi i xi b un in in i n u . The guidelines recommend against antibiotic prophylaxis before nondental procedures, including upper endoscopy, colonoscopy, and transesophageal echo for all patients. The most common choice for prophylaxis is amoxicillin 2 g by mouth 30–60 minutes before the procedure. Cephalexin, azithromycin, and clindamycin are acceptable alternatives, but dosage should be based on the patient’s history of allergies.

CASe CORReLATION • See also Case 6 (acute valvular regurgitation), Case 7 (chronic valvular regurgitation), and Case 10 (valvular stenosis).

COMPReHeNSION Qu eSTIONS 28.1 A 55-year-old man with severe mitral regurgitation is undergoing a screening colonoscopy in 2 weeks. He also has a history of hypertension, gastroesophageal reflux, and osteoarthritis. His medications include lisinopril, chlorthalidone, omeprazole, and acetaminophen. He is allergic to penicillin. Prior to undergoing the colonoscopy, he should be prescribed which of the following: A. Amoxicillin 2 grams per os (PO) 1 hour before the procedure B. Metronidazole 1 gram PO 1 hour before the procedure C. Clindamycin 600 mg PO 1 hour before the procedure D. No antibiotics

374

CASE FILES: CARDIO LOGY

28.2 A 76-year-old woman with severe aortic stenosis status following aortic valve replacement presents with left-sided facial droop, fever, and malaise. On exam, she is febrile to 100.5°F, is tachycardic to 105 bpm, and has a blood pressure of 105/60 mmHg. She is lethargic and has a III/VI early diastolic murmur in the aortic position. Her neurologic exam reveals left-sided facial droop. Laboratory workup shows a leukocytosis to 17,000 cells/mm3 and a creatinine of 2.0 mg/dL. In addition to IV fluids, antibiotics, and a head CT, the patient should undergo which diagnostic test: A. Transthoracic echocardiogram B. Bilateral carotid ultrasound C. Transesophageal echocardiogram D. CT angiogram of the head and neck 28.3 A 31-year-old IV drug user presents to the emergency department with fevers, chills, and night sweats. He is febrile, tachycardic, and hypotensive. His exam also reveals retinal hemorrhages with a pale white center, and he has several linear deposits in the nailbed of his right index and middle fingers. He is admitted to the hospital and given IV fluids and vancomycin. Serial sets of blood cultures are positive for Staphylococcus aureus. Transthoracic echocardiogram does not reveal any evidence of valvular vegetations. His ECG shows sinus tachycardia with a prolonged PR interval. What is the next step in management? A. Transesophageal echocardiogram B. Chest CT with contrast C. Repeat transthoracic echo in 72 hours D. Urine drug screen

ANSWeRS 28.1 d . The patient does not require any antibiotic prophylaxis for infective endocarditis prior to his screening colonoscopy. The current guidelines recommend that patients with prosthetic heart valves, a history of infective endocarditis, congenital heart disease, and cardiac transplant patients with valvular regurgitation be given antibiotic prophylaxis before undergoing certain dental procedures. The guidelines recommend against antibiotic prophylaxis before nondental procedures, including upper endoscopy, colonoscopy, and transesophageal echo for all patients. 28.2 c . The patient should undergo a transesophageal echocardiogram to evaluate for infective endocarditis. Patients with prosthetic heart valves who are suspected to have infective endocarditis should undergo transesophageal echocardiogram instead of transthoracic echo. Although she is presenting with facial droop, a carotid ultrasound or a CT of the head and neck would not be the next diagnostic test of choice, given the high degree of suspicion for endocarditis.

SECTIO N II: CLINICAL CASES

375

28.3 a . The patient has a definite diagnosis of infective endocarditis, fulfilling one major criterion (sustained positive blood cultures of a microorganism known to cause endocarditis) and four minor criteria (fever, IV drug use, vascular phenomena, and immunologic phenomena) of the Duke criteria. His prolonged PR interval is concerning for perivalvular extension of the infection, such as an abscess, invading the conduction system of the heart. This is best evaluated with an urgent transesophageal echocardiogram.

CLINCIAL Pe ARLS C

Ninety percent of patients with infective endocarditis have positive blood cultures, and staphylococci and streptococci represent 80% of identified causative microorganisms.

C

The Duke criteria can be used to make a clinical diagnosis of infective endocarditis.

C

Indications for surgery include moderate to severe heart failure, severe aortic or mitral regurgitation, perivalvular abscess or fistula formation, valvular dehiscence or perforation, fungal endocarditis, and very large vegetations (>10 mm) to prevent embolization.

C

Complications of infective endocarditis include septic emboli, acute systolic heart failure, acute valvular regurgitation, perivalvular abscess and fistula, and atrioventricular conduction delays.

C

The current guidelines recommend that patients with prosthetic heart valves, a history of infective endocarditis, congenital heart disease, and cardiac transplant patients with valvular regurgitation be given antibiotic prophylaxis before undergoing invasive dental procedures.

ReFeReNCeS Baddour LM, Wilson WR, Bayer AS, et al. Infective endocarditis: Diagnosis, antimicrobial therapy, and management of complications: A statement for healthcare professionals from the committee on rheumatic fever, endocarditis, and Kawasaki disease, Council on Cardiovascular Disease in the Young, and the Councils on Clinical Cardiology, Stroke, and Cardiovascular Surgery and Anesthesia, American Heart Association: Endorsed by the Infectious Diseases Society of America. Circulation. 2005;111(23):e394–e434. Hoen B, Duval X. Clinical practice. infective endocarditis. N Engl J Med. 2013;368(15):1425–1433. Kang DH, Kim YJ, Kim SH, et al. Early surgery versus conventional treatment for infective endocarditis. N Engl J Med. 2012;366(26):2466–2473. Nishimura RA, Carabello BA, Faxon DP, et al. ACC/AHA 2008 guideline update on valvular heart disease: Focused update on infective endocarditis: A report of the American College of Cardiology/ American Heart Association task force on practice guidelines endorsed by the Society of Cardiovascular Anesthesiologists, Society for Cardiovascular Angiography and Interventions, and Society of Thoracic Surgeons. Catheter Cardiovasc Interv. 2008;72(3):E1–E12.

This page intentionally left blank

CASE 29 A 37-year-old woman presents to her primary care physician with a 9-month history of swelling in her legs. She admits to cutting down on her weekly exercise routine as she “feels extremely tired with exercise.” Her medical history is significant for limited systemic sclerosis (SSc) diagnosed 8 years ago. On exam, vital signs are within normal limits. Heart auscultation reveals a loud P2, and a systolic murmur at the left sternal border that increases with inspiration. Jugular venous pressure is elevated at 8 cm H 2O. The lungs are clear to auscultation. Examination of the lower extremities reveals 2+ pitting edema that extends to the midleg. Chest x-ray shows mildly dilated pulmonary arteries but is otherwise normal. C C C

What is the most likely diagnosis? What is the best next diagnostic step? What is the best next therapeutic step?

378

CASE FILES: CARDIO LOGY

ANSWER TO CASE 29: P l onary Hypertension Summary: A 37-year-old woman with a history of limited systemic sclerosis (SSc) is now presenting with signs and symptoms consistent with right-heart failure. • M

k

• N x • N x

n n p

: Pulmonary arterial hypertension (PAH). p: Echocardiogram. p: Pulmonary vasodilators in addition to diuretics.

ANALYSIS Objectives 1. Understand the classification of pulmonary hypertension (PH). 2. Recognize the integral role of right-heart catheterization in the diagnosis of PH. 3. Interpret hemodynamic values based on heart catheterization. 4. Recognize the relatively poor prognosis of this disease despite various therapeutic options.

Considerations This is a young woman who is presenting with signs and symptoms of right-heart failure. Her presenting symptom is lower limb edema. The differential diagnosis of bilateral lower limb edema commonly includes venous insufficiency, nephrotic syndrome, cirrhosis, and heart failure. Her exertional symptoms and diminished functional capacity, in addition to the jugular venous distension, make heart failure more likely. A common cause of right-heart failure seen in clinical practice is actually left-heart failure. Given the clear lung exam and lack of pulmonary edema on chest x-ray, PAH is the likely cause of the right-heart failure. Moreover, left-heart failure is usually caused by ischemic and structural diseases of the heart, entities that are rare in this age group.

APPROACH TO: P l onary Hypertension DEFINITIONS Pu l Mo N a r y c a Pi l l a r y w e d g e Pr e s s u r e (Pc w P): Value obtained during right-heart catheterization that reflects left atrial pressure and therefore filling pressures on the left side of the heart. Normal values range from 5 to 12 mmHg ( 25 mmHg. Pu l Mo N a r y v e N o u s h y Pe r t e N s i o N : Left-heart disease (left atrial hypertension) as the cause of PH. t r a N s Pu l Mo N a r y g r a d i e N t (t Pg ): PCWP – mPAP. Normal value < 12 mmHg.

CLLINICAL APPROACH Causes and Classification The n f Ph m n m n m b mPa P > 25 mmh . The current classification scheme utilizes differences in clinical features and treatment of the various groups (Table 29-1). An important diagnostic goal is to distinguish PAH from PH due to left-heart disease. The latter is referred to as p m n n p n n and has a distinctly different prognosis and therapeutic approach. It should be noted that chronic thromboembolic pulmonary hypertension (CTEPH), a complication of pulmonary hypertension that occurs in approximately 4% of patients with acute pulmonary emboli, is classified as a separate group because of its potential surgical curability. The normal physiologic response to hypoxia is vasodilation. A notable exception is the pulmonary circulation, in which hypoxia causes vasoconstriction in an attempt to redistribute blood to better-ventilated areas. This partly explains the pathophysiology of PH in lung disease (type 3 PH). It should also be noted that in all types of PH, as the right ventricle progressively fails, hypoxia ensues, leading to a vicious cycle.

Clinical Presentation Patients with PH usually present with insidious symptoms at a more advanced stage of the disease. Symptoms include shortness of breath and fatigue with exertion, as Table 29-1 • WHO CLINICAL CLASSIFICATION OF Pu LmONARY HYPERTENSION 1 P l onary arterial hypertension • Idiopathic • Heritable • Drugs and toxins induced (eg, anorexigens) • Associated with CTD (especially SSc), HIV, portal hypertension, congenital heart disease, schistosomiasis, and chronic hemolytic anemia • Persistent PH of newborn 1′ Pulmonary venoocclusive disease and/ or pulmonary capillary hemangiomatosis 2 PH d e to left-heart disease • Systolic dysfunction, diastolic dysfunction, and valvular disease 3 PH d e to l ng disease and/ or hypoxia • Obstructive lung disease, interstitial lung disease, sleep-disordered breathing 4 Chronic thro boe bolic PH 5 miscellaneo s • Hematologic disorders, sarcoidosis, and others Abbreviations: CTD, connective tissue disease; PH, pulmonary hypertension; SSc,systemic sclerosis.

380

CASE FILES: CARDIO LOGY

well as reduced exercise tolerance. Chest pain may occur even in the presence of normal coronaries due to right ventricular wall strain. With progression of disease and worsening of right-heart failure, patients develop peripheral edema and even syncope. s n f Ph n n f n j n p , p n (n n n p p ), p m n mp n n f n n , p n m m b f p n b n n n pn, , n x m m . Other signs are specific to the underlying etiology. Murmurs indicate valvular disease that is associated with pulmonary venous hypertension (eg, murmurs of aortic stenosis and mitral regurgitation). Lung exam may reveal fine crackles in interstitial lung disease, coarse crackles in left heart failure, or prolonged expiratory phase and reduced breath sounds in obstructive lung disease. Stigmata of liver disease, including spider angiomata, may be detected. Connective tissue diseases manifest with various skin and joint changes, such as skin thickening that occurs in SSc.

Diagnosis The echocardiogram is usually the first test to objectively identify elevated pulmonary pressures by estimating the pulmonary artery systolic pressure (PASP). Pa s P > 40 mmh pp x m n f mPa P > 25 mmh . An echocardiogram will also assess for the various causes of pulmonary venous hypertension, including left-sided valvular abnormalities and systolic and diastolic dysfunction. However, the diagnosis of PH and an accurate assessment of left- and right-sided filling pressures can be made only by pulmonary artery (right-heart) catheterization. This modality will directly measure mPAP. Furthermore, it allows assessment of filling pressures on the left side of the heart by measuring the PCWP, which will help to differentiate between PAH and pulmonary venous hypertension. Cardiac output and pulmonary vascular resistance can also be calculated. He odyna ics (Fig re 29-1) e mPa P, n m Pc w P: This excludes left-heart disease as a cause of PH. The diagnosis is PAH. e mPa P, Pc w P, n m t Pg : The normal TPG indicates that the elevated mPAP can be entirely explained by the PCWP, that is, left-heart failure resulting in PH and right-heart failure. This entity is referred to as p m n n p n n. e mPa P, Pc w P, t Pg : In this scenario the elevated TPG indicates that the mPAP is elevated out of proportion to the PCWP. It can be explained by one of two scenarios: (1) pulmonary venous hypertension combined with PAH, or (2) isolated pulmonary venous hypertension in which remodeling of the pulmonary vasculature has occurred as a protective mechanism against pulmonary edema. This results in a fixed PH that persists regardless of volume status. If PAH is confirmed by heart catheterization, the next step is to determine pulmonary vasodilator reactivity. An acute pulmonary vasodilator

SECTIO N II: CLINICAL CASES

381

mPAP >25

Ye s

P CWP 180/120 mmHg) with evidence of acute end-organ damage. Typical emergencies include encephalopathy, thrombotic or hemorrhagic cerebrovascular accident, acute aortic dissection, myocardial ischemia/infarction, acute renal failure, retinal hemorrhages or papilledema, and eclampsia. Severe hypertension without the presence of endorgan damage does not constitute a hypertensive emergency. This syndrome implicates immediate lowering of the patient’s blood pressure, typically with parenteral agents, while monitoring the patient in an intensive care unit (ICU). h y pEr TEN SIv E u r g EN c y: Unlike a hypertensive emergency, this is defined as severe hypertension without end-organ damage. Blood pressure, therefore, can be lowered more slowly over a period of days to weeks. Patients may be treated with oral antihypertensives and typically treated as outpatients.

CLINICAL APPROACH Initial Evaluation and Management High-to-normal blood pressures ranging from 120 to 139 mmHg systolic and 80 to 89 mmHg diastolic are considered to be e e tensi n (Table 30-1). This entity

390

CASE FILES: CARDIO LOGY

Table 30-1 • HYPERTENSION CLASSIFICATION FOR ADu LTS Blood Press re Cate ory

Systolic Blood Press re ( H )

Diastolic Blood Press re ( H )

Normotension

< 120

and

< 80

Lifestyle modifications advised

Prehypertension

120–139

or

80–89

Lifestyle modifications essential

Stage I hypertension

140–159

or

90–99

Lifestyle modifications and thiazide diuretic, unless other compelling indication

Stage II hypertension

>160

or

≥100

Lifestyle modifications and thiazide diuretic and additional antihypertensive medication, unless other compelling indication

Treat ent

does confer some increased risk for cardiovascular disease. St e I e tensi n is diagnosed with two or more consecutive blood pressure measurements of ≥140 mmHg systolic or ≥90 mmHg diastolic. St e II e tensi n is diagnosed with two or more consecutive blood pressure measurements of ≥160 mmHg systolic or ≥100 mmHg diastolic. Hypertension is also defined as the need for antihypertensive therapy. The st n e tensi n is i i t i e tensi n, w i is t e se xi te 95% i b ess e. The remaining 5–7% of cases of hypertension are related to secondary causes of hypertension (Table 30-2),

Table 30-2 • COmmON SECONDARY CAu SES OF HIg H BLOOD PRESSu RE Chronic renal diseases Parenchymal (glomerulonephritis, diabetic nephropathy, hypertensive nephrosclerosis, polycystic kidney disease) Renovascular (renal artery stenosis, fibromuscular dysplasia) Obstructive uropathy Endocrinopathy Hyperthyroidism Cushing syndrome and other states with gluocorticoid excess, including chronic steroid use Primary aldosteronism and other states with excess mineralocorticoid levels Pheochromocytoma Oral contraceptive usage Acromegaly Parathyroid disease miscellaneo s Coarctation of the aorta Obstructive sleep apnea Medication-induced hypercalcemia

SECTIO N II: CLINICAL CASES

391

which may be reversible. Therefore, it is of great importance to be familiar with the presenting findings and potential laboratory abnormalities that may be associated with various causes of secondary hypertension. Secondary causes of hypertension should be suspected when patients present with: (1) hypertension at age 55 years; (2) previously well-controlled hypertension that becomes poorly controlled; (3) sudden-onset or aggressive hypertension; or (4) an abdominal aortic bruit and clinical features suggestive of bilateral renal artery stenosis. Clinical features of renal artery stenosis include resistant hypertension (hypertension requiring the use of three or more antihypertensive medications), multiple episodes of flash pulmonary edema, renal failure with unknown cause (especially with normal urine sediment or without proteinuria), and renal failure associated with the use of either angiotensin-converting enzyme inhibitors (ACEIs) or angiotensin-receptor blockers (ARBs).

Cardiac Risk Factors and Evaluation for Target Organ Damage One of the hallmarks of treating a patient with newly diagnosed hypertension is e ti n n t e t ent t e i isk t s in addition to identification of en n s n ti n (Table 30-3). Results from these investigations will guide therapy. Classical risk factors for the development of cardiovascular diseases include age, gender, smoking status, diabetes mellitus, LDL-C, HDL-C, and presence of hypertension. Other factors that determine risk include chronic renal diseases and family history of premature coronary heart disease. In order to determine end-organ damage to the eyes, kidneys, heart, brain, and peripheral vasculature, studies including urinalysis, complete blood cell count, blood chemistries, and ECGs need to be completed (Table 30-4). Treatment of hypertension is guided by the level of hypertension in addition to other cardiac risk factors or the development of endorgan damage. Table 30-3 • COmPELLINg INDICATIONS THAT g u IDE HYPERTENSION THERAPY AND TARg ET ORg AN DIAg NOSES CAu SED BY HYPERTENSION Co pellin indications Coronary artery disease Cerebrovascular disease Chronic heart failure Diabetes mellitus Chronic renal insufficiency Valvular heart disease Peripheral vascular disease E idence for tar et or an da a e Coronary artery disease Left ventricular hypertrophy Prior myocardial infarction Angina pectoris Chronic heart failure Transient ischemic attack of stroke Nephropathy Peripheral vascular disease Retinopathy

392

CASE FILES: CARDIO LOGY

Table 30-4 • LABORATORY TESTINg AND STu DIES IN THE EvALu ATION OF HYPERTENSION Reco ended Urinalysis Complete blood cell count Chemistries 12-lead electrocardiogram Additional Creatinine clearance Microalbuminuria 24-hour urinary protein Blood calcium Uric acid Fasting triglyceride LDL-C Hemoglobin A1c TSH Echocardiography

Therapy The t e t ent e tensi n e es t e in i en e e tensi n essi n, e t i e, st ke, n ni en ins i ien . In addition to medical therapy, weight reduction, exercise, and diet are essential to lowering blood pressure. When initiating pharmacologic treatment, a low dose of drug should be chosen, then titrated upward on a schedule reflecting the patient’s age, response, and medical need. The optimal target for blood pressure lowering is typically

Case Files Cardiology pdf am-medicine com [PDF] | Online Book Share (2024)

References

Top Articles
Latest Posts
Article information

Author: Patricia Veum II

Last Updated:

Views: 5611

Rating: 4.3 / 5 (64 voted)

Reviews: 95% of readers found this page helpful

Author information

Name: Patricia Veum II

Birthday: 1994-12-16

Address: 2064 Little Summit, Goldieton, MS 97651-0862

Phone: +6873952696715

Job: Principal Officer

Hobby: Rafting, Cabaret, Candle making, Jigsaw puzzles, Inline skating, Magic, Graffiti

Introduction: My name is Patricia Veum II, I am a vast, combative, smiling, famous, inexpensive, zealous, sparkling person who loves writing and wants to share my knowledge and understanding with you.